Download as pdf or txt
Download as pdf or txt
You are on page 1of 167

histology

LECTURE / INSTRUCTOR: JURA SORNILLO JUNQUEIRA’S BASIC HISTOLOGY TEXT AND ATLAS

CHAPTER 1: HISTOLOGY AND ITS METHODS OF STUDY Biopsy - tissue samples removed during surgery which are fixed
Histology – study of tissues of the body and how it is arranged in vials of formalin for processing and microscopic analysis in a
to make an organ. Also focuses on the optimization of cell’s pathology laboratory. It is rapidly frozen in liquid nitrogen
function through its structure and arrangements. because it, does not inactivate most enzymes and does not
dissolve cell lipids.
Two interacting components of tissue that function together in
Cryostat – a microtome with a subfreezing temperature used to
well-coordinated manner:
cut sections rapidly.
1. Cells – produce ECM Staining – used to give color since most cells and EXM are
2. Extracellular matrix (ECM) – consists of macromolecules completely colorless.
that forms complex structures. Supports the cells in terms Dyes stain materials - behaves like acidic or basic compounds
of fluid transportation as well as carrying away wastes and and forming electrostatic (salt) linkages with ionizable radicals
secretory products. of macromolecules in tissues.
Basophilic - net negative charge (anionic) components have an
PREPARATION OF TISSUES affinity for basic dyes (toluidine blue, alcian blue, and methylene
Preparation of Tissues is conducted through sections since blue).
most of them are thick for light to pass through. *Hematoxylin behaves like a basic dye, staining basophilic tissue
1. Fixation - preserves tissue structure and prevents components.
degradation by enzymes through fixatives. Acidophilic – net positive charge (cationic) components have an
- small pieces of tissue are placed in solutions of chemicals that affinity for acidic dyes (eosin, orange G, and acid fuchsin).
cross-link proteins and inactivate degradative enzymes, which
preserves cell and tissue structure. Hematoxylin and eosin (H&E) -
Fixative - solutions made up of stabilizing or cross-linking Hematoxylin stains DNA in the cell
compounds. react with the amine groups (NH2) of proteins, nucleus, RNA-rich portions of the
preventing their degradation by common proteases. cytoplasm, and the matrix of cartilage,
Formalin – buffered isotonic solution of 37% formaldehyde producing a dark blue or purple color;
and is widely used fixative for light microscopy. Eosin stains other cytoplasmic
Glutaraldehyde – fixative used for electron microscopy. structures and collagen pink.
*Glutaraldehyde treated tissue is then immersed in buffered
osmium tetroxide, which preserves (and stains) cellular lipids as Periodic Acid-Schiff (PAS) reaction -
well as proteins. utilizes the hexose rings of
2. Dehydration – the water is removed using increasingly polysaccharides and other
concentrated alcohol solutions (50%, 70%, 90%, 100%). carbohydrate-rich tissue structures
3. Clearing - alcohol is removed in organic solvents in which and stains such macromolecules
both alcohol and paraffin are miscible. It gives the tissue a distinctly purple or magenta.
translucent appearance.
Feulgen reaction – stains DNA of cell nuclei.
4. Infiltration - the tissue is then placed in melted paraffin (52˚C-
*Heat and organic solvents remove lipids.
60˚C) until it becomes completely infiltrated with paraffin.
Lipid-soluble dyes (Sudan black) – are used to stain lipids
5. Embedding - the paraffin-infiltrated tissue is placed in a small
which can be useful in diagnosis of metabolic diseases that
mold with melted paraffin and allowed to harden at room
involve intracellular accumulations of cholesterol,
temperature.
phospholipids, or glycolipids.
6. Trimming - the resulting paraffin block is trimmed to expose
Metal impregnation - uses solutions of silver salts to visual
the tissue for sectioning (slicing) on a microtome.
certain ECM fibers and specific cellular elements in nervous
Paraffin - used routinely for light microscopy. tissue.
Plastic resins - used for both light and electron microscopies. *Slide preparation, from tissue fixation to observation with a light
Microtome - used for sectioning paraffin-embedded tissues for microscope, may take from 12 hours to 2½ days, depending on the
light microscopy. size of the tissue, the embedding medium, and the method of
staining.
* 3-10 μm thickness for light microscopy.
Mounting of coverslip on the side with clear adhesive - final step
*Less than 1 μm thick for electron microscopy.
before microscopic observation.
AMUAN, BERJAMIN, ERMITANIO, LARIOSA, RESURRECCION – BSMLS2B 1
histology
LECTURE / INSTRUCTOR: JURA SORNILLO JUNQUEIRA’S BASIC HISTOLOGY TEXT AND ATLAS

LIGHT MICROSCOPY *Fluorescent compounds with affinity for specific cell


Light microscopy - interaction of light with tissue components macromolecules may be used as fluorescent stains.
and are used to reveal and study tissue features.
Acridine orange - binds both DNA and RNA
Bright-field microscope - stained tissue which emit slightly different fluorescence,
is examined with ordinary light passing allowing them to be localized separately in
through the preparation. It includes an cell.
optical system and mechanisms to move
and focus the specimen.

Optical components: DAPI and Hoechst stain - specifically bind


1. Condenser - collects and focuses a cone of light that DNA and are used to stain cell nuclei,
illuminates the tissue slide on the stage. emitting a characteristic blue fluorescence
2. Objective lens - enlarging and projecting the image of the under UV.
object toward the observer; In a light microscope, resolution and Phase-Contrast microscopy - uses a
magnification of cells are largely dependent on this part of the lens system that produces visible
microscope. images from transparent objects and,
Magnifications: importantly, can be used with living,
X4 - observing a large area (field) of the tissue at low cultured cells without fixation or
magnification. staining. Light changes its speed when
X10 - medium magnification of a smaller field. passing through cellular and
X40 - high magnification of more detailed areas. extracellular structures with different refractive indices. These
3. Eyepiece (or ocular lens) - further magnifying (X10) the changes are used by the phase-contrast system to cause the
image and projecting it onto the viewer’s retina or a charge- structures to appear lighter or darker in relation to each other.
coupled device (CCD) highly sensitive to low light levels with a
camera and monitor. Differential interference microscopy
*Total magnification = objective x ocular lenses. (with Nomarski optics) - produces an
Resolving power - smallest distance between two structures at image of living cells with a more
which they can be seen as separate object. Determines the apparent three-dimensional (3D) aspect.
quality of the image, its clarity and richness of detail, and
depends mainly on the quality of its objective lens.
*Maximal resolving power of the light microscope (LM) is
approximately 0.2 μm. Smaller or thinner objects can’t be Confocal microscopy - reduces
distinguished by LM). the stray lights which reduces
Virtual microscopy - conversion of a stained tissue preparation the contrast within the image by
to high-resolution digital images and permits study of tissues using a small point of high-
using a computer or other digital device, without an actual intensity light (laser) and a plate
stained slide or a microscope. with a pinhole aperture in front of
Advantages: the image detector where in it
 Cost achieves s high resolution and
 Ease of use sharp focus.
*Virtual microscopy is rapidly replacing light microscopes and
collections of glass slides. *The point light source, the focal point of the lens, and the
detector’s pinpoint aperture are all optically conjugated or
Fluorescence Microscopy - tissue sections are usually aligned to each other in the focal plane where no unfocused light
irradiated with ultraviolet (UV) light and the emission is in the could pass through the pinhole.
visible portion of the spectrum. The fluorescent substances
appear bright on a dark background.
Fluorescence - cellular substances are irradiated by light of a
proper wavelength, they emit light with a longer wavelength.

AMUAN, BERJAMIN, ERMITANIO, LARIOSA, RESURRECCION – BSMLS2B 2


histology
LECTURE / INSTRUCTOR: JURA SORNILLO JUNQUEIRA’S BASIC HISTOLOGY TEXT AND ATLAS

Polarizing Microscopy - allows the resolution since it binds cellular macromolecules, increasing
recognition of stained or unstained their electron density and visibility.
structures made of highly organized Cryofracture and freeze etching - techniques that allow TEM
subunits. Polarizing filter – normal study of cells without fixation or embedding since the tissue is
light vibrates when it passes rapidly frozen in liquid nitrogen and then cut or fractured with a
through. knife. A replica of the frozen exposed surface is produced in a
*If a second filter is placed in the vacuum by applying thin coats of vaporized platinum or other
microscope above the first one, with metal atoms. This split the lipid bilayers, exposing protein
its main axis perpendicular to the first filter, no light passes components whose size, shape, and distribution.
through. If, however, tissue structures containing oriented
Scanning Electron Microscopy
macromolecules are located between the two polarizing filters,
(SEM) - provides a high resolution
their repetitive structure rotates the axis of the light emerging
view of the surfaces of cells, tissues,
from the polarizer and they appear as bright structures against
and organs but it does not pass
a dark background.
through the specimen. The surface
Birefringence - ability to rotate the direction of vibration of
of the specimen is first dried and
polarized light by crystalline substances or substances
spray-coated with a very thin layer of
containing highly oriented molecules, such as cellulose,
heavy metal (gold) which reflects
collagen, microtubules, and actin filaments.
electrons in a beam scanning the
ELECTRON MICROSCOPY specimen. The reflected electrons
Electron microscopy - interaction of tissue components with (secondary electrons) are captured
beams of electrons. by a detector, producing signals that
*The wavelength in an electron beam is much shorter than that are processed to produce a 3D black-
of light, allowing a 1000-fold increase in resolution. and-white image.

Transmission Electron Microscope AUTORADIOGRAPHY


Autoradiography -localizing newly synthesized macromolecules
(TEM) - imaging system like an
in cells or tissue sections
upside-down microscope has a
metallic (tungsten) filament cathode Radioactively Labeled Metabolites - incorporated into specific
that emits electron which travels to macromolecules (DNA, RNA, protein, glycoproteins, and
an anode at an accelerating voltage polysaccharides) and emit weak radiation that is restricted to
of 60 and 120 kV that permits those regions where the molecules are located.
resolution around 3nm that allows Silver bromide crystals – photographic emulsion that act as
isolated particles magnified as microdetectors of the radiation in the same way that they
much as 400,000 times to be respond to light in photographic film. It reduced by the radiation
viewed in detail using very thin produce small black grains of metallic silver, which under either
(40-90 nm), resin-embedded tissue the light microscope or TEM indicate the locations of
sections. radiolabeled macromolecules in the tissue.
*A beam of electrons focused using Silver grains - indicate the cells or regions of cells in which
electromagnetic “lenses” (hole in the anode) passes through the specific macromolecules were synthesized just prior to fixation.
tissue section to produce an image with black, white, and
intermediate shades of gray regions.
Brighter or electron-lucent - electrons passed readily.
Darker or electron-dense - electrons were absorbed or
deflected.
*Heavy metal ions (osmium tetroxide, lead citrate, and uranyl
compounds) are often added to the fixative or dehydrating LIGHT MICROSCOPE TEM
solutions used for tissue preparation to improve contrast and

AMUAN, BERJAMIN, ERMITANIO, LARIOSA, RESURRECCION – BSMLS2B 3


histology
LECTURE / INSTRUCTOR: JURA SORNILLO JUNQUEIRA’S BASIC HISTOLOGY TEXT AND ATLAS

CELL AND TISSUE CULTURE Peroxidase - promotes the oxidation of substrates with the
Cell culture - direct observation of cellular (living cells & transfer of hydrogen ions to hydrogen peroxide. (b)
tissues) behavior under a phase-contrast microscope. Dehydrogenases - transfer hydrogen ions from one substrate
“in vitro” – in glass; outside normal biological context. to another, such as many enzymes of the citric acid (Krebs)
“in vivo” – within the living. Cells are bathed in fluid derived cycle, allowing histochemical identification of such enzymes in
from blood plasma and containing many different molecules mitochondria.
required for survival and growth. SPECIFIC MOLECULES
Primary cell cultures – is a preparation of cells where in cells Tagged compounds or macromolecules – used in identifying
are grown in complex solutions of known composition (salts, specific macromolecule in a tissue section which it binds
amino acids, vitamins) to which serum or specific growth specifically. It must be visible with the light or electron
factors are added. They are then dispersed mechanically or microscope which are tagged with detectible label.
enzymatically from a tissue or organ and placed with sterile Examples:
procedures in a clear dish to which they adhere, usually as a Phalloidin - compound extracted from mushroom, (Amanita
single layer. phalloides), interacts strongly with the actin protein of
Cell line – part of cells that was maintained in vitro for long microfilaments.
periods of time and become immortalized. Protein A - purified from (Staphylococcus aureus) bacteria. It
Transformation – process where in changes in the finite, binds to the Fc region of antibody molecules, and can therefore
genetically programmed life span of cells promotes cell be used to localize naturally occurring or applied antibodies
immortality. bound to cell structures.
ENZYME HISTOCHEMISTRY Lectins - glycoproteins from plant seeds, bind to carbohydrates
Enzyme Histochemistry (cytochemistry) - method for localizing with high affinity and specificity.
cellular structures using a specific enzymatic activity present in
those structures. The tissues used are mildly fixed or unfixed Immunohistochemistry - highly specific interaction between
and is sectioned using a cryostat since heat and organic macromolecules (antigen and its labeled antibody) which are
solvents has adverse effects on enzymic activities. used to identify and localize many specific proteins.
Process:
1. Tissue sections are immersed in a solution containing the Antibodies – product of body’s
substrate of the enzyme to be localized. immune cells after it interacts
2. Enzyme is allowed to act on its substrate. (bind specifically) against other
3. Section is then put in contact with a marker compound that macromolecules (protein) to
reacts with a product of the enzymatic action on the eliminate it. It belongs to the
substrate. immunoglobulin family of
4. The product from the marker, that is insoluble and visible glycoproteins and are secreted
by light or electron microscopy, precipitates over the site of by lymphocytes.
enzymes which identifies their location.
Examples: Antigens – macromolecules that are recognized as “foreign,”
Phosphatases not a normal part of the organism, and potentially dangerous.
- remove phosphate groups from macromolecules. (a)
Polyclonal antibodies - different groups (clones) of
lymphocytes are used to recognize different parts of protein x
where in each clone produces an antibody against its specific
part.

Hybridoma cells – isolated and activated lymphocytes which is


placed in a culture where in it will be fused with lymphocytic
tumor cells. They produce different clones that produce
different antibodies.

AMUAN, BERJAMIN, ERMITANIO, LARIOSA, RESURRECCION – BSMLS2B 4


histology
LECTURE / INSTRUCTOR: JURA SORNILLO JUNQUEIRA’S BASIC HISTOLOGY TEXT AND ATLAS

Monoclonal antibody - each In situ hybridization (ISH) -


clone can be isolated and Hybridization at stringent
cultured separately so that conditions allows the specific
the different antibodies identification of sequences in
against protein x can be genes or RNA. This occur with
collected separately. cellular DNA or RNA when nucleic
Advantage: acid sequences in solution are
It can be selected to be highly specific and to bind strongly to applied directly to prepared cells
the protein to be detected, with less nonspecific binding to other and tissue sections.
proteins that are similar to the one of interest. “in situ” – on site
Process: Importance:
1. Tissue section (contains the protein of interest) is incubated 1. Determining if a cell has a specific sequence of DNA, such
in a solution containing antibody (either monoclonal or as a gene or part of a gene.
polyclonal) against this protein. 2. Identifying the cells containing specific messenger RNAs
2. Antibody binds specifically to the protein. (mRNAs) (in which the corresponding gene is being
3. Protein’s location in the tissue or cells can be seen with transcribed).
either the light or electron microscope by visualizing the 3. Determining the localization of a gene in a specific
antibody after a rinse. chromosome.
*Antibodies are commonly tagged with fluorescent compounds, Process:
with peroxidase or alkaline phosphatase (histochemical 1. DNA and RNA of the cells must be initially denatured by heat
detection), or with electron dense gold particles (TEM). or other agents to become completely single-stranded.
2. Probe is used to detect nucleotide sequences of interest.
Probe - consisting of single-stranded complementary DNA
(cDNA). The probe may be obtained by cloning, polymerase
chain reaction (PCR) amplification of the target sequence, or
chemical synthesis if the desired sequence is short.
3. Probe is tagged with nucleotides containing a radioactive
isotope (autoradiography) or modified with a small
Direct method - just involves a labeled antibody that binds the
compound such as digoxigenin (immunocytochemistry).
protein of interest.
4. A solution containing the probe is placed over the specimen
Indirect method - involves sequential application of two
under conditions allowing hybridization and after the excess
antibodies and additional washing steps. It is more sensitive
unbound probe is washed of.
since there is an extra level of antibody binding that amplifies
5. localization of the hybridized probe is revealed through its
the visible signal.
label.
*More than one labeled secondary antibody can bind each
Microscopic preparations - end result of a series of processes
primary antibody molecule.
that began with collecting the tissue and ended with mounting a
Primary antibody – binds to the protein of interest but it is not
coverslip on the slide.
tagged (in indirect method).
Artifacts - minor structural abnormalities due to the distortion
Secondary antibody – has a
of the tissues.
detectable tag which attaches to
Shrinkage - create artificial spaces between cells and other
the primary antibody since it is
tissue components. Such spaces can also result from the loss
“foreign” and came from another
of lipids or low-molecular-weight substances not preserved by
species.
the fixative or removed by the dehydrating and clearing fluids.
Hybridization - specific binding *Slight cracks in sections may also appear as large spaces in
between two single strands of the tissue.
nucleic acid, which occurs under appropriate conditions if Small wrinkles - confused with linear structures in tissue.
the strands are complementary and produces double-strand Precipitates from the stain - confused with cellular structures
molecules. such as cytoplasmic granules.
*The more similar, the more hybrid.

AMUAN, BERJAMIN, ERMITANIO, LARIOSA, RESURRECCION – BSMLS2B 5


histology
LECTURE / INSTRUCTOR: JURA SORNILLO JUNQUEIRA’S BASIC HISTOLOGY TEXT AND ATLAS

Three-dimensional (3D) - cut into very thin sections, the  This is used to manipulate the stage to focus the image under the
microscope in LPO and Scanner. (Coarse Adjustment knob)
sections appear microscopically to have only two dimensions:  This part of the microscope controls the amount of the intensity and the
length and width. size of the beam of light that passes through the specimen. (Diaphragm)
*FRONT AND BACK PART ARE ALWAYS MISSING IN 3D  This type of Microscopy uses a lens system that produces visible images
from transparent objects and unstained tissue sections. (Phase-Contrast
Round structures - may actually be portions of spheres or Microscopy)
tubes.  In preparing tissue for routine light microscopic study, the tissue is placed
Two-dimensional (2D) - appearance will also vary depending on in melted paraffin until it becomes completely permeated with the
substance. (Infiltration)
the plane of section.  In a light microscope, resolution and magnification of cells are largely
dependent on which component? (Objective lenses)
 This type of Microscopy uses ordinary light to pass through the
preparation. (Bright-Field Microscopy)
 This histologic technique extracts the water gradually from the fixed tissue.
(Dehydration)
 This part of the microscope holds the objective lenses and allows
switching of the objectives to change magnification. (Revolving nosepiece)
 This histologic technique involves the addition of a solvent that is miscible
to both alcohol and embedding medium, and it gives the tissue a
translucent appearance. (Clearing)
 Cellular storage deposits of glycogen, a free polysaccharide, could be best
detected histologically by using what procedure? (Periodic Acid-Schiff
reaction)
 This is defined as the smallest distance between two structures at which
they can be seen as separate objects under the microscope. (Resolving
power)
 This simple combination is the most commonly used in staining.
(Hematoxylin and Eosin)
 This is used to manipulate the stage to focus the image under the
REVIEW: microscope in HPO and OIO. (Fine Adjustment knob)
 In preparing tissue for routine light microscopic study, which procedure  This part of the microscope holds the specimen used for viewing and has
immediately precedes clearing the specimen with an organic solvent? stage clips to hold the slide in place. (Mechanical Stage)
(Dehydration)
 Which of the following staining procedures relies on the cationic and
anionic properties of the material to be stained? (Hematoxylin & eosin
staining)
 In a light microscope used for histology, resolution and magnification of
cells are largely dependent on which component? (Objective lens)
 Cellular storage deposits of glycogen, a free polysaccharide, could best be
detected histologically using what procedure? (Periodic Acid-Schiff
reaction)
 Adding heavy metal compounds to the fixative and ultrathin sectioning of
the embedded tissue with a glass knife are techniques used for which
histological procedure? (Transmission electron microscopy)
 Resolution in electron microscopy greatly exceeds that of light microscopy
due to which of the following? (The wavelength of the electrons in the
microscope beam is shorter than that of a beam of light.)
 Microscopic autoradiography uses radioactivity and can be employed to
study what features in a tissue section? (Cellular sites where various
macromolecules are synthesized)
 To identify and localize a specific protein within cells or the extracellular
matrix one would best use what approach? (Immunohistochemistry)
 In situ hybridization is a histological technique used to visualize what type
of macromolecule? (Nucleic acids)
 Hospital laboratories frequently use unfixed, frozen tissue specimens
sectioned with a cryostat for rapid staining, microscopic examination, and
diagnosis of pathological conditions. Besides saving much time by
avoiding fixation and procedures required for paraffin embedding, frozen
sections retain and allow study of what macromolecules normally lost in
the paraffin procedure? (Lipids)
 In this type of Microscopy, tissue sections are irradiated with UV light and
the emission is in the visible portion of the spectrum. (Fluorescence
Microscopy)

AMUAN, BERJAMIN, ERMITANIO, LARIOSA, RESURRECCION – BSMLS2B 6


histology
LECTURE / INSTRUCTOR: JURA SORNILLO JUNQUEIRA’S BASIC HISTOLOGY TEXT AND ATLAS

CHAPTER 2: THE CYTOPLASM


Cells – basic structural and functional units, and the smallest
living parts of the body.
Eukaryotic – has nucleus surrounded by cytoplasm, a system of
membranous organelles, nonmembranous molecular
assemblies, and a cytoskeleton.
Prokaryotic – has a cell wall, lacks nuclei and membranous
cytoplasmic structures.
CELL DIFFERENTIATION
Blastomeres – cells produced by the first zygotic cellular
divisions.
Embryonic Stem cells – explanted to tissue culture cells of the
inner cell mass
Cell differentiation – the process by which cells of an embryo
become specialized structurally to augment specific cytoplasmic
activities for functions at the level of tissues and organs
Organelles - metabolically active structures or complexes, with
or without membranes, in the cytoplasm of eukaryotic cells.
PLASMA MEMBRANE
Plasma membrane (cell membrane or plasmalemma) - the lipid
bilayer with embedded proteins that surrounds a cell and is seen
only with the TEM.
molecules cross the plasma membrane through channel
Integrins (plasma membrane protein) - linked to both the
proteins called aquaporins.
cytoskeleton and ECM components; allow continuous exchange
Carriers - transmembrane proteins that bind small molecules
of influences
and translocate them across the membrane via conformational
Glycolipids - outer layer’s ipids extend outward from the cell
changes.
surface
Endocytosis - cellular uptake of macromolecules or fluid by
Glycocalyx - delicate cell surface coating
plasma membrane engulfment or invagination, followed by the
*The lipid bilayer forms from amphipathic phospholipids,
“pinching off” of a filled membranous vesicle in the cytoplasm.
stabilized by cholesterol, and contains:
*Major types of endocytosis include:
-- integral proteins - incorporated directly within the lipid bilayer
-- phagocytosis - uptake of particulate material
-- peripheral proteins - bound to one of the two membrane
-- pinocytosis - uptake of dissolved substances
surfaces on the cytoplasmic surface.
-- receptor-mediated endocytosis - uptake of specific
*The polypeptide chains of many integral proteins span the
molecules bound to integral membrane receptor proteins
membrane from one side to another several times and are
Exocytosis - a type of cellular secretion in which cytoplasmic
accordingly called multipass proteins
membrane vesicles fuse with the plasma membrane and release
*Membrane proteins move laterally within the lipid bilayer, with
their contents to the extracellular space.
less movement in areas referred to as lipid rafts, which have
Constitutive secretion - used for products that are released
higher concentrations of cholesterol and saturated fatty acids.
from cells continuously
*Integral membrane proteins include:
Regulated secretion - occurs in response to signals coming to
-- receptors for external ligands,
the cells (the release of digestive enzymes from pancreatic cells
-- channels for passive or active movement of molecules across
in response to specific stimuli)
the membrane, and
membrane trafficking - membrane movement and recycling
-- pumps for active membrane transport.
*All types of cell signaling use membrane receptor proteins that
Diffusion - transports small, nonpolar molecules directly
are often linked to enzymes such as kinases or adenylyl cyclase
through the bilayer
whose activities initiate intracellular signaling pathways.
Channels - multipass proteins forming transmembrane pores
where ions or small molecules pass selectively. Water

AMUAN, BERJAMIN, ERMITANIO, LARIOSA, RESURRECCION – BSMLS2B 7


histology
LECTURE / INSTRUCTOR: JURA SORNILLO JUNQUEIRA’S BASIC HISTOLOGY TEXT AND ATLAS

o In endocrine signaling, the signal molecules (here called Endoplasmic Reticulum


hormones) are carried in the blood from their sources to - The ER is a convoluted network of membrane enclosing
target cells throughout the body. continuous spaces called cisternae and extending from the
o In paracrine signaling, the chemical ligand diffuses in nucleus to the plasma membrane.
extracellular fluid but is rapidly metabolized so that its effect - Rough ER has a granular, basophilic cytoplasmic surface
is only local on target cells near its source. due to the presence of polysomes making most membrane
o In synaptic signaling, a special kind of paracrine interaction, proteins, proteins in certain other organelles, or for
neurotransmitters act on adjacent cells through special exocytosis; RER is always well developed in cells actively
contact areas called synapses. secreting proteins.
o In autocrine signaling, signals bind receptors on the same *Proteins to be processed through the RER contain initial signal
cells that produced the messenger molecule. peptides which bind receptors in the ER membrane, localizing
o In juxtacrine signaling, important in early embryonic tissue them to that organelle.
interactions, the signaling molecules are cell membrane– *After translocation across the membrane into the cisterna, the
bound proteins which bind surface receptors of the target proteins undergo posttranslational modification and folding in
cell when the two cells make direct physical contact. a process monitored by RER molecular chaperones and
Signal Transduction - activating a series of intermediary enzymes.
enzymes downstream to produce changes in the cytoplasm, the - Smooth ER (SER) lacks ribosomes, but includes enzymes
nucleus or both. for lipid and glycogen metabolism, for detoxification
CYTOPLASMIC ORGANELLES reactions, and for temporary Ca2 + sequestration.
Ribosomes
- The two ribosomal subunits, each a complex of rRNA and
many proteins, attach to mRNA and translate that message
into protein.
- Multiple ribosomes on the same mRNA make up a
polyribosome (polysome), and an abundance of these
produces basophilic cytoplasm after H&E staining.

Golgi Apparatus
- The Golgi apparatus is a dynamic organelle consisting of
stacked membranous cisternae in which proteins made in
RER are processed further and packaged for secretion or
other roles.
*Proteins in transport vesicles enter the cis or receiving face of
the Golgi, move through medical cisternae of the Golgi network
for enzymatic modifications, and are released in other vesicles
at the trans face.
*Vesicle movement through the Golgi apparatus is guided by
specific coat proteins such as COPII and COPI.
*Important protein modifications in the Golgi apparatus include
sulfation and many glycosylation reactions.

AMUAN, BERJAMIN, ERMITANIO, LARIOSA, RESURRECCION – BSMLS2B 8


histology
LECTURE / INSTRUCTOR: JURA SORNILLO JUNQUEIRA’S BASIC HISTOLOGY TEXT AND ATLAS

*Modified proteins leave the Golgi apparatus after packaging in *The mitochondrial matrix contains enzymes for β-oxidation of
vesicles with coat proteins that direct movement to lysosomes, fatty acids and the citric acid (Krebs) cycle.
the plasma membrane, or secretion by exocytosis. *The inner membrane includes enzyme assemblies of the
Lysosomes electron-transport system and ATP synthase.
- Primary lysosomes emerge from the Golgi apparatus *Mitochondria of stressed cells may release cytochrome c from
containing inactive acid hydrolases specific for degrading a the inner membrane, triggering a regulated series of events
wide variety of cellular macromolecules. culminating in cell death (apoptosis).
- Secondary lysosomes are more heterogeneous, having
fused with vesicles produced by endocytosis that contain Peroxisomes
material to be digested by the hydrolytic enzymes. - Peroxisomes are small spherical organelles containing
*During autophagy, lysosomes digest unneeded or enzymes for various metabolic reactions, notably for
nonfunctional organelles after these are surrounded by oxidation and detoxification, and catalase that breaks
membrane that then fuses with a lysosome. down the H2 O2 resulting from those reactions.
* Residual Bodies - products of digestion in secondary
lysosomes are released to the cytoplasm for reuse; final Cytoskeleton
condensed vesicles containing any indigestible molecules. *The cytoskeleton contains three types of polymers:
(1) microtubules 25 nm in diameter;
(2) actin filaments or microfilaments (5-7 nm); and
(3) intermediate filaments (8-10 nm).
*Microtubules are semirigid tubular structures with walls
composed of polymerized tubulin heterodimers; their structure
is often very dynamic, with steady addition and dissociation of
tubulin.
- Microtubules are important in maintaining cell shape and
as tracks for transport of vesicles and organelles by the
motor proteins kinesin and dynein.
*Microfilaments are short, flexible, highly dynamic filaments of
actin subunits, in which changes in length and interactions with
binding proteins regulate cytoplasmic viscosity and movement.
*Myosins are motor proteins that bind and move along actin
filaments, carrying vesicles or producing cytoplasmic
movement.
- Movements of cytoplasm produced by actin filaments and
myosins are important for endocytosis, cell cleavage after
Proteasomes mitosis, and cell locomotion on substrates.
- Proteasomes are small cytoplasmic protein complexes *Intermediate filaments are the most stable cytoskeletal
which degrade improperly folded proteins after they are component, conferring strong mechanical stability to cells.
tagged with the polypeptide ubiquitin. - Intermediate filaments are composed of various protein
subunits in different cells; they include vimentin; nuclear
Mitochondria lamins; neurofilament proteins; and keratins, which are
- Mitochondria are the major sites of ATP synthesis and are especially important in epithelial cells.
abundant in cells or cytoplasmic regions where large
amounts of energy are expended. Inclusions
*Mitochondria are usually elongated organelles and form by - Unlike organelles, inclusions are not metabolically active
fission of preexisting mitochondria. and are primarily storage sites, such as lipid droplets,
*Mitochondria have two membranes: a porous outer membrane glycogen granules, pigment granules, or residual bodies
encloses the intermembrane space and an inner membrane (also called lipofuscin).
with many folds (cristae) enclosing a gel-like matrix.

AMUAN, BERJAMIN, ERMITANIO, LARIOSA, RESURRECCION – BSMLS2B 9


histology
LECTURE / INSTRUCTOR: JURA SORNILLO JUNQUEIRA’S BASIC HISTOLOGY TEXT AND ATLAS

CHAPTER 3: THE NUCLEUS Nucleolus - very basophilic or electron-dense area of chromatin


Nucleus is the command center of the cell and contains the localized where rRNA transcription and ribosomal subunits
molecular machinery to replicate the DNA and to synthesize and assembly occur.
process all types of RNA. *By TEM, an active nucleolus is seen to have fibrous and
granular parts where rRNA forms and ribosomal subunits are
assembled, respectively.
THE CELL CYCLE
Cell cycle - the sequence of events that controls cell growth and
division.
-- G1 phase - the longest part of the cycle, begins immediately
after mitosis and includes all preparations for DNA replication.
-- S Phase - the period of DNA and histone synthesis.
-- G2 phase - the cell prepares for division during mitosis.
COMPONENTS OF THE NUCLEUS
*Cell cycling is controlled by the sequential appearance of key
Nuclear Envelope - forms a selectively permeable barrier
cytoplasmic proteins, the cyclins, which bind cyclin-dependent
between the nuclear and cytoplasmic compartments.
kinases (CDKs).
-- penetrated by nuclear pore complexes, large assemblies of
*CDKs phosphorylate and activate the enzymes and
nucleoporins with 8-fold symmetry through which proteins and
transcription factors whose functions characterize each phase
protein-RNA complexes move in both directions.
of the cell cycle.
-- supported internally by a meshwork called nuclear lamina,
*Progress through the cell cycle stages is monitored at
and composed of intermediate filament subunits called lamins.
checkpoints, including the G1 restriction point; only when each
*Cytoplasm is separated from nucleoplasm by the nuclear
phase’s activities are completed are the cyclins changed to
envelope, a double set of membranes with a narrow perinuclear
trigger those of the next phase.
space; the outer membrane binds ribosomes and is continuous
Mitosis
with the RER.
*Stages of mitotic cell divisions include:
Chromatin - the combination of DNA and its associated proteins.
-- Prophase - when chromosomes condense, the nuclear
-- euchromatin - active in transcription stains lightly
envelope disassembles, and the microtubular spindle forms.
-- heterochromatin - inactive chromatin stains more darkly
-- Metaphase - when chromosomes are aligned
-- Constitutive heterochromatin is generally similar in all cell
-- Anaphase - when they begin to separate toward the two
types and contains mainly repetitive, gene-poor DNA
centrosomes
sequences, including the large chromosomal regions called
-- Telophase - when nuclear envelope re-forms around the
centromeres and telomeres, which are located near the middle
separated chromosomes; Telophase ends with cytokinesis or
(most often) and at the ends of chromosomes respectively.
cell cleavage into two daughter cells by a contractile ring of actin
-- Facultative heterochromatin contains other regions of
filaments and myosin.
DNA with genes where transcription is variably inactivated in
*Stem cells occur in all tissues with rapid cell turnover; they
different cells by epigenetic mechanisms and can undergo
divide slowly in an asymmetric manner, with one daughter cell
reversible transitions from compact, transcriptionally silent
remaining a stem cell and one becoming committed toward
states to more open, transcriptionally active conformations
differentiation.
*The DNA molecule initially wraps around complexes of basic
*Cells committed to differentiate (transit amplifying or
proteins called histones to form nucleosomes, producing a
progenitor cells) typically divide more rapidly than stem cells
structure resembling beads on a string.
before slowing or stopping division to differentiate.
*Additional levels of chromatin fiber condensation are less well
understood and involve nonhistone proteins, including
complexes of condensins.
*The extra X chromosome in cells of female mammals forms
facultative heterochromatin and can be seen as the Barr body.
*Karyotyping – process of analyzing chromosomes.

AMUAN, BERJAMIN, ERMITANIO, LARIOSA, RESURRECCION – BSMLS2B 10


histology
LECTURE / INSTRUCTOR: JURA SORNILLO JUNQUEIRA’S BASIC HISTOLOGY TEXT AND ATLAS

Meiosis - the process by which two successive cell divisions CHAPTER 4: EPITHELIAL TISSUE
produce cells called gametes containing half the number of Epithelium - a tissue in which cells are bound tightly together
chromosomes found in somatic cells. structurally and functionally to form a sheet-like or tubular
*Prophase of the first meiotic division is a unique, extended structure with little extracellular material between the cells.
period in which homologous chromosomes pair and undergo *Cells in epithelia each have an:
genetic recombination during the process called synapsis. -- apical side facing the sheet’s free surface and
*Synaptic pairs separate toward two daughter cells at the first -- basal side facing a basement membrane and underlying
meiotic division. connective tissue.
*The second meiotic division occurs with no intervening S phase *Epithelia are often specialized for absorption or transcytosis,
and separates the sister chromatids into two final cells that are pinocytosis of material at the apical side and exocytosis at the
haploid. basolateral side (or vice versa).
*Cells of most epithelia exhibit continuous renewal, with the
locations of stem cells and rates of cell turnover variable in
various specialized epithelia.
Functions of Epithelial Tissues:
o Covering, lining, and protecting surfaces
o Absorption
o Secretion
CHARACTERISTIC FEATURES
*Epithelial cell nuclei vary in shape and may be elliptic (oval),
spherical, or flattened, with nuclear shape corresponding
roughly to cell shape.
Apoptosis - the process by which redundant or defective cells
*Epithelial cells are frequently indistinguishable by light
are rapidly eliminated in a manner that does not provoke a local
microscopy because of its lipid-rich membranes.
inflammatory reaction in the tissue.
Lamina Propria - connective tissue that underlies the epithelia
*Apoptosis involves a cascade of events controlled by the Bcl-2
lining the organs of the digestive, respiratory, and urinary
family of proteins regulating the release of death-promoting
systems.
factors from mitochondria.
Papillae - area of contact between the two tissues may be
*Cytochrome C from mitochondria activates cytoplasmic
increased by small evaginations.
proteases called caspases, which degrade proteins of the
- projects from the connective tissue into the epithelium.
cytosol, cytoskeleton, and cell membrane.
- occur most frequently in epithelial tissues subject to
*Endonucleases are activated, which degrade all nuclear DNA.
friction, such as the covering of the skin or tongue.
*Cell and nuclear volumes shrink rapidly, and the cell membrane
*Epithelial cells generally show polarity, with organelles and
changes produce extensive blebbing of the cell surface.
membrane proteins distributed unevenly within the cell.
*Late in apoptosis, the cell breaks into many small apoptotic
Basal Pole - region of the cell contacting the ECM and
bodies that undergo phagocytosis by neighboring cells.
connective tissue.
*Apoptosis occurs rapidly, with little or no release of proteins
Apical Pole – the opposite end which usually facing a space.
that would trigger inflammation, unlike the death of injured cells
*Regions of cuboidal or columnar cells that adjoin neighboring
by necrosis that typically induces local inflammation.
cells comprise the cells’ lateral surfaces; cell membranes here
often have numerous folds which increase the area and
functional capacity of that surface.
Basement Membrane
– a semipermeable filter
for substances reaching
epithelial cells from
below.

AMUAN, BERJAMIN, ERMITANIO, LARIOSA, RESURRECCION – BSMLS2B 11


histology
LECTURE / INSTRUCTOR: JURA SORNILLO JUNQUEIRA’S BASIC HISTOLOGY TEXT AND ATLAS

*Functions of Basal Membrane: Hemidesmosomes - anchoring junctions composed of


– helping to provide structural support for epithelial cells and transmembrane integrins attach cells to proteins of the basal
attach epithelia to underlying connective tissue. lamina.
– its components help organize integrins and other proteins in Focal adhesion or focal contact - basal anchoring junction
the plasma membrane of epithelial cells, maintaining cell polarity found in cells that are moving during epithelial repair or
and helping to localize endocytosis, signal transduction. reorganization.
Focal adhesion kinase - a signaling protein which upon integrin
– mediate many cell-to-cell interactions involving epithelia and
binding to laminin or other specific ECM proteins initiates a
mark routes for certain cell migrations along epithelia.
cascade of intracellular protein phosphorylation affecting cell
– serves as a scaffold that allows rapid epithelial repair and adhesion, mobility, and gene expression
regeneration. Gap or communicating junctions - are points of cell contact
*Two Parts of Basal Membrane: where both plasma membranes have numerous hexameric
o basal lamina - a thin, electron-dense, sheet-like layer of complexes of transmembrane connexons, each forming a
fine fibrils; meshwork of IV collagen & laminin. channel allowing passage of small molecules from one cell to
Contains: the other.
– Type IV Collagen – monomers of type IV collagen *Connexin – gap junction proteins which produces connexons
– Laminin - large glycoproteins that attach to SPECIALIZATIONS OF THE APICAL CELL SURFACE
transmembrane proteins called integrins at the cells’ basal Microvilli - small membrane
surface and project through the network of type IV collagen. projections with cores of actin
filaments that generally function to
– Nidogen and Perlecan - a short, rod-like protein and a
increase epithelial cells’ apical
proteoglycan, both of
surface area for absorption.
these cross-link laminin
*The average microvillus is about
to the collagen network
1-μm long and 0.1-μm wide.
and help determine the
*The total surface area can be
porosity of the basal
increased by 20- or 30-fold.
lamina and the size of
Stereocilia - long microvilli with
molecules able to filter
specialized mechanosensory
through it.
function in cells of the inner ear
o reticular lamina -
and for absorption in tissues of
contains type III collagen
the male reproductive tract.
and anchoring fibrils of VII collagen, all secreted by cells of
the immediately adjacent connective tissue.

Intercellular Junctions
Cilia - long, highly motile
- well developed in epithelia and consist of three major types,
apical structures, larger
with different functions:
than microvilli, containing
o Tight or occluding junctions (zonula occludens)
internal arrays of
– form a seal between adjacent cells
microtubules
– formed by interacting transmembrane proteins such as
- larger projecting
claudin and occludin
structures with a well-
*linear arrangements of these linked proteins surround the
organized core of
apical ends of the cells and prevent paracellular passage of
microtubules (in a 9 + 2
substances between the cells
arrangement called the
o Adherent or anchoring junctions (zonula adherens)
axoneme) in which
– sites of strong cell adhesion
restricted, dynein-based
– formed by interacting proteins of the cadherin family
sliding of microtubules
*Adherent junctions - encircle epithelial cells just below their
causes ciliary movement
tight junctions or scattered, spot-like attachment sites called
that propel material along
desmosomes or maculae adherens, both of which are attached
an epithelial surface.
to cytoplasmic keratins.
*Cilia exhibit rapid beating patterns that move a current of fluid
and suspended matter in one direction along the epithelium.

AMUAN, BERJAMIN, ERMITANIO, LARIOSA, RESURRECCION – BSMLS2B 12


histology
LECTURE / INSTRUCTOR: JURA SORNILLO JUNQUEIRA’S BASIC HISTOLOGY TEXT AND ATLAS

*Ciliary motion occurs through successive changes in the -- Pseudostratified epithelia


conformation of the axoneme, in which various accessory are thick and appear to have
proteins make each cilium relatively stiff, but elastic. several cell layers; all cells
MORPHOLOGICAL TYPES OF EPITHELIA attach to the basal lamina but
Epithelia’s two main groups: not all extend to the free
o Covering Epithelia - lining epithelial surface.
o Secretory Epithelia - secretory
Simple Epithelia – an epithelium in which the basement Epithelial Secretion/Glands
membrane has one cell layer. *The major function in many epithelial cells is synthesis and
*Based on cell shape, it is classified as: secretion of specialized products
-- squamous (thin cells), Goblet cell – similar individual cells abundant in the lining of the
small intestine and respiratory tract, which secretes lubricating
mucus that aids the function of these organs.
Glands - specialized organs composed primarily of epithelia.
-- develop from covering epithelia in the fetus by cell
-- cuboidal (cell width and proliferation and growth into the underlying connective tissue,
thickness roughly similar), followed by further differentiation.
o Exocrine glands have epithelial ducts carrying secretions to
specific sites; the ducts of simple glands are unbranched
and those of compound glands are branched.
-- columnar (cells are taller than -- the secretory portions of exocrine glands may form
they are wide) round, saclike acini (also called alveoli) or elongated
tubules; both types of secretory units may themselves
branch.
o Endocrine glands lack ducts; secreted substances are
Stratified Epithelia – contain two or hormones carried throughout the body by the interstitial
more epithelial layers fluid and blood, with specificity produced by the hormone
*Almost all such epithelia are receptors of target cells.
stratified squamous, in which the -- thin-walled blood vessels (capillaries) adjacent to
outer cell layers are thin and endocrine cells absorb their secreted hormone products for
flattened. transport in blood to target cells throughout the body.
*Classified according to the cell shape of the superficial outer -- Endocrine glands lack myoepithelial cells and are
layer(s): squamous, cuboidal, or columnar. specialized for either protein or steroid hormone synthesis,
-- Stratified squamous keratinized epithelium such as the with cytoplasmic staining characteristic of RER or SER.
epidermis cover the body surface, protecting underlying tissues *Glands can be:
from excess water loss (dehydration) and microbial invasion. -- Simple (ducts not branched)
-- Stratified squamous nonkeratinized epithelium lines moist -- Compound (ducts with two or more branches)
internal cavities (eg, mouth, esophagus, and vagina) where *Secretory portions can be:
water loss is not a problem. Here the flattened cells of the -- Tubular (either short or long and coiled)
surface layer retain their nuclei and most metabolic functions. -- Acinar (rounded and saclike)
-- Stratified cuboidal epithelium occurs in the excretory ducts *Glands have three basic secretory mechanisms:
of salivary and sweat glands. o Merocrine - most common method of protein or
-- Stratified columnar epithelium seen in the conjunctiva lining glycoprotein secretion and involves typical exocytosis from
the eyelids, where it is both protective and mucus secreting. membrane-bound vesicles or secretory granules.
-- Transitional epithelium or o Holocrine - cells accumulate product continuously as they
urothelium lines only on the urinary enlarge and undergo terminal differentiation, culminating in
tract, extending from the kidneys to complete cell disruption which releases the product and cell
the proximal part of the urethra, debris into the gland’s lumen; terminally differentiated cells
and is characterized by a superficial filled with lipid product are released.
layer of large, dome-like cells sometimes called umbrella cells. o Apocrine - product accumulates at the cells’ apical ends,
portions of which are then extruded to release the product
together with small amounts of cytoplasm & cell membrane

AMUAN, BERJAMIN, ERMITANIO, LARIOSA, RESURRECCION – BSMLS2B 13


histology
LECTURE / INSTRUCTOR: JURA SORNILLO JUNQUEIRA’S BASIC HISTOLOGY TEXT AND ATLAS

Mucous Glands - exocrine glands producing mucus CHAPTER 5: CONNECTIVE TISSUE


*Oligosaccharide components of mucus stain poorly with Connective tissue provides a matrix that supports and
routine dyes but stain well with PAS stain. physically connects other tissues and cells together to form the
Serous Glands - exocrine glands producing largely enzymes organs of the body.
(proteins) and stain darkly with H&E due to the cells’ content of Interstitial fluid – gives metabolic support to cells as the
RER and secretory granules. medium for diffusion of nutrients and waste products.
Seromucous Glands - having both serous acini and mucous Extracellular Matrix (ECM) – major constituent of connective
tubules with clustered serous cells. The product of these glands tissue consists of protein fibers (collagen and elastic) and
is a mixture of digestive enzymes and watery mucus. ground substance, which is a complex of anionic, hydrophilic
TRANSPORT ACROOSE EPITHELIA proteoglycans, glycosaminoglycans (GAGs), and multiadhesive
Transcellular Transport – a process when some epithelial cells glycoproteins (laminin, fibronectin, and others).
specialize in the transfer of ions (by ion pumps) and water (via *Glycoproteins help stabilize the ECM by binding to other matrix
the membrane channels called aquaporins) in either direction components and to integrins in cell membranes.
across the epithelium. Mesenchymal cells - undifferentiated and have large nuclei,
* Apical tight junctions prevent paracellular diffusion or backflow with prominent nucleoli and fine chromatin. They are often said
between the cells. to be “spindle-shaped,” with their scant cytoplasm extended as
*Epithelia of kidney tubules are key sites for ion and water two or more thin cytoplasmic processes.
transport, maintaining the body’s overall balance of salt and *All adult connective tissues are derived from an embryonic
water. form of connective tissue called mesenchyme, which contains
*Cells of the proximal renal tubules are specialized structurally uniformly undifferentiated cells scattered in a gel-like matrix.
for transcellular transport. CELLS OF CONNECTIVE TISSUE
*Osmotic and electrical balance is maintained by the passive Fibroblasts (fibrocytes) - the
transfer of chloride ions (Cl–) and water into the cell. major cells of connective tissue
Transcytosis – when cells cross the thin cells in both directions proper, are elongated, irregularly
and release their contents on the opposite side by exocytosis. shaped cells with oval nuclei that
-- occurs between the apical and basolateral membranes synthesize and secrete most
domains in cells of simple cuboidal and columnar epithelia and components of the ECM.
is important in many physiologic processes. *Active fibroblast has more
RENEWAL OF EPITHELIAL CELLS abundant and irregularly
Epithelia are normally capable of rapid repair and replacement branched cytoplasm, containing
of apoptotic or damaged cells. much rough endoplasmic
*Cells of undamaged regions quickly begin active proliferation reticulum (RER) and a well-developed Golgi apparatus, with a
and the organ or tissue affected is regenerated. large, ovoid, euchromatic nucleus and a prominent nucleolus.
Adipocytes (fat cells) - are very large cells specialized for
storage of triglycerides; they predominate in a specialized form
of connective tissue called adipose tissue.
Macrophages - are short-lived cells that differentiate in
connective tissue from precursor cells called monocytes
circulating in the blood; they function in ECM turnover,
phagocytosis of dead cells and debris, and antigen presentation
to lymphocytes.
Mast cells - are oval or irregularly shaped cells of connective
tissue, between 7 and 20 μm in
diameter, filled with basophilic
secretory granules which often
obscure the central nucleus.
-- originate from blood cell
precursors and are filled with
granules for the release of various
vasoactive agents and other
substances during inflammatory
and allergic reactions.

AMUAN, BERJAMIN, ERMITANIO, LARIOSA, RESURRECCION – BSMLS2B 14


histology
LECTURE / INSTRUCTOR: JURA SORNILLO JUNQUEIRA’S BASIC HISTOLOGY TEXT AND ATLAS

-- metachromasia, which means that they can change the color


of some basic dyes (eg, toluidine blue) from blue to purple or
red. -- immediate hypersensitivity reactions – allergic reactions
due to the release of certain chemical mediators stored in mast
cells (e.g. anaphylactic shock - causes antibody-producing cells
to produce an immunoglobulin of the IgE class which binds
avidly to receptors on the surface of mast cells and triggers the
rapid release of histamine, leukotrienes, chemokines, and
heparin from the mast cell granules which can produce the
sudden onset of the allergic reaction).
*Molecules released from these cells’ secretory granules
includes:
o Heparin - a sulfated GAG that acts locally as an
anticoagulant
o Histamine - promotes increased vascular permeability and
smooth muscle contraction
o Serine proteases - activate various mediators of
inflammation
o Eosinophil and neutrophil chemotactic factors - attract
those leukocytes
o Cytokines - polypeptides directing activities of leukocytes FIBERS OF CONNECTIVE TISSUE
and other cells of the immune system The most important and abundant fibers of connective tissue
o Phospholipid precursors - are converted to prostaglandins, are composed of the protein collagen, of which there are some
leukotrienes, and other important lipid mediators of the 20 related types.
inflammatory response *Synthesis of collagen by fibroblasts and certain other cells
involves posttranslational modifications in the RER, notably
hydroxylation of the numerous prolines and lysines, and
formation of helical trimeric subunits of procollagen.
*Upon exocytosis, the nonhelical ends of the procollagen
subunits are removed, forming trimeric collagen molecules
that aggregate and become covalently bound together in large
collagen fibrils.
Collagenases – specific enzyme that initiates degredation.
Metalloproteinases (MMPs), produced primarily by
macrophages, and clip collagen fibrils or sheets in such a way
that they are then susceptible to further degradation by non-
specific proteases.
Crossbanding - a characteristic pattern of highly regular
assembly of collagens in the fibrils visible ultrastructurally along
Plasma cells – are short-lived cells the fibrils of some collagen types.
that differentiate from B *Fibrils of type I collagen are bundled together by other forms
lymphocytes and are specialized for of nonfibrillar, linking collagens to produce large collagen
the abundant secretion of specific bundles.
antibodies (immunoglobulins). Reticular fibers – cinsist mainly of
Their average lifespan is only 10-20 collagen type III which stain very
days. dark with silver stains
Leukocytes normally wander through all types of connective (argyrophilic) and are abundant in
tissue proper, providing surveillance against bacterial invaders immune and lymphoid tissues.
and stimulating tissue repair. *Reticular fibers produced by
*Inflammation, a vascular and cellular defensive response to fibroblasts occur in the reticular
injury or foreign substances, including pathogenic bacteria or lamina of basement membranes
irritating chemical substances. and typically also surround adipocytes, smooth muscle and
nerve fibers, and small blood vessels.
AMUAN, BERJAMIN, ERMITANIO, LARIOSA, RESURRECCION – BSMLS2B 15
histology
LECTURE / INSTRUCTOR: JURA SORNILLO JUNQUEIRA’S BASIC HISTOLOGY TEXT AND ATLAS

Elastic fibers, or sheets called TYPES OF CONNECTIVE TISSUE


elastic lamellae, are composed Connective tissue - usually classified as loose or dense
of the proteins elastin and according to the amount of collagen and ground substance
fibrillin, which exist in a present.
stretchable conformation that o Loose connective tissue (or
provides elastic properties to areolar tissue) has relatively
connective tissues rich in this more ground substance than
material. collagen, and it typically
*Elastic fibers are stained surrounds small blood
more darkly than collagen with vessels and occupies areas
other stains such as orcein and adjacent to other types of
aldehyde fuchsin. epithelia.
*Elastic fibers (and lamellae) are a composite of fibrillin (350 o Dense irregular connective
kDa), which forms a network of microfibrils, embedded in a tissue is filled primarily with
larger mass of cross-linked elastin (60 kDa), randomly distributed bundles
*Elastin accumulates around the microfibrils, eventually making of type I collagen, with some
up most of the elastic fiber, and is responsible for the rubberlike elastic fibers, providing
property. resistance to tearing from all
GROUND SUBSTANCE directions as well as some
Ground substance - the watery, largely unstained extracellular elasticity.
material that is more abundant than fibers in some types of o Dense regular connective
connective tissue proper. tissue, prominent in tendons
-- mixture of three major kinds of macromolecules: and ligaments, features
glycosaminoglycans (GAGs), proteoglycans, and bundles of essentially parallel
multiadhesive glycoproteins. type I collagen, providing
-- allows diffusion of small molecules and acts as both a great strength (but little
lubricant and a barrier to the penetration of invaders. stretch) in binding together
o GAGs - mucolpolysaccharides components of the
*The major types of GAGs are: musculoskeletal system.
-- hyaluronan (hyaluronic acid), which is a very long polymer of o Reticular tissue consists of
the disaccharide glucosamine-glucuronate; forms a viscous, delicate networks of type III
pericellular network that binds a considerable amount of water collagen and is most
which allows molecular diffusion through connective tissue and abundant in certain lymphoid
in lubricating various organs and joints. organs where the fibers form
-- various shorter chains of sulfated GAGs composed of other attachment sites for
disaccharide polymers. lymphocytes and other
*Sulfated GAGs such as chondroitin sulfate and keratan sulfate immune cells.
have various sizes and compositions, but they are all bound to o Mucoid tissue is a gel-like
the core proteins of proteoglycans and are produced in the Golgi connective tissue with few
apparatus before secretion. cells found most abundantly
o Proteoglycans – synthesized on RER, mature in the golgi around blood vessels in the
complex, and secreted from cells by exocytosis. umbilical cord.
-- attach to polymers of HA via linker proteins to form huge
complexes in ground substance that bind water and other
substances, including certain polypeptide growth factors that
help regulate fibroblast proliferation.
o Multiadhesive glycoproteins such as fibronectin and
laminin have binding sites for collagens and for integrin
proteins in cell membranes, thus allowing temporary
attachments between cells and the ECM required for cell
migration and positioning.

AMUAN, BERJAMIN, ERMITANIO, LARIOSA, RESURRECCION – BSMLS2B 16


histology
LECTURE / INSTRUCTOR: JURA SORNILLO JUNQUEIRA’S BASIC HISTOLOGY TEXT AND ATLAS

CHAPTER 6: ADIPOSE TISSUE Storage and Mobilization of Lipids:


Adipose Tissue - a connective tissue which stores fat cells. Chylomicrons – formed from ingested lipids or dietary fats that
-- derived from mesenchyme and specialized for energy storage are brought to the cells thru circulation
in lipid droplet with triglycerides. VLDLs (very low-density llipoproteins) – transports
-- has two types: white and brown fat tissues chylomicrons; synthesized from lipids in liver cells.
*Adipocytes – stores lipids from: Lipoprotein lipase – enzyme synthesized by the adipocytes and
-- dietary fats packaged as chylomicrons in the intestines
transferred to the capillary cell membrane.
-- triglycerides produced in the liver and circulating as VLDLs
*Insulin stimulates glucose uptake by adipocytes w/c
-- fatty acids synthesized locally
*Lipids are mobilized from adipocytes by hormone-sensitive accelerates its conversion into triglycerides, and the production
lipase activated by norepinephrine released from the adrenal of lipoprotein lipase.
gland and various peptide hormones. *Hormonal activity of adipocytes produces leptin which helps
*Cells of adipose tissue are supported by reticular fibers, with regulate the appetite under normal conditions and participates
connective tissue septa dividing the tissue into lobules of in regulating the formation of new adipose tissue.
various sizes.
*Function:
-- energy storage
-- key regulator of overall body metabolism
-- thermal insulator
-- responds to both nervous and hormonal stimuli
WHITE ADIPOSE TISSUE

Also called as “unilocular” becausethese are often empty in


standard light microscopy.
-- has a flattened nucleus, and a signet-ring appearance.
-- found in many organs throughout the body, typically forming
about 20% of the body weight in adults.
*Adipocytes of white fat are typically very large cells, ranging in
diameter from 50 to 150 μm.
*These cells each contain primarily one large lipid droplet (they
are unilocular), causing the nucleus and remaining cytoplasm to
be pushed against the plasmalemma.
*Fatty acids are released from white adipocytes by lipase activity
when nutrients are needed and carried throughout the body on
plasma proteins such as albumin.
Leptin is a polypeptide hormone with target cells in the
hypothalamus that is released from white adipocytes and helps
regulate eating behavior.

AMUAN, BERJAMIN, ERMITANIO, LARIOSA, RESURRECCION – BSMLS2B 17


histology
LECTURE / INSTRUCTOR: JURA SORNILLO JUNQUEIRA’S BASIC HISTOLOGY TEXT AND ATLAS

BROWN ADIPOSE TISSUE CHAPTER 7: CARTILAGE


Brown fat comprises up to 5% of the newborn body weight but Cartilage is a tough, durable form of supporting connective
smaller amounts in adults. tissue, characterized by an extracellular matrix (ECM)
- quickly metabolized *Cartilage ECM has a firm consistency that allows the tissue to
*Adipocytes of this tissue are typically smaller than those of bear mechanical stresses without permanent distortion.
white fat and contain primarily many small lipid droplets (they *Cartilage provides cushioning and sliding regions within
are multilocular) in cytoplasm containing many mitochondria skeletal joints and facilitates bone movements.
Chondrocytes synthesize and maintain all ECM components and
and a central nucleus.
are located in matrix cavities called lacunae.
*Function:
-- exhibit low metabolic activity.
-- fatty acids released in adipocytes of brown fat are metabolized
*The physical properties of cartilage depend on electrostatic
in mitochondria of these cells for thermogenesis rather than
bonds between type II collagen fibrils, hyaluronan, and the
ATP synthesis, using uncoupling protein-1.
sulfated glycosaminoglycans on densely packed
-- stores lipid as multiple droplets
proteoglycans.
-- best developed in hibernating animals for heat generation
*The high content of bound water allows cartilage to serve as a
*The number of brown adipocytes increases during cold
shock absorber, an important functional role.
adaptation, usually appearing as clusters of multilocular cells in
*All types of cartilage lack vascular supplies and chondrocytes
white adipose tissue.
receive nutrients by diffusion from capillaries in surrounding
*Autonomic nerves also promote brown adipocyte
connective tissue (the perichondrium).
differentiation and prevent apoptosis in mature brown fat cells.
Perichondrium is a sheath of dense connective tissue that
surrounds cartilage.
Articular cartilage – covers the ends of bones in movable joints
and erodes in the course in the course of arthritic degeneration,
lacks perichondrium and is sustained by the diffusion of oxygen
and nutrients from synovial fluid.
HYALINE CARTILAGE
Hyaline cartilage - homogeneous
and semitransparent in the fresh
state.
-- located where it makes possible
for bone growth.

AMUAN, BERJAMIN, ERMITANIO, LARIOSA, RESURRECCION – BSMLS2B 18


histology
LECTURE / INSTRUCTOR: JURA SORNILLO JUNQUEIRA’S BASIC HISTOLOGY TEXT AND ATLAS

*A schematic representation of the most abundant molecules in *More flexible than hyaline cartilage, elastic cartilage is found in
cartilage matrix shows the interaction between type II collagen the auricle of the ear, the walls of the external auditory canals,
fibrils and proteoglycans linked to hyaluronan. the auditory (Eustachian) tubes, the epiglottis, and the upper
respiratory tract.

FIBROCARTILAGE
Matrix is basophilic due to proteoglycans. Most of the collagen Fibrocartilage - a mingling of hyaline cartilage and dense
in hyaline cartilage is type II. connective tissue.
Aggrecan - most abundant proteoglycan of hyaline cartilage. -- found in intervertebral discs, and all places where it serves as
Chondronectin - a structural multiadhesive glycoprotein and very tough, yet cushioning support tissue for bone.
another important component of cartilage matrix. *Chondrocytes of fibrocartilage occur singly and often in aligned
Territorial Matrix - surrounds the lacunae. isogenous aggregates, producing type II collagen and other ECM
Chondrocytes - cells responsible for cartilage formation, and components.
they are crucial for the process of endochondral ossification, *The relative scarcity of proteoglycans overall makes
which is useful for bone development. fibrocartilage matrix more acidophilic than that of hyaline or
*In living tissue chondrocytes fill their lacunae completely. elastic cartilage.
*Because cartilage matrix is avascular, chondrocytes respire *There is no distinct surrounding perichondrium in
under low-oxygen tension. Hyaline cartilage cells metabolize fibrocartilage.
glucose mainly by anaerobic glycolysis. Nutrients from the blood *Fibrocartilage provides very tough, strong support at tendon
diffuse to all the chondrocytes. insertions and in intervertebral discs and certain other joints.
Growth hormone or somatotropin - the pituitary-derived protein
which is the major regulator of hyaline cartilage growth.
Perichondrium - a dense connective tissue, essential for the
growth and maintenance of cartilage.
*The outer region of the perichondrium consists largely of
collagen type I fibers and fibroblasts, but an inner layer
adjoining the cartilage matrix also contains mesenchymal stem
cells which provide a source for new chondroblasts that divide
and differentiate into chondrocytes.
ELASTIC CARTILAGE
-- contains an abundant
network of elastic fibers in
addition to a meshwork of
collagen type II fibrils,
which give fresh elastic
cartilage a yellowish color.

AMUAN, BERJAMIN, ERMITANIO, LARIOSA, RESURRECCION – BSMLS2B 19


histology
LECTURE / INSTRUCTOR: JURA SORNILLO JUNQUEIRA’S BASIC HISTOLOGY TEXT AND ATLAS

CARTILAGE FORMATION, GROWTH, AND REPAIR CHAPTER 8: BONE


All cartilage forms from embryonic mesenchyme in the process Bone tissue provides solid support for the body, and serve as
of chondrogenesis. reservoir of calcium, phosphate, and other ions for maintenance
of constant concentrations in body fluids.
*Confers mechanical and metabolic functions to the skeleton.
*Metabolites from blood are unable to diffuse through calcified
bone matrix communication of blood capillaries to the bone
depends on the Canaliculi
*All bones are lined on their internal surfaces with endosteum
and the external surface with periosteum.
BONE CELLS
*Bone is a specialized connective tissue compose of calcified
extracellular material (bone matrix) and 3 major types:
o Osteocytes
(a) Mesenchyme is the precursor for all types of cartilage.
(b) Mitosis and initial cell differentiation produces a tissue with - found in cavities (lacunae)
condensations of rounded cells called chondroblasts. between bone matrix layers
(c) Chondroblasts are then separated from one another again by (lamellae) with cytoplasmic
their production of the various matrix components, which processes called canaliculi
collectively swell with water and form the very extensive ECM. that extends into the matrix.
(d) Multiplication of chondroblasts within the matrix gives rise -- enclosed singly within a
to isogenous cell aggregates surrounded by a condensation of lacuna in the bone matrix.
territorial matrix. -- Maintain the calcified matrix.
*During embryonic development, the cartilage differentiation -- Network of dendritic processes extending from
takes place primarily from the center outward; therefore, the osteocytes has be called a mechanosensory monitoring
more central cells have the characteristics of chondrocytes, areas where loading has increased or decreased and
whereas the peripheral cells are typical chondroblasts.
signaling cells to adjust ions levels and maintain bone
*Once formed, the cartilage tissue enlarges both by interstitial
matrix.
growth, involving mitotic division of preexisting chondrocytes,
and by appositional growth, which involves chondroblast o Osteoblast - growing cells which synthesize and secrete
differentiation from progenitor cells in the perichondrium. the organic components of the matrix.
*Cartilaginous structures grow by mitosis of existing -- Produces bone matrix that includes type 1 collagen
chondroblasts in lacunae (interstitial growth) or formation of fibers, proteoglycans, and glycoproteins such as
new chondroblasts peripherally from progenitor cells in the osteonectin.
perichondrium (appositional growth). -- When their synthetic activity is completed, some
*In articular cartilage, cells and matrix near the articulating osteoblasts differentiate as osteocytes entrapped in matrix
surface are gradually worn away and must be replaced from bound lacunae, some flatten and cover the matrix surface
within, because there is no perichondrium to add cells by as bone lining cells, and the majority undergo apoptosis.
appositional growth. -- Matrix components are secreted at the cell surface that
*Except in young children, damaged cartilage undergoes slow
produces a collagen-rich material called osteoid.
and often incomplete repair, primarily dependent on cells in the
-- Osteocalcin, secreted by osteoblasts, will bind
perichondrium.
Ca2+ions and will allow an increase of local concentration
*In damaged areas the perichondrium produces a scar of dense
connective tissue instead of forming new cartilage. The poor of Ca2+.
capacity of cartilage for repair or regeneration is due in part to -- Osteoblast will then release matrix vesicles that are rich
its avascularity and low metabolic rate. in alkaline phoisphate which increases the local PO43-
ions; Ca2+ and PO43-concentrations will continue to
increase.
-- Matrix vesicles serve as foci for the formation of
hydroxyapatite [Ca10(PO4 )6 (OH)2] crystals, the first
visible step in calcification.

AMUAN, BERJAMIN, ERMITANIO, LARIOSA, RESURRECCION – BSMLS2B 20


histology
LECTURE / INSTRUCTOR: JURA SORNILLO JUNQUEIRA’S BASIC HISTOLOGY TEXT AND ATLAS

o Osteoclasts - giant PERIOSTEUM AND ENDOSTEUM


multi nucleated cells Periosteum - outer fiberous layer of dense CT containing mostly
involved in removing collagen type I, fibroblasts, and blood vessels.
calcifies matrix and -- bundles of peristeal collagen, called perorating or sharpey’s
remodeling bone fibers penetrate the bone matrix and bind periosteum to bone.
tissue. -- Inner layers is more cellular, containing osteoblasts bone
-- are very large, lining cells, and osteoprogenitor cells.
motile cells with Endosteum - the internal covering of small trabeculae of bony
multiple nuclei that are essential for matrix resorption matrix that project into narrow cavity.
during bone growth and remodeling. TYPES OF BONES
-- Osteoclast development requires two polypeptides o Compact bone
produced by osteoblasts: - dense area near the surface
macrophage-colony-stimulating factor, and - represents 80% of total bone
receptor activator of nuclear factor-κB ligand (RANKL). mass
-- In areas of bone undergoing resorption, osteoclasts on o Cancellous (trabecular)
the bone surface lie in depressions or cavities in the matrix bone
known as resorption lacunae (or Howship lacunae). - deeper bone with
-- an active osteoclast, the membrane domain that contacts interconnecting cavities
the bone forms a circular sealing zone that binds the cell - constitutes 20% of total bone
tightly to the bone matrix and surrounds an area with many mass.
surface projections, called the ruffled border. *In long bones, the bulbous areas are called epiphyses and are
-- Into this subcellular pocket the osteoclast pumps protons composed of cancellous bone with a thin covering of compact
to acidify and promote dissolution of the adjacent bone
hydroxyapatite, and releases matrix metalloproteinases and *The cylindrical part the diaphysis is almost totally dense
other hydrolytic enzymes from lysosome-related secretory compact bone, with a thin region of cancellous bone on the inner
vesicles for the localized digestion of matrix proteins. surface around the central marrow cavity.
-- Activity is activated by PTH which targets osteoblasts to *Short bones such as those of the wrist and ankle usually have
secrete RANKL and MCSF to increase osteoclast activity. cores of cancellous bone surrounded completely by compact
bone.
*The flat bones that form the calvaria (skullcap) have two layers
of compact bone called plates, separated by a thicker layer of
cancellous bone called the diploë.
*both compact and cancellous bones typically show two types
of organization: mature lamellar bone, with matrix existing as
discrete sheets, and woven bone, newly formed with randomly
arranged components.
o Lamellar Bone
- most bones are organized as
this in adults
- consists of multiple layers of
calcified, each is 3-7nm thick.
BONE MATRIX - organized as parallel sheets
*Consists of 90% type I collagen, including proteoglycans
or concentrically around a
and multiadhesive glycoproteins such as osteonectin.
central canal.
*Calcium binding proteins, osteocalcin, and the
phosphatases released from cells in matrix vesicles
*Highly organized collagen fibers causes birefringence in
promote calcification of the matrix.
polarizing light microscopy
*The association of minerals with collagen fibers during
calcification provides the hardness and resistance of bones.

AMUAN, BERJAMIN, ERMITANIO, LARIOSA, RESURRECCION – BSMLS2B 21


histology
LECTURE / INSTRUCTOR: JURA SORNILLO JUNQUEIRA’S BASIC HISTOLOGY TEXT AND ATLAS

*An osteon (or Haversian system) refers to the complex of STEPS:


concentric llamelae and are typically 100-250 nm in diameter. 1. A bone cellar develops beneath the perichondrium around
*All cells of osteon receive nutrients from central canal. the middle of the cartilage models.
*Outer boundary of each osteon is called cement line. 2. Bone collar causes chondrocytes hypertrophy in underlying
*Central canals communicate with each other through cartilage.
perforating canals (Volksmann canals). 3. Formation of primary ossification center is formed by
*Scattered among the osteons are irregular shaped groups of invading the osteoprogenitor cells.
parallel lamellae is called the interstitial lamellae (these 4. At birth secondary ossification centers begin to develop in
lamellae are remaining from partially destroyed osteons by the epiphyses and primary and secondary ossification
osteoclasts) centers will be separated by epiphyseal plate (provides for
*Compact bone also includes parallel lamellae organized as continued bone elongation)
multiple external circumferential llamellae immediately 5. Ossification center merge when full structure is achieved
beneath the periosteum.
Bone remodeling - occurs continually throughout life *The epiphyseal plate shows distinct regions of cellular activity:
1. Osteoclasts remove old bone and form small, tunnel-like
cavities.
2. These tunnels are quickly invaded by osteoprogenitor cells
3. Osteoblasts develop and line walls of the tunnels and begins
to secrete osteoid in a cyclic manner, forming a new osteon
with concentric lamallae of bone and trapped osteocytes.
o Woven Bone
- nonlamellar
- random disposition of type 1 collagen fibers
- first bone to appear in embryonic development and in
fracture repair
- has lower mineral content but has a higher osteocytes
proportion
- forms more quickly but less strength than lamellar bone
OSTEOGENESIS
Bone development occurs one of two processes:
1. Intramembranous ossification
- osteoblasts differentiate directly from mesenchyme and begin o Zone of reserve - composed of typical hyaline cartilage
secreting osteoid o Proliferative zone - cartilage cells divide, enlarge and
- most flat bones begins this way secrete type II collagen and become organized into
*Within the condensed mesenchymal cells, bone formation columns.
begins ossification centers, areas in which osteoprogenitor o Zone of hypertrophy - contains swollen, terminally different
cells arise. chondrocytes which compress the matrix into aligned
*Osteoid calcifies, forming areas of woven bone with spicules by secretion of type X collagen.
osteocytes, lacunae, and canaliculi. o Zone of calcified cartilage - chondrocytes is about to
*Anatomical bone forms as woven bone gets replaced by a undergo apoptosis release matrix vesicles and osteocalcin
compact bone. to begin matrix ossification.
*Mesenchymal regions that do not give undergo ossification will o Zone of ossification - bone tissue first appears.
give rise to the periosteum and endosteum of a new bone.
2. Endochondral ossification *Growth in bone circumference occurs through appositional
- pre-existing matrix of hyaline cartilage is eroded and invaded growth, with formation of the bone collar on central marrow
by osteoblasts which then begin osteoid secretion. cavity and the central marrow cavity enlarges via osteoclasts in
*The hyaline is shaped as a smalls version/model of the bone to the endosteum.
be formed.
- This type of ossification forms most bones in our body
AMUAN, BERJAMIN, ERMITANIO, LARIOSA, RESURRECCION – BSMLS2B 22
histology
LECTURE / INSTRUCTOR: JURA SORNILLO JUNQUEIRA’S BASIC HISTOLOGY TEXT AND ATLAS

METABOLIC ROLE OF BONE CHAPTER 9: NERVOUS TISSUE AND NERVOUS SYSTEM


*Skeleton serves as a calcium reservoir that contains 99% of the Nerve Tissue is distributed throughout the body as an integrated
body’s total calcium in hydroxyapalite crystals. communications network.
*Calcium mobilization is regulated by paracrine interaction *The general organization of the nervous system has two major
among bone cells: divisions:
-- Parathyroid hormone (PTH) - increase blood calcium by Central nervous system (CNS) - consisting of the brain and
stimulating osteoclasts and osteocytes resorb bone matrix and spinal cord
release calcium. PTH receptors are on osteoblasts; which Peripheral nervous system (PNS) - composed of the cranial,
respond by secreting RankL to stimulate osteoclast activity. spinal, and peripheral nerves conducting impulses to and from
the CNS (sensory and motor nerves, respectively) and ganglia
-- Calcitonin - produced by thyroid, can reduce elevated blood
that are small aggregates of nerve cells outside the CNS.
by opposing PTH, targets osteoclasts to slow their activity.
Neurons - cells that typically have numerous long processes.
JOINTS Glial cells - have short processes, support and protect neurons,
o Synarthroses - allow very little to no movement and participate in many neural activities, neural nutrition, and
-- Syntoses: bones linked to bone and allow essentially no defense of cells in the CNS.
movement. Neurons respond to environmental changes (stimuli).
- joints held together by sutures (Ex: uniting skull bones) Irritable or Excitable cells - rapidly change their potential in
o Amphiarthroses - allows some movement response to stimuli (eg, neurons, muscle cells, some gland
-- Syndesmoses cells).
- join bone by dense connective tissue only. Membrane depolarization - activates sodium channels.
-- Symphyses Action potential, the depolarization wave, or the nerve impulse
- thick pad of fiber cartilage between thin articular cartilage - capable of traveling long distances along neuronal processes,
covering the ends of the bones. transmitting such signals to other neurons, muscles, and
- occur in the midline of the body glands.
o Diarthroses or synovial joints
- permit free bone movement
- capsule encloses a sealed Joint Cavity
- joint cavity that contains synovial fluid and is lined by a
synovial membrane
- synovial membrane characterized by 2 specialized cells:
-- type A cells (Macrophage-like synovial cells)
- remove wear-and-tear debris from synovial fluid.
- regulate inflammation within diarthrotic joints.
-- type B cells (Fibroblastic synovial cells)
- produce hyaluronan to produce synovial fluid.

DEVELOPMENT OF NERVE TISSUE


Nervous tissue develops in the early embryo when the dorsal
ectoderm neural plate folds lengthwise to form the neural tube,
the precursor of the CNS, and releases neural crest cells,
precursors for much of the PNS.

AMUAN, BERJAMIN, ERMITANIO, LARIOSA, RESURRECCION – BSMLS2B 23


histology
LECTURE / INSTRUCTOR: JURA SORNILLO JUNQUEIRA’S BASIC HISTOLOGY TEXT AND ATLAS

Neural tube is the primordium of the brain and spinal cord, and Cell body – acts as a trophic center, producing cytoplasm for
the process of its formation is called neurulation. the nerve processes.
NEURONS Nissl bodies (chromatin granules), intensely basophilic, occupy
Neuron - the functional unit in both the CNS and PNS is the the perikaryon and dendrites but not the axon hillock. They are
neuron. more prominent in motor than in sensory neurons.
Neurons consist of: Neurofilaments - are proteins selectively expressed in the
cell body (perikaryon) - containing the nucleus, cytoskeleton of neurons, and increased levels are a marker of
damage.
axon - a long cytoplasmic extension, and
dendrites - one or more shorter processes covered with many
Dendrites - the principal signal reception and processing sites
synapses. on neurons.
*Neurons can be classified according to the number of Dendritic spine - a small, club-like cell protrusion from
processes extending from the cell body: neuronal dendrites that form the postsynaptic component of
-- Multipolar neurons, each with one axon and two or more most excitatory synapses in the brain.
dendrites, are the most common. *Cross-linked with certain fixatives and impregnated with silver
-- Bipolar neurons, with one dendrite and one axon, comprise stains, neurofilaments are also referred to as neurofibrils by light
the sensory neurons of the retina, the olfactory epithelium, and microscopists.
the inner ear. *Changes in dendritic spines are of key importance in the
-- Unipolar or pseudounipolar neurons, which include all other constant changes of the neural plasticity that occurs during
sensory neurons, each have a single process that bifurcates embryonic brain development and underlies adaptation,
close to the perikaryon, with the longer branch extending to a learning, and memory postnatally
peripheral ending and the other toward the CNS.
-- Anaxonic neurons, with many dendrites but no true axon, do Axon – vary in length and diameter according to its type.
not produce action potentials, but regulate electrical changes of Axolemma - the plasma membrane of the axon the and its
adjacent CNS neurons. contents are known as axoplasm.
Axon hillock - where axons originate from a pyramid-shaped
region of the perikaryon.
Axon collaterals - An axon typically develops side branches, so
that one neuron can send information to several others. These
collaterals, just like the roots of a tree, split into smaller
extensions called terminal bouton. Each of these has a synaptic
terminal on the tip.
*Axons generally branch less profusely than dendrites, but do
undergo terminal arborization, a multiple branching at the end
of a nerve fiber.
Anterograde transport - transport from the soma to the distal
axon through kinesin.
Retrograde transport - transport from distal regions back to the
soma via dyenein.
Sensory neurons are afferent, receiving stimuli from receptors
Nerve impulse, or an action potential - an electrochemical
throughout the body.
Motor neurons are efferent, sending impulses to effector process initiated at the axon hillock when other impulses
organs such as muscle fibers and glands. received at the cell body or dendrites meet a certain threshold.
Somatic motor nerves are under voluntary control and typically *The action potential is propagated along the axon as a wave of
innervate skeletal muscle. membrane depolarization produced by voltage-gated Na+ and
Autonomic motor nerves - control the involuntary or K+ channels in the axolemma that allow diffusion of these ions
unconscious activities of glands, cardiac muscle, and smooth into and out of the axoplasm.
muscle. Resting potential - the imbalance of electrical charge that exists
Interneurons - establish relationships among other neurons, between the interior of electrically excitable neurons (nerve
forming complex functional networks or circuits in the CNS. cells) and their surroundings.

AMUAN, BERJAMIN, ERMITANIO, LARIOSA, RESURRECCION – BSMLS2B 24


histology
LECTURE / INSTRUCTOR: JURA SORNILLO JUNQUEIRA’S BASIC HISTOLOGY TEXT AND ATLAS

Depolarization – a process when the inside of the cell becomes Astrocytes, the most numerous cell of the CNS, all produce
less negative. hundreds of processes to cover and provide regulated
microenvironments for neuronal perikarya, synapses, and
capillaries.

Ependymal cells are


epithelial-like cells, lacking
basement membranes,
which line the fluid-filled
cerebral ventricles and
central canal of the spinal
cord.

Synapse – where nerve impulses are transmitted from one Microglia differs from all other
neuron to another, or from neurons and other effector cells. glial cells in originating from
*Neurotransmitter is released at the presynaptic membrane and blood monocytes, not from
binds receptors on the postsynaptic cell, initiating a new action neural tissue precursors; they
potential there; synaptic cleft separates these presynaptic and mediate immune defense
postsynaptic membranes. activity within the CNS.
*Major categories of neurotransmitters:
-- Amino acids, such as glutamate and γ-aminobutyrate (GABA)
-- Monoamines, such as serotonin (5-hydroxytryptamine or 5- Schwann cells (neurolemmocytes) enclose all axons in nerves
HT) and catecholamines, such as dopamine, all of which are of the PNS, producing myelin sheaths around large-diameter
synthesized from amino acids axons, whose impulse conductivity is augmented at the nodes
-- Polypeptides, such as endorphins and substance P. of Ranvier between successive Schwann cells.

Satellite cells are located


GLIAL CELLS AND NEURONAL ACTIVITY within PNS ganglia,
Glial cells support neuronal survival and activities, and are 10 aggregated sensory or
times more abundant than neurons in the mammalian brain. autonomic neuronal cell
Oligodendrocytes wrap processes around portions of axons in bodies, where they enclose
the CNS, forming myelin sheaths that insulate the axons and each perikaryon and regulate
facilitate nerve impulses. its microenvironment.

AMUAN, BERJAMIN, ERMITANIO, LARIOSA, RESURRECCION – BSMLS2B 25


histology
LECTURE / INSTRUCTOR: JURA SORNILLO JUNQUEIRA’S BASIC HISTOLOGY TEXT AND ATLAS

CENTRAL NERVOUS SYSTEM Cerebral nuclei - contain large numbers of aggregated neuronal
*The major structures comprising the CNS are the cell bodies.
cerebrum, Cerebral cortex - function in the integration of sensory
information and the initiation of voluntary motor responses.
Cerebellar cortex - coordinates muscular activity throughout the
body and is organized with three layers.
Pyramidal neuron - important neurons of the cerebrum w/c
transforms synaptic inputs into a patterned output of action
potentials.
*3 layers of cerebral cortex:
molecular layer (outer),
Purkinje cells (middle),
granular layer (inner)
cerebellum, *In cross sections of the spinal cord, the white matter is
peripheral and the gray matter forms a deeper.

Anterior horns - contain cell bodies of very large motor neurons


whose axons make up the ventral roots of spinal nerves.
Posterior horns - contain interneurons which receive sensory
fibers from neurons in the spinal (dorsal root) ganglia.
Central canal - continuous with the ventricles of the brain, is
lined by ependymal cells, and contains CSF.
*The skull and the vertebral column protect the CNS, but
between the bone and nervous tissue are membranes of
spinal cord.
connective tissue called the meninges.
*Three meningeal layers are distinguished:
o dura, consists of dense irregular connective tissue
organized as an outer periosteal layer continuous with the
periosteum of the skull and an inner meningeal layer.
*The dural venous sinuses lie between the periosteal and
meningeal layers of the dura mater.
*The epidural space is the area between the dura mater (a
membrane) and the vertebral wall, containing fat and small
blood vessels.
*The CNS is completely covered by connective tissue layers, the *The dural venous sinuses lie between the periosteal and
meninges, but CNS tissue contains very little collagen. meningeal layers of the dura mater.
*The central nervous system of the brain is made up of two o arachnoid, layer contains much CSF, which helps cushion
kinds of tissue: the CNS within its bony enclosure.
-- grey matter contains the cell bodies, dendrites and the axon *Subarachnoid usually refers to the space between
terminals, where all synapses are. the arachnoid and the pia mater that contains CSF.
-- white matter is made up of axons, which *Arachnoid villi act as one-way valves for the flow of CSF into
connect different parts of grey matter to each other; the main venous blood, and hydrostatic pressure is the main stimulus
components of white matter are myelinated axons often that causes these valves to open.
grouped together as tracts. *Subarachnoid usually refers to the space between
*Gray matter makes up the thick cortex or surface layer of both the arachnoid and the pia mater that contains CSF.
the cerebrum and the cerebellum; most white matter is found in o pia mater consists of flattened, mesenchymally derived
deeper regions. cells closely applied to the entire surface of the CNS tissue.

AMUAN, BERJAMIN, ERMITANIO, LARIOSA, RESURRECCION – BSMLS2B 26


histology
LECTURE / INSTRUCTOR: JURA SORNILLO JUNQUEIRA’S BASIC HISTOLOGY TEXT AND ATLAS

*The innermost pia mater Blood vessels penetrate CNS tissue


through long perivascular spaces, (also known as a Virchow–
*Robin space, is a fluid-filled space surrounding certain blood
vessels) covered by pia mater.
Unmyelinated nerve fibers conduct impulses at low velocities.
The blood-brain barrier (BBB) is a functional barrier that allows
They represent the majority of peripheral sensory and
much tighter control than that in most tissues over the passage
autonomic fibers. They are also found in the spinal cord and
of substances moving from blood into the CNS tissue.
brain.
Capillary endothelium - main structural component of the BBB
*In most CNS regions, neurons are also protected by the blood- Endoneurium is a thin connective tissue layer immediately
brain barrier, consisting of the perivascular feet of astrocytic surrounding Schwann cells in peripheral nerves, containing a
processes and the nonfenestrated capillary endothelial cells’ few nonfenestrated capillaries and much reticulin.
tight junctions.
Choroid plexus - consists of elaborate folds of vascularized pia Groups of axons with Schwann cells and endoneurium are
mater covered by ependymal, that project from walls of the bundled together as fascicles by a sleeve of perineurium.
cerebral ventricles; there water is removed from capillaries and The Blood Nerve Barrier (BNB) consists of the endoneurial
transferred into the ventricles as CSF. microvessels within the nerve fascicle and the investing
Arachnoid villi - provide the main pathway for absorption of CSF perineurium.
back into the venous circulation.
Externally, peripheral nerves have a dense, irregular fibrous coat
PERIPHERAL NERVOUS SYSTEM called the epineurium, which extends deeply to fill the space
The main components of the peripheral nervous system (PNS)
between fascicles.
are the nerves, ganglia, and nerve endings. Peripheral nerves
are bundles of nerve fibers (axons) individually surrounded by Ganglia, which can be either sensory or autonomic, contain
Schwann cells and connective tissue. neuronal cell bodies and their satellite cells and are surrounded
by connective tissue continuous with that of nerves.
Nerves are analogous to tracts in the CNS, containing axons
enclosed within sheaths of glial cells. NEURAL PLASTICITY AND REGENERATION
Certain regions of the CNS, such as near the ependyma, retain
In peripheral nerves, axons are sheathed by Schwann cells or rare neural stem and progenitor cells that allow some
neurolemmocytes. replacement of neurons throughout life; neural plasticity
As axons of large diameter grow in the PNS, they are engulfed involving formation and remodeling of synaptic connections is
along their length by a series of differentiating neurolemmocytes also prevalent throughout life.
and become myelinated nerve fibers. The complexity and distances of the neuronal and glial
The multiple layers of Schwann cell membrane unite as a thick interconnections with the CNS make regeneration and
myelin sheath. restoration of function within this tissue after major injury very
difficult.
The major dense line (MDL) is a 3-nm compartment between
two cytoplasmic leaflets of stacked myelin membranes, mostly The more simply organized peripheral nerves have better
occupied by a myelin basic protein (MBP) phase. capacity for axonal regeneration, a process involving
reactivation of the perikaryon, Schwann cells, and macrophages.
Myelin clefts which are conical tube-like cytoplasmic structures
that cross the compact myelin and connect the Schwann cell
peripheral (abaxonal) cytoplasm to the periaxonal (adaxonal)
cytoplasm.

Between adjacent Schwann cells on an axon the myelin sheath


shows small nodes of Ranvier where the axon is only partially
covered by interdigitating Schwann cell processes.

AMUAN, BERJAMIN, ERMITANIO, LARIOSA, RESURRECCION – BSMLS2B 27


histology
LECTURE / INSTRUCTOR: JURA SORNILLO JUNQUEIRA’S BASIC HISTOLOGY TEXT AND ATLAS

CHAPTER 10: MUSCLE TISSUE Myofibrils - smaller subunits composing each muscle fiber that
Functions: extend the entire length of the fiber.
-- Movement Myofilament - composes the myofibrils formed by the
-- Maintenance of posture contractile thin protein actin and the thick protein myosin.
-- Joint stabilization Muscle satellite cells - small population of reserve progenitor
-- Heat generation cells that remains adjacent to most fibers of differentiated
*Actin microfilaments and associated proteins generate the skeletal muscle.
forces necessary for the muscle contraction, which drives
movement within organ systems, of blood, and of the body as a
whole.
*All muscle cells are of mesodermal origin and differentiate by
a gradual process of cell lengthening with abundant synthesis
of the myofibrillar proteins actin and myosin.
Fibers – elongated cells
Sarcoplasm (Gr. sarkos = flesh + plasma = thing formed) –
cytoplasm of muscle cells
Sarcoplasmic reticulum – smooth ER of muscle cells
Sarcolemma (sarkos + Gr. lemma = husk) – cell membrane
(plasmalemma) and its external lamina of the muscle cells.
*3 types of muscle tissue: skeletal, cardiac, smooth.
SKELETAL MUSCLE Organization of a skeletal muscle:
Skeletal (striated) muscle) Three thin layers of connective tissue surrounds and organizes
- consists of muscle fibers, the contractile fibers:
which are long, cylindrical o epimysium - external sheath of dense irregular connective
multinucleated cells with tissue, surrounds the entire muscle. Septa of this tissue
diameters of 10-100 μm. extend inward, carrying the larger nerves, blood vessels,
*During embryonic muscle and lymphatics of the muscle.
development, mesenchymal o perimysium - thin connective tissue layer that immediately
myoblasts (L. myo = surrounds the fascicle (bundle of muscle fibers).
muscle) fuse, forming o endomysium - a very thin, delicate layer of reticular fibers
myotubes with many nuclei and scattered fibroblasts surrounds the external lamina of
which further differentiate individual muscle fiber that brings O2 to the muscle fibers.
to form striated muscle *Collagens in the connective tissue of the muscle serve to
fibers. transmit the mechanical forces generated by the contracting
*Elongated nuclei - found muscle cells/fibers; individual muscle fibers seldom extend from
peripherally just under the one end of a muscle to the other
sarcolemma, a characteristic nuclear location unique to skeletal Myotendinous junctions - joins the muscle to bone, skin, or
muscle fibers/cells. another muscle.
*Development of skeletal muscle: Organization within muscle fibers:
-- Begins when myoblast (mesenchymal cells) differentiate.
aligning and fusing together to make myotubes (longer,
multinucleated tubes).
-- Myotubes synthesize the proteins to make up myofilaments
and gradually begin to show cross-striations by light
microscopy.
-- Myotubes continue differentiating to form functional
myofilaments, and the nuclei are displaced against the
sarcolemma.

AMUAN, BERJAMIN, ERMITANIO, LARIOSA, RESURRECCION – BSMLS2B 28


histology
LECTURE / INSTRUCTOR: JURA SORNILLO JUNQUEIRA’S BASIC HISTOLOGY TEXT AND ATLAS

*Longitudinally sectioned skeletal muscle fibers show striations Myosin heads - bind both actin, forming transient crossbridges
of alternating light and dark bands. The sarcoplasm is highly between the thick and thin filaments, and ATP, catalyzing energy
organized, containing myofibrils (primarily long cylindrical release (actomyosin ATPase activity).
filament bundles that run parallel to the long axis of the fiber
o A-bands (anisotropic or birefringent in polarized light
microscopy) - dark bands on the myofibrils
o I-bands (isotropic, do not alter polarized light) – light bands
o H zone – lighter zone in the A band that has only the rod-
like portions of the myosin molecule and no thin filaments
o M-line (Ger. Mitte = middle) – bisects the H zone that
contains myomesin and creatine kinase

*Containing a myosin-binding protein myomesin that holds the


thick filaments in place, and creatine kinase catalyzes transfer of
phosphate groups from phosphocreatine, a storage form of
high-energy phosphate groups, to ADP, helping to supply ATP
for muscle contraction Helical actin filaments - are 1.0-μm long and 8-nm wide and
*The banding pattern of A and I of sacromeres is due to the run between the thick filaments.
regular symmetric arrangement of myofilaments (myosin & F- G-actin monomer - contains a binding site for myosin
actin).
o Z-line or Z disc (Ger. Zwischen = between) - a dense dark *Thin filaments have two tightly associated regulatory proteins:
transverse line that bisects I bands. Tropomyosin - 40-nm-long coil of two polypeptide chains
located in the groove between the two twisted actin strands
Troponin - a complex of three subunits: TnT (attaches to
tropomyosin); TnC (binds Ca2+) and TnI (regulates the actin-
myosin interaction).

Sarcomere - smallest structural and functional contractile unit


of the muscle that can be found between two adjacent Z lines.
It is a repeating contractile units seen along the entire length of Organization of important myofibril components:
each myofibril and are highly characteristic features of the o α-actinin - binds thin actin filaments to dense Z disc and
sarcoplasm of skeletal and cardiac muscle fibers. They are about exhibit opposite polarity on each side of the disc
2.5-μm long in resting muscle. o titin (3700 kDa) - large accessory proteins in the I band that
Myosin - a large complex (~500 kDa) with two identical heavy anchors, positions, and centers myosin filaments and the Z
chains and two pairs of light chains. line. It also acts as a spring between the end of the myosin
Myosin filament - are 1.6-μm long and 15-nm wide; they occupy filament and the Z line.
the A band at the middle region of the sarcomere. o nebulin - extends the length of the thin filaments actin and
Myosin heavy chains - thin, rod-like motor proteins (150-nm anchors them to the Z disc laterally. It regulates the length
long and 2-3 nm thick) twisted together as myosin tails of the actin polymers during myogenisis.
Myosin light chains - can be found in globular projection. Four o desmin - acts as a support that extends from Z line of one
of these will form a head at one end of each heavy chain. myofibril to the adjacent myofibril, linking them together
and attaching them to the sarcolemma which stabilizes the
position of muscle myofibrils within the sarcoplasm.

AMUAN, BERJAMIN, ERMITANIO, LARIOSA, RESURRECCION – BSMLS2B 29


histology
LECTURE / INSTRUCTOR: JURA SORNILLO JUNQUEIRA’S BASIC HISTOLOGY TEXT AND ATLAS

o myomesin - holds the thick filaments in place


o creatine kinase - catalyzes transfer of phosphate groups
from phosphocreatine (storage form of high-energy
phosphate groups) to ADP, helping to supply ATP for
muscle contraction.
o sarcoplasmic reticulum - membranous smooth ER that
contains pumps and other proteins for Ca2+ sequestration
and surrounds the myofibrils

Principle:
*Calcium release from cisternae of the sarcoplasmic reticulum
through voltage-gated ca2+ channels is triggered by membrane
depolarization produced by a motor nerve
*To trigger ca2+ release from sarcoplasmic reticulum
throughout the muscle fiber simultaneously and produce
uniform contraction of all myofibrils

Transverse or T-tubules - tubular infoldings long fingerlike 1 - A nerve impulse triggers release of ACh from the synaptic
invaginations of the cell membrane penetrate deeply into the knob into the synaptic cleft. ACh binds to ACh receptors in the
sarcoplasm and encircle each myofibril near the aligned A- and motor end plate of the neuromuscular junction, initiating a
I-band boundaries of sarcomeres. muscle impulse in the sarcolemma of the muscle fiber.
Terminal cisternae (Triad in TEM) - Adjacent to each T-tubule 2 - As the muscle impulse spreads quickly from the sarcolemma
that allows depolarization of the sarcolemma in a T-tubule to along T tubules, calcium ions are released from terminal
affect the sarcoplasmic reticulum and trigger release of Ca2+ cisternae into the sarcoplasm.
ions into cytoplasm around the thick and thin filaments, which 3 - Calcium ions bind to troponin. Troponin changes shape,
initiates contraction of sarcomeres. moving tropomyosin on the actin to expose active sites on actin
Mechanism of contraction: molecules of thin filaments. Myosin heads of thick filaments
*Ca2+ binding to troponin causes tropomyosin to change shape attach to exposed active sites to form cross bridges.
and allow the myosin heads to bind the actin subunits, forming 4 - When the impulse stops, calcium ions are actively
crossbridges between thick and thin filaments. transported into the sarcoplasmic reticulum, tropomyosin re-
*The myosin heads then pivot with ATP hydrolysis, which pulls covers active sites, and filaments passively slide back to their
the thin filaments along the thick filaments. relaxed state.
*With Ca2+ and ATP present, a contraction cycle ensues in 5 - When the impulse stops, calcium ions are actively
which myosin heads repeatedly attach, pivot, detach, and return, transported into the sarcoplasmic reticulum, tropomyosin re-
causing the filaments to slide past one another, shortening the covers active sites, and filaments passively slide back to their
sarcomere relaxed state.
*When the membrane depolarization ends, Ca2+ is again
sequestered, ending contraction and allowing the sarcomeres to *Muscle spindles & tendons:
lengthen again as the muscle relaxes. Proprioceptors (L. proprius = one’s own + capio = to take)
*Synapses of motor axons with skeletal muscle are called motor - sensory receptors in Striated muscles and myotendinous
end plates (MEPs), neuromuscular junctions (NMJs), or junctions that provides central nervous system (CNS) with data
myoneural junctions; the neurotransmitter is acetylcholine. from the musculoskeletal system
*A motor axon may form many terminal branches, each ending Muscle spindles - stretch detectors in the muscle fascicles that
on an MEP of a muscle fiber; all fibers innervated by branches is approximately 2-mm long and 0.1-mm wide
of that axon comprise a motor unit. Intrafusal fibers - thin muscle fibers filled with nuclei found in
perimysium, with concentric layers of flattened cells, and
interstitial fluid
Golgi tendon organs - much smaller encapsulated structures
that enclose sensory axons penetrating among the collagen

AMUAN, BERJAMIN, ERMITANIO, LARIOSA, RESURRECCION – BSMLS2B 30


histology
LECTURE / INSTRUCTOR: JURA SORNILLO JUNQUEIRA’S BASIC HISTOLOGY TEXT AND ATLAS

bundles at the myotendinous junction. It detects changes in Fast glycolytic fibers - specialized for rapid, shortterm
tension within tendons produced by muscle contraction and act contraction, having few mitochondria or capillaries and
to inhibit motor nerve activity if tension becomes excessive. depending largely on anaerobic metabolism of glucose derived
*Both help regulate the amount of effort required to perform from stored glycogen, features that make such fibers appear
movements that call for variable amounts of muscular force. white. Rapid contractions lead to rapid fatigue as lactic acid
produced by glycolysis accumulates
Fast oxidative-glycolytic fibers - have physiological and
histological features intermediate between those of the other
two types.
CARDIAC MUSCLE
*Form complex
junctions between
interdigitating
processes.
Mature cardiac
muscle cells - 15-
30 μm in diameter
and 85-120 μm
long, with a striated
banding pattern
comparable to that
of skeletal muscle
Cardiac muscle cell - usually has only one nucleus and is
centrally located.
Cardiac muscle fibers - are also striated, but they consist of
individual cylindrical cells, each containing one (or two) central
nuclei and linked by adherent and gap junctions at prominent
intercalated discs.
Intercalated discs - represent the interfaces between adjacent
cells and consist of many junctional complexes. Transverse
regions of these irregular, steplike discs are composed of many
desmosomes and fascia adherens junctions, which together
provide strong intercellular adhesion during the cells’ constant
contractile activity.
*Unique characteristic of cardiac muscle is the presence of
transverse lines that cross the fibers at irregular intervals where
the myocardial cells join
Secretory granules - 0.2-0.3 μm in diameter are found near
atrial muscle nuclei and are associated with small Golgi
complexes.
- these release the peptide hormone atrial natriuretic factor
(ANF) that acts on target cells in the kidney to affect Na+
*Skeletal Muscle Fiber Types: excretion and water balance.
Slow oxidative muscle fibers - adapted for slow contractions *The contractile cells of the heart’s atria thus also serve an
over long periods without fatigue, having many mitochondria, endocrine function
many surrounding capillaries, and much myoglobin, all features
that make fresh tissue rich in these fibers dark or red in color.

AMUAN, BERJAMIN, ERMITANIO, LARIOSA, RESURRECCION – BSMLS2B 31


histology
LECTURE / INSTRUCTOR: JURA SORNILLO JUNQUEIRA’S BASIC HISTOLOGY TEXT AND ATLAS

*Fibers of smooth muscle (also called visceral muscle) are


elongated, tapering, and unstriated cells, each of which is
enclosed by an external lamina and a network of type I and type
III collagen fibers comprising the endomysium
Smooth muscle cells - 20 μm (small vessel) - 500 μm (pregnant
uterus)
Nucleus - single and elongated in the central broadest part. Its
diameter is 5-10 μm
Peristalsis - contract in a coordinated manner to produce a
wave that moves the tract’s contents.

SMOOTH MUSCLE
Smooth muscle - is a major
component of blood vessels
and of the digestive,
respiratory, urinary, and
reproductive tracts and their
associated organs.
*Specialized for slow, steady
contraction under the
influence of autonomic nerves
and various hormones.

AMUAN, BERJAMIN, ERMITANIO, LARIOSA, RESURRECCION – BSMLS2B 32


histology
LECTURE / INSTRUCTOR: JURA SORNILLO JUNQUEIRA’S BASIC HISTOLOGY TEXT AND ATLAS

*Regeneration of muscle tissue: Adipose tissue


Skeletal muscle - although the multinucleated cells cannot
undergo mitosis, the tissue can still display limited regeneration.
*The source of regenerating cells is the sparse population of
mesenchymal satellite cells lying inside the external lamina of
each muscle fiber
Principle:
*After injury the normally quiescent satellite cells become
activated, proliferating and fusing to form new skeletal muscle
fibers.
*Similar activity of satellite cells has been implicated in muscle
growth after extensive exercise, a process in which they fuse
Anaphase
with existing fibers to increase muscle mass beyond that which
occurs by cell hypertrophy.
*Following major traumatic injuries, scarring and excessive
connective tissue growth interferes with skeletal muscle
regeneration.
Cardiac muscle - lacks satellite cells and shows very little
regenerative capacity beyond early childhood. Defects or Simple squamous epithelial cells
damage (infarcts) to heart muscle are generally replaced by
proliferating fibroblasts and growth of connective tissue,
forming only myocardial scars.
Smooth muscle - composed of simpler, smaller, mononucleated
cells, is capable of a more active regenerative response. After
injury, viable smooth muscle cells undergo mitosis and replace
the damaged tissue. As discussed in Chapter 11, contractile
Generate and conduct
pericytes from the walls of small blood vessels participate in the Axon
nerve impulses to
repair of vascular smooth muscle
other cells
REVIEW:

Simple cuboidal epithelium


Pseudostratified epithelium
found in Trachea

Cardiac muscle

Skeletal muscle

AMUAN, BERJAMIN, ERMITANIO, LARIOSA, RESURRECCION – BSMLS2B 33


histology
LECTURE / INSTRUCTOR: JURA SORNILLO JUNQUEIRA’S BASIC HISTOLOGY TEXT AND ATLAS

Stratified squamous CHAPTER 11: THE CIRCULATORY SYSTEM


nonkeratinized The circulatory system pumps and directs blood cells and
epithelium substances carried in blood to all tissues of the body.
*Consists of:
-- Heart which propels blood through the system
-- Arteries - series of vessels efferent from the heart that
become smaller as they branch into the various organs, carry
blood to the tissues
-- Capillaries - the smallest vessels, are the sites of O2, CO2,
nutrient, and waste product exchange between blood and
tissues.
Simple columnar epithelium Microvasculature or microvascular bed - complex network of
thin, anastomosing tubules
-- Veins - the convergence of venules into a system of larger
channels that continue enlarging as they approach the heart,
toward which they carry the blood to be pumped again.

Pulmonary circulation - where blood is oxygenated in the lungs


Systemic circulation - where blood brings nutrients and
removes wastes in tissues throughout the body
Lymphatic capillaries - are thin-walled, closed-ended tubules
carrying lymph, that merge to form vessels of steadily
increasing size; returns fluid from tissue spaces all over the
body to the blood.
*The internal surface of all components of the blood and
lymphatic systems is lined by a simple squamous epithelium
called endothelium.
HEART
Loose areolar connective tissue Right and Left ventricles - propel blood to the pulmonary and
systemic circulation
Right and Left atria - receive blood from the body and the
pulmonary veins
Superficial layer of large, dome like
*Three major layers of the heart:
cells that protect underlying tissues
o Endocardium consists of a thin inner layer of endothelium
from the hypertonic effects of urine. and supporting connective tissue.
-- subendocardial layer - a middle myoelastic layer of
smooth muscle fibers and connective tissue, and a deep
layer of connective tissue.
o Myocardium – the thickest layer of the heart which consists
mainly of cardiac muscle with its fibers arranged spirally
around each heart chamber.
o Epicardium - a simple squamous mesothelium supported
by a layer of loose connective tissue containing blood
Little ground substance with few vessels and nerves.
-- The epicardium corresponds to the visceral layer of the
cells and collagen fibers are
pericardium, the membrane surrounding the heart.
random in arrangement.
*Dense fibrous connective tissue of the cardiac skeleton forms
part of the interventricular and interatrial septa, surrounds all
valves of the heart.

AMUAN, BERJAMIN, ERMITANIO, LARIOSA, RESURRECCION – BSMLS2B 34


histology
LECTURE / INSTRUCTOR: JURA SORNILLO JUNQUEIRA’S BASIC HISTOLOGY TEXT AND ATLAS

such as heparin, tissue plasminogen activator, and von


Willebrand factor.
- cells regulate local vascular tone and blood flow by secreting
various factors that stimulate smooth muscle contraction or
relaxation.
- cells induce specific white blood cells to stop and undergo
transendothelial migration at sites of injury or infection. - cells
also secrete various factors called interleukins that affect the
activity of local white blood cells during inflammation.
- secrete various growth factors, including proteins promoting
proliferation of specific white blood cell lineages and cells that
make up the vascular wall.
Smooth muscle fibers - occur in the walls of all vessels larger
than capillaries and are arranged helically in layers. In arterioles
*Functions of dense irregular connective: and small arteries, the smooth muscle cells are connected by
-- Anchoring and supporting the heart valves. many more gap junctions and permit vasoconstriction and
-- Providing firm points of insertion for cardiac muscle. vasodilation which are of key importance in regulating the
--Helping coordinate the heartbeat by acting as electrical overall blood pressure.
insulation between atria and ventricles. Connective tissue components are present in vascular walls in
Conducting system of the heart - generates and propagates variable amounts and proportions based on local functional
waves of depolarization that spread through the myocardium to requirements:
stimulate rhythmic contractions. -- Elastic fibers provide the resiliency required for the vascular
*Consists of: wall to expand under pressure.
-- Sinoatrial (SA) node or pacemaker -- Elastin - major component in large arteries where it forms
-- Atrioventricular (AV) node with AV bundle (of His) and parallel lamellae, regularly distributed between the muscle
subendocardial conducting network. layers.
Purkinje fibers -- Collagen fibers are found in the subendothelial layer, between
- located just beneath the endocardium of both ventricles the smooth muscle layers, and in the outer covering
- are distinguished from contractile fibers by their greater *Variations in the amount and composition of ground substance
diameter, abundant glycogen, and more sparse bundles of components such as proteoglycans and hyaluronate also
myofibrils. contribute to the physical and metabolic properties of the wall
*Masses of dense irregular connective tissue make up the in different vessels, especially affecting their permeability
cardiac skeleton, which surrounds the bases of all heart valves, VASCULATURE
separates the atria from the ventricles, and provides insertions *Three major layers or tunics:
for cardiac muscle. (1) tunica intima includes the endothelium, connective tissue,
*Ganglionic nerve cells and nerve fibers are present in the and an internal elastic lamina in larger vessels;
regions close to the SA and AV nodes, where they affect heart (2) tunica media contains alternating layers of smooth muscle
rate and rhythm and collagen or elastic lamellae; and
*Stimulation of the parasympathetic division (vagus nerve) (3) tunica adventitia (or externa) contains connective tissue,
slows the heartbeat, whereas stimulation of the sympathetic small vessels (vasa vasorum), and nerves.
nerve accelerates activity of the pacemaker.
TISSUES OF THE VASCULAR WALL
Walls of all blood vessels except capillaries contain smooth
muscle and connective tissue in addition to the endothelial
lining.
Endothelium - a specialized epithelium that acts as a
semipermeable barrier between two major internal
compartments: the blood and the interstitial tissue fluid.
*Function:
- presents a nonthrombogenic surface on which blood will not
clot and actively secretes agents that control local clot formation

AMUAN, BERJAMIN, ERMITANIO, LARIOSA, RESURRECCION – BSMLS2B 35


histology
LECTURE / INSTRUCTOR: JURA SORNILLO JUNQUEIRA’S BASIC HISTOLOGY TEXT AND ATLAS

*Through the vasculature, endothelial cells are not simply heart (1) continuous capillaries with
and vessel liners; they actively produce factors that prevent many tight junctions so that all
blood clotting, factors that cause adjacent smooth muscle cells exchange must occur through
to contract or relax, and factors that initiate inflammation at sites the cells;
of damage or infection.

*Arteries are grouped by size and wall composition:


o large elastic arteries, with fenestrated elastic laminae in the
thick tunica media;
o muscular, medium-sized arteries; and
o small arteries, with fewer than 10 layers of smooth muscle
in the media.
(2) fenestrated capillaries with
*A microvasculature too small for surgical manipulation small pores or fenestrations
permeates most organs and consists of: through the cells; and
o arterioles, with one to three
smooth muscle layers;

(3) discontinuous capillaries,


or sinusoids, with larger
o capillaries, consisting only lumens, large spaces between
of an intima endothelial layer; the endothelial cells, and a
discontinuous basal lamina.

*Capillary beds generally drain into venules, the last segment of


the microvasculature; postcapillary venules are the sites at
o venules, with large lumens which white blood cells enter damaged or infected tissues.
and thin walls, which drain *The endothelium of continuous capillaries and postcapillary
capillaries. venules is frequently surrounded by thin cells called pericytes,
whose contractions facilitate blood flow and which can give rise
to smooth muscle and connective tissue during microvascular
remodeling or repair.
*Two alternative microvascular pathways include:
(1) arteriovenous anastomoses, or AV shunts, in which
arterioles can bypass a capillary bed, and
*Terminal arterioles branch into metarterioles, in which smooth (2) venous portal systems, in which venules draining a capillary
muscle sphincters contract to resist blood flow and relax bed quickly branch again to form another capillary bed.
cyclically to allow pulsatile flow of blood into an anastomosing LYMPHATIC VASCULAR SYSTEM
capillary bed, where metabolic exchange with surrounding cells Lymphatic Vessels - with thin walls and increasing amounts of
occurs. connective tissue and smooth muscle which never form clearly
distinct outer tunics, and have valves comprised of complete
*Capillaries are classified as three structural and functional intimal folds
types, with features that allow different degrees of molecular or *Interstitial fluid that is not pulled into venules by colloidal
even cellular exchange: osmotic pressure drains as lymph into blind vessels called
lymphatics, or lymphatic capillaries, which have very thin
endothelial cell walls with spaces between the cells.

AMUAN, BERJAMIN, ERMITANIO, LARIOSA, RESURRECCION – BSMLS2B 36


histology
LECTURE / INSTRUCTOR: JURA SORNILLO JUNQUEIRA’S BASIC HISTOLOGY TEXT AND ATLAS

CHAPTER 12: BLOOD


The liquid portion of circulating blood is plasma, while the cells
and platelets comprise the formed elements; upon clotting,
some proteins are removed from plasma and others are
released from platelets, forming a new liquid termed serum.

* Important protein components of plasma include:


-albumin,
-diverse α- and β-globulins,
-proteins of the complement system, and
-fibrinogen, all of which are secreted within the liver, as well as
*Lymphatics converge into larger, thin-walled lymphatic vessels
the immunoglobulins.
in which lymph is propelled by movements of surrounding
muscles and organs, with intimal valves keeping the flow
unidirectional. Red blood cells or erythrocytes
*The largest lymphatic vessels: - make up the hematocrit portion (~45%) of a blood sample
the thoracic duct and right lymphatic duct, both with walls - enucleated, biconcave discs 7.5 µm in diameter, filled with
having tunics like those of veins, return lymph to the circulatory hemoglobin for the uptake, transport, and release of O2, and
system by joining veins near the heart. with a normal life span of about 120 days.

White blood cells or leukocytes


- broadly grouped as granulocytes (neutrophils, eosinophils,
basophils) or agranulocytes (lymphocytes, monocytes).
*All leukocytes become active outside the circulation,
specifically leaving the microvasculature in a process involving
cytokines, selective adhesion, changes in the endothelium, and
transendothelial migration or diapedesis.
*All granulocytes have specialized lysosomes called azurophilic
granules and smaller specific granules with proteins for various
cell-specific functions.
o Neutrophils, the most abundant type of leukocyte, have
polymorphic, multilobed nuclei, and faint pink cytoplasmic
granules that contain many factors for highly efficient
phagolysosomal killing and removal of bacteria.

AMUAN, BERJAMIN, ERMITANIO, LARIOSA, RESURRECCION – BSMLS2B 37


histology
LECTURE / INSTRUCTOR: JURA SORNILLO JUNQUEIRA’S BASIC HISTOLOGY TEXT AND ATLAS

o Eosinophils have bilobed nuclei and eosinophilic specific


granules containing factors for destruction of helminthic
parasites and for modulating inflammation.

o Monocytes are larger agranulocytes with distinctly indented


or C-shaped nuclei that circulate as precursors of
macrophages and other cells of the mononuclear
phagocyte system.
o Basophils, the rarest type of circulating leukocyte, have
irregular bilobed nuclei and resemble mast cells with
strongly basophilic specific granules containing factors
important in allergies and chronic inflammatory conditions,
including histamine, heparin, chemokines, and various
hydrolases.

o Platelets are small (2-4 µm) cell fragments derived from


megakaryocytes in bone marrow, with a marginal bundle
of actin filaments, alpha granules and delta granules, and
an open canalicular system of membranous vesicles; rapid
degranulation on contact with collagen triggers blood
clotting.
o Lymphocytes, agranulocytes with many functions as T- and
B-cell subtypes in the immune system, range widely in size,
depending on their activation state, and have roughly
spherical nuclei with little cytoplasm and few organelles.

AMUAN, BERJAMIN, ERMITANIO, LARIOSA, RESURRECCION – BSMLS2B 38


histology
LECTURE / INSTRUCTOR: JURA SORNILLO JUNQUEIRA’S BASIC HISTOLOGY TEXT AND ATLAS

CHAPTER 13: HEMOPOIESIS *2 major lineages of progenitor cells:


Hemopoiesis AKA Hematopoiesis 1. Lymphoid cells - migrate from the bone marrow to the
- the formation of blood cells. thymus or the lymph nodes, spleen, and other lymphoid
- occurs in the bone marrow of adults and children structures, where they proliferate and differentiate.
STEM CELLS, GROWTH FACTORS, & DIFFERENTIATION 2. Myeloid cells - includes granulocytes, monocytes,
Hemopoiesis, Development, and Aging erythrocytes, and megakaryocytes.

Progenitor & Precursor Cells


Colony Forming Units (CFU) - is the progenitor cells for blood
cells because they give rise to colonies of only one cell type
when cultured in vitro or injected into a spleen.
*Four Major types of progenitor cells/CFUs:
1. Erythroid lineage of erythrocytes
2. Thrombocytic lineage of megakaryocytes for platelet
formation
3. Granulocyte-monocyte lineage of all three granulocytes and
monocytes
4. Lymphoid lineage of B lymphocytes, T lymphocytes, and
natural killer cells
Progenitor cells - committed to forming each type of mature
blood cell, proliferate and differentiate within
microenvironmental niches of stromal cells, other cells, and
Origin and Maturation of Cells: ECM with specific growth factors.
Erythropoiesis = Erythrocytes Colony stimulating Factors (CSF)
Granulopoiesis = Granulocytes - hemopoietic growth factor
Monocytopoiesis = Monocytes - also known as cytokines that stimulate proliferation of
Thrombocytopoiesis = Platelets progenitor and precursor cells and promote cell differentiation
Lymphopoiesis = Lymphocytes and maturation within specific lineages.
BONE MARROW
Pluripotent cells -found in the medullary canals of long bones and in the small
- capable of asymmetric division and self- renewal. cavities of cancellous bones.
Hemopoietic stem cells - can be isolated by using fluorescence- Red bone marrow – an active hemopoiesis
labeled antibodies to mark specific cell surfaces antigens and
passing the cell population through a Fluorescence-activated
cell-sorting (FACS).
*Stem cells are studied using experimental techniques that
permit analysis of hemopoiesis in vivo and in vitro.
In vivo technique - injecting the bone marrow of normal donor
mice into irradiated mice whose hematopoietic cells have been
destroyed.
In vitro technique - uses semisolid tissue culture media
containing substances to identify and study cytokines promoting
hemopoietic cell growth and differentiation.
-contains a reticular connective tissue stroma (Gr. stroma, bed),
Hemopoeitic Stem cells - single type of pluripotent in the bone hemopoietic cords or islands of cells, and sinusoidal
marrow that can give rise to all the blood cell types. capillaries. The stroma is a meshwork of specialized fibroblastic
cells called stromal cells (also called reticular or adventitial
cells) and a delicate web of reticular fibers supporting the
hemopoietic cells and macrophages.

AMUAN, BERJAMIN, ERMITANIO, LARIOSA, RESURRECCION – BSMLS2B 39


histology
LECTURE / INSTRUCTOR: JURA SORNILLO JUNQUEIRA’S BASIC HISTOLOGY TEXT AND ATLAS

Yellow bone marrow – consists mostly of adipocytes *Erythropoiesis requires approximately a week and involves
three to five cell division between the progenitor cell stage and
the release of functional cells into the circulation.
Erythropoietin
- a growth factor produced by cells in the kidneys, stimulates
production of mRNA for the protein components of hemoglobin
and is essential for the erythrocyte production.
Proerythroblasts
- first stage of erythrocyte maturation
- distinct erythroid progenitor cell
- a large cell with loose, lacy chromatin, nucleoli, and basophilic
cytoplasm.
Basophilic Erythroblasts - slightly smaller with cytoplasmic
basophilia and a more condensed nucleus.
Polychromatophilic erythroblast
*Newborn all bone marrow is red, but as child grows, most of - cell volume is reduced, polysomes decrease, and some
the marrow changes gradually to the yellow variety. Severe cytoplasmic areas begin to be filled with hemoglobin, producing
bleeding or hypoxia, yellow marrow reverts to red. regions of both basophilia and acidophilia in the cell
*The hematopoietic niche in marrow includes the stroma, Orthochromatophilic Erythroblasts
osteoblasts, and megakaryocytes. Between the hematopoietic - also called as normoblasts
cords run the sinusoids, which have discontinuous endothelium, - cell and nuclear volumes continue to condense and basophilia
through which newly differentiated blood cells and platelets is gradually lost, producing cells with uniformly acidophilic
enter the circulation. cytoplasm.
MATURATION OF ERYTHROCYTES Reticulocytes
-Erythrocyte maturation is an example of terminal cell - late in this stage the nucleus is ejected and then undergoes
differentiation involving hemoglobin synthesis and formation of phagocytosis by macrophages.
a small, enucleated, biconcave corpuscle. - cell still retains a few polyribosomes which, when treated with
-It takes 7-9 days for formation of mature RBC the dye brilliant cresyl blue, form a faintly stained network.

Changes during Erythropoiesis


1. Cell and nuclear volume decrease
2. Nucleoli diminish in size and disappear
3. Chromatin density increases until a nucleus presents a
pyknotic appearance and is finally extruded from the cell.
4. Gradual decrease of polyribosomes
5. Simultaneous increase of hemoglobin
6. Mitochondria and other organelles gradually disappear.

AMUAN, BERJAMIN, ERMITANIO, LARIOSA, RESURRECCION – BSMLS2B 40


histology
LECTURE / INSTRUCTOR: JURA SORNILLO JUNQUEIRA’S BASIC HISTOLOGY TEXT AND ATLAS

*Erythropoietic islands or cords within marrow contain the red


blood cell lineage: proerythroblasts, erythroblasts with
succeeding developmental stages called basophilic,
polychromatophilic, and orthochromatophilic that reflect the
cytoplasmic transition from RNA-rich to hemoglobin-filled.
*At the last stage of erythropoiesis cell nuclei are extruded,
producing reticulocytes that still contain some polyribosomes
but are released into the circulation.

MATURATION OF GRANULOCYTES
Granulopoieisis - involves cytoplasmic changes dominated by
synthesis of proteins for the azurophilic granules and specific
granules. MATURATION OF AGRANULOCYTES
Monocyte
- progenitor cell is Monoblast, further differentiation of
monoblast leads to Promonocyte, they divide and
Monocyte develops. They enter tissues and mature as
macrophages. From round to kidney shaped to indented
nucleus.
Lymphocyte
- originate mainly from thymus, but progenitor cells
originate from in the bone marrow. Progenitor is
lymphoblast, capable of dividing 2-3 times to form
lymphocytes.
Prolymphocytes
- clumped chromatin and abundant cytoplasm.
Lymphocyte
Myeloblast - from bone marrow, some are from thymus where they
- most immature recognizable cell in the myeloid mature to T lymphocytes. Lymphocytes remained in the
series. bone marrow mature to B lymphocytes.
Promyelocyte
- basophilic cytoplasm, azurophilic granules and Platelets - originate by fragmentation from megakaryocytes,
myeloperoxidase; first to be distinguished differentiate from megakaryoblast.
Megakaryocytes - contains mitochondria, RER and golgi
Myelocyte
apparatus. Best seen in bone marrow, also occur in interstitial
- increase in specific granules. Last stage of cell division. tissue of spleen and lungs.
o Neutrophilic myelocyte
o Eosinophilic myelocyte Proplatelets
o Basophilic myelocyte - branching process that exposes megakaryocytes to
Metamyelocyte/juvenile blood circulation and pinches of to produce platelets.
- specific granules occupy most of the cytoplasm. Demarcation membranes - invagination of
Predominant WBC in adult bone marrow plasma membrane in a megakaryocyte.
o Neutrophilic metamyelocyte
o Eosinophilic metamyelocyte
o Basophilic metamyelocyte
Band cell/ stab cell
- intermediate stage where nucleus is elongated but not
yet polymorphic.

AMUAN, BERJAMIN, ERMITANIO, LARIOSA, RESURRECCION – BSMLS2B 41


histology
LECTURE / INSTRUCTOR: JURA SORNILLO JUNQUEIRA’S BASIC HISTOLOGY TEXT AND ATLAS

CHAPTER 14: THE IMMUNE SYSTEM & LYMPHOID ORGANS


The immune system provides defense or immunity against
infectious agents.

Lymphocytes are formed initially in primary lymphoid organs


(the thymus and bone marrow).

Most lymphocyte activation and proliferation occur in


secondary lymphoid organs (the lymph nodes, the spleen,
tonsils, Peyer patches, and appendix)

The immune cells located diffusely in the digestive, respiratory,


or urogenital mucosae comprise what is collectively known as
mucosa-associated lymphoid tissue (MALT).

Lymph nodule, small, localized collection of lymphoid tissue,


usually located in the loose connective tissue beneath wet
epithelial (covering or lining) membranes.

INNATE IMMUNITY
The system of defenses termed innate immunity involves
immediate, nonspecific actions, including physical barriers
such as the skin and mucous membranes of the gastrointestinal,
respiratory, and urogenital tracts.
Cytokines coordinate cell activities in the innate and adaptive
Toll-like receptors (TLRs) on leukocytes allow the recognition
immune responses; diverse group of peptides and
and binding of surface components of such invaders.
glycoproteins, usually with low molecular masses (between 8
Natural killer (NK) cells destroy various unhealthy host cells. and 80 kDa) and a paracrine mode of action.

*Antimicrobial chemicals that form a major part of innate Chemotaxis is the directed migration of a cell in response to a
immunity: chemical stimulus, such as a growth factor.
o Hydrochloric acid (HCl) and organic acids in specific
Chemokines are signaling proteins secreted by cells of the
regions lower the pH locally to either kill entering
immune system that stimulate the movement of other cells.
microorganisms directly or inhibit their growth.
o Defensins, short cationic polypeptides produced by Interleukins are a group of cytokines that are able to modulate
neutrophils and various epithelial cells that kill bacteria by cellular behavior and play key roles in cell signaling.
disrupting the cell walls.
ANTIGENS AND ANTIBODIES
o Lysozyme, an enzyme made by neutrophils and cells of
A molecule that is recognized by cells of the adaptive immune
epithelial barriers, which hydrolyzes bacterial cell wall
system is called an antigen.
components, killing those cells.
o Complement, a system of proteins in blood plasma, Immune cells recognize and react to small molecular domains
mucus, and macrophages that react with bacterial surface of the antigen known as antigenic determinants or epitopes.
components to aid removal of bacteria.
o Interferons, paracrine factors from leukocytes and virus- The immune response to antigens may be cellular (in which
infected cells that signal NK cells to kill such cells and lymphocytes are primarily in charge of eliminating the antigen),
adjacent cells to resist viral infection. humoral (in which antibodies are primarily responsible for the
response), or both.
ADAPTIVE IMMUNITY
Adaptive immunity, acquired gradually by exposure to An antibody is a glycoprotein of the immunoglobulin family that
microorganisms, is more specific, slower to respond and involve interacts specifically with an antigenic determinant.
production of memory lymphocytes. Antibodies are secreted by plasma cells that arise by terminal
The adaptive immune response involves B and T lymphocytes. differentiation of clonally proliferating B lymphocytes whose
receptors recognize and bind specific epitopes.

AMUAN, BERJAMIN, ERMITANIO, LARIOSA, RESURRECCION – BSMLS2B 42


histology
LECTURE / INSTRUCTOR: JURA SORNILLO JUNQUEIRA’S BASIC HISTOLOGY TEXT AND ATLAS

Actions of Antibodies: *All T lymphocytes have cell surface protein receptors (TCRs)
o Complement activation promotes phagocytosis. with variable regions that recognize specific antigens. Cell
o Opsonization greatly increases the efficiency of activation requires costimulation by the TCR and either CD4 or
phagocytosis by these leukocytes at the sites of infection. CD8, which characterize helper and cytotoxic T cells,
o Primitive lymphocytes called NK cells, which are then respectively.
activated to kill the infected cell by releasing perforin and
various granzymes. B-cell receptors (BCRs) are immunoglobulin molecules
projecting from the plasmalemma
Antigen presentation is the expression of antigen molecules on
the surface of a macrophage or other antigen-presenting cell in T Lymphocytes:
association with MHC class II molecules when the antigen is o Helper T cells (CD4+) - part of the
being presented to a CD4 + helper T cell or in association with adaptive immune system, which has the ability to learn
MHC class I molecules when presentation is to CD8 + cytotoxic about, adjust against and remember different types of
T cells. infection.
o CTLs (CD8+) - are differentiated effector T lymphocytes that
Cells of Adaptive Immunity:
specifically kill target cells bearing an appropriate antigenic
o Antigen-Presenting Cells are an active endocytotic system
complex (peptide–MHC) recognized by their T cell receptor.
and expression of MHC class II molecules for presenting
o Regulatory T cells (CD4+CD25+) - T cells which have a role
peptides of exogenous antigens. Besides dendritic cells and
in regulating or suppressing other cells in the immune
all monocyte-derived cells, “professional” APCs also
system.
include thymic epithelial cells.
o Gamma delta T cells - have extraordinary properties
o Lymphocytes both regulate and carry out adaptive
including the capacity for tumor cell killing.
immunity.
B Lymphocytes:
* In classical secondary lymphoid organs (SLOs) such as lymph
o The B cell antigen receptors (BCRs) play an important role
nodes, tonsils, and Peyer’s patches, it is well established that
in the clonal selection of B cells and their differentiation into
fibroblastic reticular cells (FRCs) play an integral role in the
antibody-secreting plasma cells.
generation of immune responses.
o Follicular dendritic cells (FDCs) are found in the lymph
*CD Markers. CD is an abbreviation for “cluster of
node germinal follicles and have several different functions,
differentiation”. CD molecules are cell surface markers which
including activation of B cells and maintenance of
are very useful for the identification and characterization
immunologic memory.
of leukocytes and the different subpopulations of leukocytes.

AMUAN, BERJAMIN, ERMITANIO, LARIOSA, RESURRECCION – BSMLS2B 43


histology
LECTURE / INSTRUCTOR: JURA SORNILLO JUNQUEIRA’S BASIC HISTOLOGY TEXT AND ATLAS

Primary lymphoid nodule Thymic Medulla


- are clusters of lymphoid -fewer lymphocytes than the cortex
cells located as solitary -lymphocytes are more mature and don’t stain as dark
nodules in the lamina -abundant in TEC’s that forms Hassal’s corpuscles
propria of tubular organs -dendritic cells are also present but not easily identified when
and also as aggregates in stained with H&E
tonsils and Peyer’s
patches.
Secondary lymphoid
nodules
- consist of germinal
centers surrounded by a
perimeter, or mantle, of
tightly packed B
lymphocytes. The
euchromatic nuclei and
abundant cytoplasm of
the large, immature lymphoblasts located in the germinal
center contribute to the pale appearance of this region. This
nodule is located in the lamina propria of the small intestine.

THYMUS
-a flat encapsulated lymphoid organ
-located in the anterior superior mediastinum
-originates from the embryos third pharyngeal pouches
-most active during childhood
-largest around puberty
-after puberty it will begin to decrease in size (less lymphatic
tissue, more adipocytes)
-important in the maturation of t cells (negative selection or
central tolerance; help prevent autoimmunity)

Thymic Cortex
-T lymphoblasts (Thymocytes)- basophilic and supported by
network of finely branched epithelial reticular cells called thymic
epithelial cells (TEC’s)

TEC’s - pale cytoplasm, large oval nuclei & cytoplasmic


processes. Involvement in the formation of the blood thymus Hassal’s corpuscles
barrier. -unique to the thymus
-increase in size and number as individual get older
Blood thymus barrier - consist of TEC’s, pericytes and
endothelial cells. Selective barrier that separate the cortex from
the blood. The TEC’s help form BTB by joining together and
surround microvasculature.

AMUAN, BERJAMIN, ERMITANIO, LARIOSA, RESURRECCION – BSMLS2B 44


histology
LECTURE / INSTRUCTOR: JURA SORNILLO JUNQUEIRA’S BASIC HISTOLOGY TEXT AND ATLAS

LYMPH NODES SPLEEN


-small secondary lymphoid organ -Largest lymphoid organ
-found along lymphatic vessels throughout the body -Receives blood from the splenic artery
-encapsulated, bean-shaped structures -Only lymphoid organ that primarily filters blood
-filters the lymph in order to fight infections -Encapsulated organ

*Spleen’s functional
tissue (Parenchyma)
-red pulp, a combination
of pink and purple stain
using H&E
-white pulp, small
nodules of lymphatic
Germinal center is where B cells differentiate to plasma cells. tissue and stain dark
Medullary Sinuses drain into the efferent lymphatic vessels purple because of
basophilic nuclei

White pulp
- only 20% of the spleen
- is secondary lymphoid
tissue associated with
small central arterioles that
are also enclosed by PALS
of T cells.

Outer cortex of lymph Red pulp


node- follicle of B cells - which filters blood,
Inner Cortex of lymph removes defective
node- more erythrocytes, and recycles
hemoglobin iron, consists
eosinophilic(pink),
of splenic cords with
homogenous, consist
macrophages and blood
mostly of T cells and high cells of all kinds and
endothelial venules splenic sinusoid.
High endothelial venules - used to distinguish the region of The splenic sinusoids are lined by unusual endothelial cells
inner cortex from medulla. They are typically tall and high called stave cells, which are elongated and aligned parallel to
endothelial cells and allow lymphocytes in the venules to move the blood flow, with open slits between the cells.
out of the blood & into the surrounding lymphoid tissue.
Blood flow in red pulp is either a closed circulation, moving
MEDULLA - lighter compare to the inner cortex because it is not from capillaries into the venous sinusoids, or an open
densely packed with cells. This is because of the medullary circulation, with capillaries opening directly into the splenic
sinuses which form spaces where lymph collects before exiting cords.
the lymph node.
Medullary cords forms the shape of the medullary sinuses and Blood filtration in the open circulation involves interaction with
contain macrophages, b cells and plasma cells. splenic cord macrophages, which remove old, swollen RBCs
unable to slip between stave cells to reenter the venous blood
flow.

AMUAN, BERJAMIN, ERMITANIO, LARIOSA, RESURRECCION – BSMLS2B 45


histology
LECTURE / INSTRUCTOR: JURA SORNILLO JUNQUEIRA’S BASIC HISTOLOGY TEXT AND ATLAS

CHAPTER 15: DIGESTIVE TRACT - muscularis mucosae (thin layer of smooth muscle) –
which separates mucosa from submucosa and
allowing local movements of the mucosa.
2. submucosa
- denser connective tissue with larger blood and lymph
vessels
- submucosal (Meissner) plexus of autonomic nerves.
- It may also contain glands and significant lymphoid
tissue.
3. thick muscularis (muscularis externa) - composed of
smooth muscle cells organized as two or more sublayers
that has a connective tissue in between that contains blood
vessels, lymph vessels, and myenteric (Auerbach) nerve
plexus:
- internal sublayer (closer to the lumen) – circular fiber
orientation
- external sublayer – longitudinal fiber orientation
*Submucosal (Meissner) plexus and myenteric (Auerbach)
nerve plexus comprises the enteric nervous system of the
GI tract
*Contractions of the muscularis, which mix and propel the
luminal contents forward, are generated and coordinated by
Function: the myenteric plexus.
-to obtain molecules necessary for the maintenance, growth, 4. Serosa – a thin layer of loose connective tissue, rich in blood
and energy needs of the body from ingested food. vessels, lymphatics, and adipose tissue, with a simple
-protects the internal milieu of the body’s connective tissue and squamous covering epithelium or mesothelium.
vasculature from the contents that passed through the tract. *In the abdominal cavity, the serosa is continuous with
*Structures within the tract allows the following: mesenteries, thin membranes covered by mesothelium on
o Ingestion, or introduction of food and liquid into the oral both sides that support the intestines.
cavity Adventitia - a connective tissue layer that merges with the
o Mastication, or chewing, which divides solid food into surrounding tissues and lacks mesothelium.
digestible pieces IgA – produced by the macrophages and lymphocytes in the
o Motility, muscular movements of materials through the lamina propria located below of the epithelium lining. This IgA
tract complex resists proteolysis by digestive enzymes and provides
o Secretion of lubricating and protective mucus, digestive important protection against specific viral and bacteria
enzymes, acidic and alkaline fluids, and bile
o Hormone release for local control of motility and secretion
o Chemical digestion or enzymatic degradation of large
macromolecules in food to smaller molecules and their
subunits
o Absorption of the small molecules and water into the blood
and lymph
o Elimination of indigestible, unabsorbed components of
food
GENERAL STRUCTURES
The GI tract is a hollow tube with a lumen of variable diameter
and a wall made up of four main layers:
1. mucosa (mucous membrane)
- epithelial lining
- lamina propria of loose connective tissue (rich in blood
vessels, lymphatics, lymphocytes, smooth muscle
cells, and often containing small glands);

AMUAN, BERJAMIN, ERMITANIO, LARIOSA, RESURRECCION – BSMLS2B 46


histology
LECTURE / INSTRUCTOR: JURA SORNILLO JUNQUEIRA’S BASIC HISTOLOGY TEXT AND ATLAS

Oral cavity - lined with stratified squamous epithelium, which


may be keratinized, partially keratinized, or nonkeratinized
depending on the location.
*Layers:
1. Masticory mucosa – develops keratinized cell layers that
resists damage from the abrasion. The lamina propria in
these regions rests directly on the periosteum of underlying
bone.
2. Lining mucosa – where nonkeratinized squamous
epithelium can be found over the soft palate, cheeks, floor
of the mouth, and pharynx (throat), and the posterior region
of the oral cavity leading to the esophagus. It overlies a thick
submucosa containing many minor salivary glands, which
secrete continuously to keep the mucosal surface wet, and
diffuse lymphoid tissue.
3. Lips (labia) – its well-developed core of striated muscle
makes it highly mobile for ingestion, speech, and other
forms of communication.
*3 differently covered surfaces:
- internal mucous surface - has lining mucosa with a thick,
nonkeratinized epithelium and many minor labial salivary
glands. Lingual papillae - elevations of the mucous membrane that
- vermilion zone of each lip (red) - is covered by very thin assume various forms and functions.
keratinized stratified squamous epithelium and is *4 types:
transitional between the oral mucosa and skin. This region 1. Filiform papillae - are very
lacks salivary or sweat glands and is kept moist with saliva numerous, have an elongated
from the tongue. The underlying connective tissue is very conical shape, and are heavily
rich in both sensory innervation and capillaries, which keratinized, which gives their
impart the pink color to this region surface a gray or whitish
- outer surface - has thin skin, consisting of epidermal and appearance. They provide a
dermal layers, sweat glands, and many hair follicles with rough surface that facilitates
sebaceous glands. movement of food during
chewing.
Tongue
- mass of striated muscle covered by mucosa, which 2. Fungiform papillae - are much
manipulates ingested material during mastication and less numerous, lightly
swallowing keratinized, and interspersed
- the muscle fibers are oriented in all directions, allowing a high among the filiform papillae.
level of mobility. They are mushroom-shaped
- connective tissue between the small fascicles of muscle is with well-vascularized and
penetrated by the lamina propria, which makes the mucous innervated cores of lamina
membrane strongly adherent to the muscular core. propria

Sulcus terminalis - v-shaped groove that separates the papillary


layer (on the anterior two-thirds) and massed lingual tonsils (on
3. Foliate papillae - consist of
the posterior third)
several parallel ridges on each
side of the tongue, anterior to
the sulcus terminalis, but are
rudimentary in humans,
especially older individuals.

AMUAN, BERJAMIN, ERMITANIO, LARIOSA, RESURRECCION – BSMLS2B 47


histology
LECTURE / INSTRUCTOR: JURA SORNILLO JUNQUEIRA’S BASIC HISTOLOGY TEXT AND ATLAS

4. Vallate papillae (circumvallate)


- the largest papillae, with
diameters of 1-3 mm. Eight to
twelve vallate papillae are
normally aligned just in front of
the terminal sulcus

Taste buds - ovoid structures within


the stratified epithelium on the
tongue’s surface, which sample the general chemical
composition of ingested material
*Approximately 250 taste buds are present on the lateral surface
of each vallate papilla, with many others present on fungiform
and foliate papillae.
Cells in the tingue:
- Gustatory (taste) cells - turn over with a 7- to 10-day life
span. They make up about the 50% of the 50-100 cells
located in the tongue Primary teeth (deciduous or milk teeth) – there are 20 which
- Supportive cells can be shed and replaced by permanent teeth
- Immature cells Crown – part of the tooth that is exposed located abve the
- Stems cells – they are slowly dividing that give rise to other gingiva
cell types Neck – connects to the gum
Roots – one or more that fit firmly into the bony sockets of the
Taste pore – found at the apical ends of the gustatory cells, jaw
microvilli project toward a 2-μm-wide opening in the structure. Dental alveoli - bony sockets in the jaws
*Molecules (tastants) dissolved in saliva contact the microvilli Enamel - hardest component of the human body, consisting of
through the pore and interact with cell surface taste receptors. 96% calcium hydroxyapatite and only 2%-3% organic material
Taste buds detect atleast 5 broad categories: including very few proteins and no collagen.
- sodium ions (salty) Cementum – bone-like tissue that can be found in the roots that
- hydrogen ions from acids (sour)s meets enamel halfway in the neck to cover a tooth.
- sugars and related compounds (sweet) Dentin - calcified tissue harder than bone, consisting of 70%
- alkaloids and certain toxins (bitter) hydroxyapatite. The organic matrix contains type I collagen and
- amino acids such as glutamate and aspartate (umami; Jap. proteoglycans secreted from the apical ends of odontoblasts
Umami = savory) Dental pulp - highly vascular and well-innervated and consists
largely of loose, mesenchymal connective tissue with much
Teeth ground substance, thin collagen fibers, fibroblasts, and
*Adult human there are normally 32 permanent teeth, arranged mesenchymal stem cells
in two bilaterally symmetric arches in the maxillary and Root canal – pulp cavities
mandibular bones that narrow in each root
*Each quadrant consists of: that extends to the apical
- eight teeth foramen at the tip of each
- two incisors root for blood vessels,
- one canine lymphatics, and nerves of
- two premolars the pulp cavity.
- three permanent molars Periodontal ligaments -
fibrous connective tissue
bundles of collagen fibers
inserted into both the
cementum and the
alveolar bone.

AMUAN, BERJAMIN, ERMITANIO, LARIOSA, RESURRECCION – BSMLS2B 48


histology
LECTURE / INSTRUCTOR: JURA SORNILLO JUNQUEIRA’S BASIC HISTOLOGY TEXT AND ATLAS

Esophagus - muscular tube, about 25-cm long in adults, which *Major regions:
transports swallowed material from the pharynx to the stomach o cardia - is a narrow transitional zone, 1.5-3 cm wide,
*Layers: between the esophagus and the stomach
- esophageal mucosa has nonkeratinized stratified o pylorus - is the funnel-shaped region that opens into the
squamous epithelium small intestine. Both these regions are primarily involved
- submucosa contains small mucus-secreting glands, the with mucus production and are similar histogically
esophageal glands, which lubricate o The much larger fundus and body regions are identical in
- Near the stomach the mucosa also contains groups of microscopic structure and are the sites of gastric glands
glands, the esophageal cardiac glands, which secrete releasing acidic gastric juice.
additional mucus.
Rugae - large, longitudinally directed folds in the mucosa and
submucosa of the empty stomach which flatten when the
stomach fills with food. The wall in all regions of the stomach is
made up of all four major layers
*Four major layers:
1. Mucosa - Changing abruptly at the esophagogastric
junction, the mucosal surface of the stomach is a simple
columnar epithelium that invaginates deeply into the lamina
propria. The invaginations form millions of gastric pits, each
with an opening to the stomach lumen
2. Submucosa - is composed of connective tissue with large
blood and lymph vessels and many lymphoid cells,
macrophages, and mast cells.
3. Muscularis has three poorly defined layers of smooth
muscle: an outer longitudinal layer, a middle circular layer,
and an innermost oblique layer. Rhythmic contractions of
the muscularis thoroughly mix ingested food and chyme
with mucus, HCl, and digestive enzymes from the gastric
Swallowing - begins with mucosa.
voluntary muscle action 4. Serosa – covers the stomach
but finishes with
involuntary peristalsis Small intestine - site where the digestive processes are
completed and where the nutrients (products of digestion) are
Stomach - a greatly absorbed by cells of the epithelial lining. The small intestine is
dilated segment of the relatively long (5 m)
digestive tract *3 segments which have same characteristics and layers:
*Functions: - Duodenum
- To continue the - Jejunum
digestion of - Ileum
carbohydrates 1. Mucosa:
initiated by the Plicae circulares - a series of permanent circular or semilunar
amylase of saliva, folds that makes up mucosa and submucosa; best developed at
- To add an acidic fluid to the ingested food and mixing its the jejunum.
contents into a viscous mass called chyme by the churning Villi - densely covering the entire mucosa of the small intestine
activity of the muscularis are short (0.5-1.5 mm) mucosal outgrowths that are projected
- To begin digestion of triglycerides by a secreted lipase to the lumen.
- To promote the initial digestion of proteins with the enzyme - are covered by a simple columnar epithelium of absorptive
pepsin cells called enterocytes, with many interspersed goblet cells.
Intestinal glands or crypts (or crypts of Lieberkühn) – are the
openings of short tubular glands located between the villi.

AMUAN, BERJAMIN, ERMITANIO, LARIOSA, RESURRECCION – BSMLS2B 49


histology
LECTURE / INSTRUCTOR: JURA SORNILLO JUNQUEIRA’S BASIC HISTOLOGY TEXT AND ATLAS

2. Submucosa: *Function:
- has larger blood and lymph vessels and the diffuse, - converting food into feces Bacterial fermentation converts
interconnected neurons of the submucosal (Meissner) nerve the chyme into feces and releases vitamins including
plexus vitamins K, B1, B2, B6, B12, and biotin.
Duodenal (or Brunner) glands – located at The proximal part of - absorbing essential vitamins produced by gut bacteria
the duodenum has in the submucosa and mucosa large clusters - reclaiming water from feces. The absorption of water by
of branched tubular mucous glands the large intestine not only helps to condense and solidify
*Mucus from these glands is distinctly alkaline (pH 8.1-9.3), feces, but also allows the body to retain water to be used
which neutralizes chyme entering the duodenum from the in other metabolic processes.
pylorus, protecting the mucous membrane, and bringing the *Layers:
intestinal contents to the optimum pH for pancreatic enzyme Mucosa - innermost layer that lacks villi found in the small
action intestine. Many mucous glands secrete mucus into the hollow
3. Muscularis: lumen of the large intestine to lubricate its surface and protect
- well developed in the small intestine, composed of an internal it from rough food particles.
circular layer, an external longitudinal layer, and between them Submucosa – surrounds the mucosa is a layer of blood vessels,
the neurons of the myenteric (Auerbach) nerve plexus which nerves and connective tissue which supports the other layers of
produce peristalsis the large intestine.
Muscularis layer - surrounds the submucosa and contains
Large intestine - can be found in the abdominal cavity. About many layers of visceral muscle cells that contract and move the
4.9 feet (1.5 m) long. large intestine. Continuous contraction of smooth muscle bands
*Parts: in the muscularis produces lumpy, pouch-like structures known
1. Cecum - inferior region of the large intestine forms a short as haustra in the large intestine
dead-end segment Serosa - forms the outermost layer. It is a thin layer of simple
2. Vermiform appendix – end part of the cecum squamous epithelial tissue that secretes watery serous fluid to
3. ascending colon - a hollow tube that climbs along the right lubricate the surface of the large intestine, protecting it from
side of the abdomen friction between abdominal organs and the surrounding muscles
4. Hepatic flexure - inferior to the diaphragm, the ascending and bones of the lower torso.
colon turns about 90 degrees toward the middle
5. Transverse colon – located at the superior medial part of Anal canal
the large intestine *Layers:
6. Splenic flexure - 90 degrees angle where the transverse Internal anal sphincter - is a thin, white muscle wrapped around
colon turns at the left side of the abdomen the anal canal. The internal anal sphincter is an involuntary
7. descending colon – runs down from splenic flexure at the smooth muscle, like the muscles of your intestines. You cannot
left side of the abdomen mentally control it
8. Sigmoid flexure – a bend slightly medially to connect
External anal sphincter - is a thick, red voluntary muscle. It is
descending colon to the sigmoid colon
the one you squeeze when you feel the urge to go to the
9. Sigmoid colon – s-shaped in the large intestine
10. Rectum - final segment of the large intestine bathroom but are not near one. Since it is voluntary muscle, like
11. Anus - where the large intestine ends the muscles in your arms and legs, you can control it
Levator ani muscle - supporting and raising the pelvic visceral
----Lopez & Lauresta---- structures. It also helps in proper sexual functioning, defecation,
Other components: urination, and allowing various structures to pass through it
• Teniae coli - represent three narrowed, thickened, equally Anal columns of Morgagni -longitudinal folds just distal to
spaced bands of the outer longitudinal layer of the dentate line, analogous to lower rectums rectal columns of
muscularis externa. They are primarily visible in the cecum Morgagni. 6-10 longitudinal or vertical mucosal folds in the
and colon and they are absent in the rectum, anal canal, and upper part of the anal canal
vermiform appendix. Dentate (pectinate) line - separates the anal canal into an upper
• Haustra coli that are visible sacculations between the teniae and lower parts. The dentate line is formed by the anal columns,
coli on the external surface of the cecum and colon. which consists of a series of anal sinuses (which drain anal
• Omental appendices that are small fatty projections of the glands at approximately the midpoint of the anal canal. It is a
serosa, observed on the outer surface of the colon. "watershed area" and that the exact transition of epithelium and
neurovascular supply is varied.

AMUAN, BERJAMIN, ERMITANIO, LARIOSA, RESURRECCION – BSMLS2B 50


histology
LECTURE / INSTRUCTOR: JURA SORNILLO JUNQUEIRA’S BASIC HISTOLOGY TEXT AND ATLAS

CHAPTER 16: ORGANS ASSOCIATED WITH THE DIGESTIVE TRACT *Salivary secretory units are drained by simple cuboidal
intercalated ducts that merge as simple columnar striated ducts,
SALIVARY GLANDS which merge as larger interlobular or excretory ducts.
Salivary glands have secretory units of either protein-secreting *Three epithelial cell types comprise the salivary secretory units:
serous cells, usually organized in round or oval acini, or of Serous cells are polarized protein-secreting cells, usually
mucin-secreting mucous cells in elongated tubules. pyramidal in shape, with round nuclei, well-stained RER, and
*Three of large salivary glands: apical secretory granules; serous cells form a somewhat
Parotid glands have only spherical unit called an acinus, which secrete enzymes and
serous and watery acini proteins
Mucous cells contain apical granules with hydrophilic mucins
that provide lubricating properties in saliva but cause poor cell
staining in routine preparations
Myoepithelial cells move secretory products into and through
the ducts.
*Cells of striated ducts have mitochondria-lined, basolateral
membrane folds specialized for electrolyte reabsorption from
the secretion; excretory ducts are unusual in having stratified
cuboidal or columnar cells.
PANCREAS
-a mixed exocrine-endocrine gland that produces both digestive
Sublingual glands are enzymes and hormones.
mixed but have primarily
mucous tubules, some with
serous demilunes;

Pancreatic islets of endocrine cells are embedded in exocrine


Submandibular glands are serous acinar tissue, which comprises most of the pancreas and
also mixed but have mainly in which the cells secrete hydrolytic digestive enzymes for
serous acini. delivery to the duodenum.
*Each pancreatic acinar cell is pyramidal, with secretory
(zymogen) granules in the narrow apical end and Golgi
complexes, much rough ER, and a large nucleus at the basal
end.
*Intercalated ducts draining pancreatic acini, including their
initial centroacinar cells that insert into the acinar lumen, secrete
bicarbonate ions (HCO3−) to neutralize chyme entering the
duodenum from the stomach.
*Two-polypeptide hormones produced by enteroendorine cells
of small intestine:
--cholesystokinin (CCK) – stimulates enzyme secretion by the
*The secretion of each gland is either serous, seromucous, or acinar cells
mucous depending on its content of the glycoprotein mucin. --secretin – promotes water and HCO3− secretion by the duct
cells

AMUAN, BERJAMIN, ERMITANIO, LARIOSA, RESURRECCION – BSMLS2B 51


histology
LECTURE / INSTRUCTOR: JURA SORNILLO JUNQUEIRA’S BASIC HISTOLOGY TEXT AND ATLAS

LIVER *Two important cells found with the sinusoids of hepatic loblues:
Liver hepatocytes are large epithelial cells with large central - The sinusoidal endothelium includes many specialized stellate
nuclei (polyploid and often binucleated), much smooth and macrophages or Kupffer cells, which recognize and remove
rough ER, and many small Golgi complexes. effete erythrocytes, releasing iron and bilirubin for uptake by
*Main digestive function of the liver is the production of bile, a hepatocytes.
complex substance required for emulsification, hydrolysis, and - Also present in the perisinusoidal spaces are hepatic stellate
uptake of fats in the duodenum. cells (or Ito cells) containing many small lipid droplets for
*Hepatocytes have many functions: storage of vitamin A and other fat-soluble vitamins.
- endocrine (plasma protein synthesis and secretion),
- exocrine (bile secretion), *Between adherent hepatocytes in the hepatic plates are
- gluconeogenesis (conversion of amino acids into glucose) grooves called bile canaliculi, sealed by tight junctions, into
- glucose storage (glycogen granules), which hepatocytes secrete water and bile components,
- storage of vitamin A and iron, including bilirubin and bile acids
- detoxification (using SER and peroxisomes).
*In each hepatic lobule, all bile canaliculi converge on the bile
*In the liver, hepatocytes are organized into irregular plates to canals (canal of Hering), which join the bile ductules in the
form polygonal hepatic lobules in which the hepatocyte plates portal areas and eventually all merge to form the left and right
radiate toward a small central vein. hepatic ducts.

*Each hepatic lobule BILE TRACT AND GALLBLADDER


is surrounded by -a hollow, pear-shaped organ attached to the lower surface of
sparse connective the liver, capable of storing 30-50 mL of bile that is concentrated
tissue that is more during storage.
abundant in the portal -consists of a mucosa composed of simple columnar epithelium
areas at the corners. and lamina propria, a thin muscularis with bundles of muscle
fibers oriented in several directions, and an external adventitia
or serosa
Cholangiocytes are simple
cuboidal or columnar cells
where all bile-conducting
ducts line after the bile
canaliculi.
*Portal triad: *The common hepatic duct
--a portal venule leads to the cystic duct that
branch from the carries bile to the
portal vein, gallbladder for temporary
--a hepatic arteriole bile storage and
branch of the hepatic concentration.
artery, and *The mucosa of the
--a bile ductule gallbladder has many folds
branch of the biliary with a large surface area, a
tree. well-vascularized lamina
propria, and a lining of columnar cholangiocytes specialized for
*In the lobules the portal venule and hepatic arteriole both water uptake from bile.
branch into irregular sinusoids between the hepatic plates where *Contraction of the gallbladder muscularis sends bile to the
the nutrient-rich and O2-rich blood mixes, flows past duodenum via the common bile duct and is induced by
hepatocytes, and drains to the central vein. cholecystokinin (CCK) from enteroendocrine cells in the
*The endothelium of the hepatic sinusoids is discontinuous and duodenum when food is present.
fenestrated; between it and the hepatocytes is the
perisinusoidal space (space of Disse), where exchange occurs
between the hepatocytes and blood plasma.

AMUAN, BERJAMIN, ERMITANIO, LARIOSA, RESURRECCION – BSMLS2B 52


histology
LECTURE / INSTRUCTOR: JURA SORNILLO JUNQUEIRA’S BASIC HISTOLOGY TEXT AND ATLAS

CHAPTER 17: THE RESPIRATORY SYSTEM *The mucosa of the nasal cavities and nasopharynx also
The function of the respiratory system is to provide oxygen to contains a rich vasculature and many seromucous glands, which
the blood, with a secondary function of sound production in the help warm, humidify, and clean inspired air.
larynx. LARYNX, TRACHEA, AND BRIONCHIAL TREE
*The respiratory system consists of: Larynx – passage of air
air conducting region (the upper respiratory tract in the head, between pharynx and
as well as the larynx, trachea, bronchi, and most bronchioles); trachea, and protects
a respiratory region with alveoli, bronchioles, and alveolar ducts lower airways, facilitates
where the gas exchange occurs. respiration, and
NASAL CAVITY manipulates pith and
*The left and right nasal cavities have two components: volume.
--vestibule *Within the lumen of the
--nasal cavity larynx, bilateral
*The left and right nasal cavities of the upper respiratory tract projecting vocal folds
have vestibules where air enters and three projections called (or vocal cords) can be placed under variable tension by the
conchae from their medial walls, which create turbulence in underlying vocalis muscles and caused to vibrate by expelled
inspired air. air, producing sounds.
*Moist vibrissae in the vestibular openings, the nares or
nostrils, filter some material from inspired air. The trachea is completely lined
*Deeper areas of the vestibules and the floor, lateral walls, and by respiratory epithelium and is
most of the conchae of the nasal cavities themselves are lined supported by C-shaped rings of
by respiratory epithelium: pseudostratified ciliated columnar hyaline cartilage, with smooth
epithelium. trachealis muscles in the
posterior opening of the rings.
Respiratory epithelium includes:
-goblet cells secreting mucus,
-ciliated columnar cells sweeping
the mucus along the surface, -
chemosensory brush cells,
-scattered endocrine cells, and
-basal stem cells.

Left and right primary bronchi


enter the two lungs and
bifurcate repeatedly as
secondary, tertiary, and
The roof and part of the superior smaller segmental bronchi
concha in each nasal cavity are with the lung tissue as the
covered by olfactory epithelium, bronchial tree.
which is pseudostratified epithelium *Bronchi and their branches are lined by respiratory mucosa,
containing: with prominent spiraling bands of smooth muscle and
-bipolar olfactory neurons, increasingly smaller pieces of hyaline cartilage.
-support cells, and *Branches of the bronchial tree with diameters of 1 mm or less
-stem cells. are generally called bronchioles, which are lined by simple
columnar or cuboidal ciliated cells, with circular smooth muscle
but no cartilage.
*Terminal bronchioles are the last branches to lack alveoli and
are lined by simple cuboidal epithelium consisting mainly of club
cells, which have innate immune and surfactant secretory
functions.

AMUAN, BERJAMIN, ERMITANIO, LARIOSA, RESURRECCION – BSMLS2B 53


histology
LECTURE / INSTRUCTOR: JURA SORNILLO JUNQUEIRA’S BASIC HISTOLOGY TEXT AND ATLAS

LUNG VASCULATURE REVIEW:


 enterocytes – cells of simple columnar epithelium of absorptive cells that
Terminal bronchioles subdivide into two or three respiratory
line the villi of small intestine
bronchioles, lined by simple cuboidal epithelium and  mucosa-submucosa-muscularis-adventitia – layers of the esophagus
interrupted by scattered squamous evaginations called alveoli,  simple columnar epithelium with goblet cells – mucosa lining of the
the sites of gas exchange. stomach
 filiform papillae – lined with keratinized stratified squamous epithelium
 the muscularis of the stomach has inner oblique, middle circular, and
outer longitudinal layer
 myoepithelial cells – small flattened cells in the salivary gland that contract
to help move secretory products into the ducts
 the precursor enzyme pepsinogen to form pepsin is activated by the acid
environment of the stomach
 presence of Peyer’s patches distinguishes the ileum from the other
portions of the small intestine
 muscularis mucosae – are smooth muscle fibers found in the mucosal
layers of the digestive organs
 Paneth cells – cells of the stomach mucosae that produces hypochloric
acid and intrinsic factor
 Subligual gland – the smallest of the salivary glands comprising mostly of
tubuloacinar glands and few serous cells
 nonkeratinized stratified squamous epithelium – lining og the hard palate
and gingiva of the oral cavity
 cholecystokinin & secretin – hormones produced by the enteroendocrine
cells of the small intestine and which regulates the exocrine pancreas
 serosa is a unique from adventitia because it has a single outer covering
called mesothelium
A respiratory bronchiole leads to an alveolar duct, which is lined  cecum, appendix, colon, rectum – parts of the large intestine
by a continuous series of alveoli and which ends in a cluster of  vocalis muscle – allows the vocal fold to be moved for phonation
alveoli called the alveolar sac.  alveoli – site of gas exchange; distinguishes between terminal and
respiratory bronchioles
 nonkeratinized stratified squamous epithelium – vocal folds covering
All alveoli are surrounded by sparse connective tissue in  cilia – protects the respiratory tract and is absent in the digestive tract
interalveolar septa that consist primarily of elastic and reticular  club cells – cuboidal cells in the terminal bronchioles that has an exocrine
fibers and a dense capillary network. function producing substances such as surfactant
 terminal bronchioles have no hyaline cartilage
 ciliated columnar cells, goblet cells, brush cells, basal cells – cell of the
*The wall of each alveolus consists of alveolar cells, or respiratory epithelium
pneumocytes, of two types:  type I pneumocyte – cells of the alveoli that makes up 95% of the alveolar
extremely thin type I alveolar cells and lining and constitutes the blood-air barrier
 pulmonary surfactant – complex film of phospholipids that lowers surface
cuboidal type II alveolar cells with surfactant secreting and tension in the alveoli
innate immune properties.  alveolar ducts – organ of the respiratory portion of the respiratory system
 respiratory bronchioles – non part of the conducting portion of the
--Type II alveolar cells are characterized ultrastructurally by respiratory system
 dust cells – cells that phagocytose erythrocytes lost from damaged
unique cytoplasmic lamellar bodies, large granules with closely capillaries and debris in the alveoli
stacked layers of membrane involved in surfactant synthesis.  superior areas of the nasal cavities do not have the respiratory
--The blood-air barrier allowing gas exchange at each alveolus epithelium lining
consists of the thin type I alveolar cell, the thin capillary  pleural cavity – a narrow cavity between the parietal and visceral layers
endothelial cells, and the fused basal laminae of these two cells.

*The surfactant material secreted by exocrine club cells and type


II alveolar cells is an oily mixture of cholesterol, phospholipids
and surfactant proteins, which forms a film and lowers surface
tension in alveoli.

Each lung is covered by visceral pleura, a layer of thin


connective tissue and mesothelium, and is continuous with
parietal pleura, a similar tissue layer that lines the pleural cavity

AMUAN, BERJAMIN, ERMITANIO, LARIOSA, RESURRECCION – BSMLS2B 54


histology
LECTURE / INSTRUCTOR: JURA SORNILLO JUNQUEIRA’S BASIC HISTOLOGY TEXT AND ATLAS

CHAPTER 18: SKIN EPIDERMIS


Skin consists mainly of a superficial stratified squamous *Stratified squamous keratinized epithelium
epithelium, the epidermis, and a thicker layer of connective *Consists of keratinocytes that undergo a terminal differentiation
tissue, the dermis, which overlies a subcutaneous hypodermis. process called keratinization in a series of steps that form
distinct epidermal strata or layers:
o stratum basale is one layer of mitotically active cuboidal
cells attached by hemidesmosomes and integrins to the
basement membrane and to each other by desmosomes;
During differentiation, the cells move upward and the
amount and types of keratin filaments increase until they
represent half the total protein in the superficial
keratinocytes.

o stratum spinosum is the thickest layer and has several


layers of polyhedral cells attached to each other by
desmosomes at the tips of short projections containing
bundled keratin, or tonofibrils.
o stratum granulosum is a
thinner layer of
keratinocytes, now
flattened and filled densely
with keratohyaline
granules containing
filaggrin and other proteins
binding the tonofibrils; the
lamellar granules undergo
exocytosis, producing a
lipid-rich, impermeable layer around the cells.
FUNCTIONS OF SKIN: o stratum lucidum, found only in thick skin, consists of a thin,
1) Protection – provides a physical barrier against thermal and translucent layer of flattened eosinophilic keratinocytes
mechanical insults such as friction and against most
held together by desmosomes; Nuclei and organelles have
potential pathogens and other materials; Melanin in the
been lost, and the cytoplasm consists almost exclusively of
epidermis protects cell nuclei from UV radiation
2) Sensory – help regulate the body’s interactions with packed keratin filaments embedded in an electron-dense
physical objects. matrix
3) Thermoregulatory – maintaining skin’s insulating o stratum corneum protects against water loss, friction, and
components microbial invasion, and consists of flattened, terminally
4) Metabolic – synthesizes vit. D, needed calcium metabolism differentiated cells, or squames, which are slowly lost.
and bone formation.
5) Sexual signaling – pigmentation and hair as visual indicators * The epidermis-dermis interface is enlarged and strengthened
of health involves in attraction. by interdigitating epidermal ridges or pegs and dermal papillae
Dermatoglyphs “fingerprints” – forms distinct patterns unique in which microvasculature also supplies nutrients and O2 for the
for each individual, appearing as combination of loops, arches, epidermis.
and whorls.

AMUAN, BERJAMIN, ERMITANIO, LARIOSA, RESURRECCION – BSMLS2B 1


histology
LECTURE / INSTRUCTOR: JURA SORNILLO JUNQUEIRA’S BASIC HISTOLOGY TEXT AND ATLAS

Melanocytes DERMIS
“pigment producing” the layer of connective tissue that
- a specialized cell of supports the epidermis and binds
the epidermis found it to the subcutaneous tissue
among the cells of (hypodermis).
the basal layer and in *has two major layers:
hair follicles. -- a superficial papillary layer -
- have pale-staining, consists of loose connective
rounded cell bodies tissue, with types I and III collagen
attached by fibers, fibroblasts and scattered
hemidesmosomes to mast cells, dendritic cells, and
the basal lamina, but leukocytes
lacking attachments -- a thicker dense irregular
to the neighboring reticular layer - much thicker, consists of dense irregular
keratinocytes. connective tissue (mainly bundles of type I collagen), with more
*Melanosome Formation: fibers, providing elasticity to the skin
*Forms two major plexuses:
o subpapillary plexus, from which capillary branches extend
into the dermal papillae and form a rich, nutritive capillary
network just below the epidermis.
o deep plexus with larger blood and lymphatic vessels lies
near the interface of the dermis and the subcutaneous
layer.
Arteriovenous Anastomoses or shunts
- located between the two major plexuses
- decrease blood flow in the papillary layer to minimize heat loss
in cold conditions and increase this flow to facilitate heat loss
when it is hot, thus helping maintain a constant body
Langerhans Cells “antigen-presenting cells”
temperature.
- bind, process, and present antigens to T lymphocytes in the
SUBCUTANEOUS TISSUE
same manner as immune dendritic cells in other organs
“hypodermis or superficial fascia”
- form a network through the epidermis, intercepting and
- consists of loose connective tissue that binds the skin loosely
sampling microbial invaders before moving to lymph nodes in to the subjacent organs
an adaptive immune response. - contains adipocytes that vary in number in different body
regions and vary in size according to nutritional state. -
promotes rapid uptake of insulin or drugs injected into this
tissue.
SENSORY RECEPTORS
functions as an extensive receiver for various stimuli from the
environment.
*Unencapsulated Receptors:
Merkel Cells “epithelial tactile cells” o Merkel cells - function as tonic receptors for sustained light
- are sensitive mechanoreceptors touch and for sensing an object’s texture
essential for light touch sensation. o Free nerve endings - respond primarily to high and low
- abundant in highly sensitive skin temperatures, pain, and itching, but also function as tactile
like that of fingertips and at the receptors
bases of some hair follicles. o Root hair plexuses - web of sensory fibers surrounding the
- originate from the same stem cells
bases of hair follicles in the reticular dermis that detects
as keratinocytes and are characterized by small, Golgi-derived
movements of the hairs.
dense-core neurosecretory granules containing peptides.

AMUAN, BERJAMIN, ERMITANIO, LARIOSA, RESURRECCION – BSMLS2B 2


histology
LECTURE / INSTRUCTOR: JURA SORNILLO JUNQUEIRA’S BASIC HISTOLOGY TEXT AND ATLAS

*Encapsulated Receptors: Keratinocytes - divide rapidly in the region immediately around


the dermal papilla and then undergo keratinization, melanin
accumulation, and terminal differentiation.
Arrector pili muscle
- a small bundle of smooth muscle cells, extends from the
midpoint of the fibrous sheath to the dermal papillary layer
- pulls the hair shafts to a more erect position, usually when it
is cold in an effort to trap a layer of warm air near the skin
- produce tiny bumps on the skin surface (“goose bumps”)
NAILS
formed in a manner similar to
o Meissner corpuscles (a) - elliptical structures consisting of hairs: keratinocytes
sensory axons winding among flattened Schwann cells proliferate in the matrix of
arranged perpendicular to the epidermis in the dermal the nail root and differentiate
papillae, and detects light touch. with the formation of hard
o Lamellated (pacinian) corpuscles (b) - large oval keratin as a growing nail
structures, specialized for sensing coarse touch, pressure plate with edges covered by
(sustained touch), and vibrations, with distortion of the skin folds.
capsule amplifying a mechanical stimulus to the axonal core *The nail root forms from the
where an impulse is initiated nail matrix in which cells
o Krause end bulbs - simpler encapsulated, ovoid structures, divide, move distally, and
with extremely thin, collagenous capsules penetrated by a become keratinized in a process somewhat similar to hair
sensory fiber that sense low frequency vibrations. formation but without keratohyaline granules.
o Ruffini corpuscles - have collagenous, fusiform capsules hyponychium - distal end of the plate becomes free of the nail
anchored firmly to the surrounding connective tissue, with bed at the epidermal fold
sensory axons stimulated by stretch (tension) or twisting SKIN GLANDS
(torque) in the skin o Sebaceous glands produce sebum by terminal
HAIR differentiation of sebocytes, the classic example of
Hairs form in hair follicles, in which keratinocytes comprising holocrine secretion, secreting this oily substance onto hair
the matrix of the deep hair bulb proliferate rapidly and undergo in the follicles or pilosebaceous units.
keratinization to form the medulla, cortex, and cuticle of a hair
root.

o Eccrine sweat glands in the dermis produce sweat that is


mostly water onto the skin surface, where its evaporation
provides an important mechanism for cooling the body.
*Three major phases of hair:
o Anagen - a generally long period of mitotic activity and
growth
o Catagen - a brief period of arrested growth and regression
of the hair bulb
o Telogen - final long period of inactivity during which the
hair may be shed

AMUAN, BERJAMIN, ERMITANIO, LARIOSA, RESURRECCION – BSMLS2B 3


histology
LECTURE / INSTRUCTOR: JURA SORNILLO JUNQUEIRA’S BASIC HISTOLOGY TEXT AND ATLAS

– consists of stratified cuboidal epithelium with three cell types: CHAPTER 19: THE URINARY SYSTEM
a. Pale-staining clear cells - located on the basal lamina its primary role is to ensure optimal properties of the blood,
produce the sweat, having abundant mitochondria and which the kidneys continuously monitor.
microvilli to provide large surface areas *General role involves a complex combination of renal functions:
b. Dark cells - filled with strongly eosinophilic granules line o Regulation of the balance between water and electrolytes
most of the lumen which undergo merocrine secretion to (inorganic ions) and the acid-base balance
release a poorly understood mixture of glycoproteins o Excretion of metabolic wastes along with excess water and
with bactericidal activity. electrolytes in urine
c. Myoepithelial cells – located on the basal lamina Urine - the kidneys’ excretory product which passes through the
contract to move the watery secretion into the duct ureters for temporary storage in the bladder before its release
to the exterior by the urethra.
o Apocrine sweat o Excretion of many bioactive substances, including many
glands drugs
- consist of simple o Secretion of renin
cuboidal, eosinophilic Renin - a protease important for regulation of blood pressure by
cells with numerous cleaving circulating angiotensinogen to angiotensin I.
secretory granules o Secretion of erythropoietin
that also undergo Erythropoietin - a glycoprotein growth factor that stimulates
exocytosis, erythrocyte production in red marrow when the blood O2 level
- restricted to skin of is low.
the axillae and o Conversion of the steroid prohormone vitamin D
perineum, have much o Gluconeogenesis during starvation or periods of prolonged
wider lumens than fasting, making glucose from amino acids to supplement
eccrine glands, develop after puberty, and secrete protein- this process in the liver
rich sweat onto the hair of hair follicles. KIDNEYS
SKIN REPAIR approximately 12-cm long, 6-cm wide, and 2.5-cm thick in
*In the first phase blood from cut vessels coagulates in the adults
wound, releasing polypeptide growth factors and chemokines Hilum - a concave medial border where nerves enter, the ureter
from the disintegrating platelets. Neutrophils and macrophages exits, and blood and lymph vessels enter and exit
undergo diapedesis locally and remove bacteria and debris from Convex lateral surface - covered by a thin fibrous capsule
the wound. Renal pelvis - can be found within the hilum which expanded
Epithelialization begins as cells of the epidermal basal layer from the upper end of the ureter.
remove their desmosomes and hemidesmosomes and migrate *renal pelvis divides into two or three major calyces
laterally beneath the blood clot that becomes an increasingly *minor calyces are smaller branches that came from each major
desiccated eschar, or scab. calyces
*Growth of epidermal cells and fibroblasts is stimulated by Renal cortex - outer, darker stained region found in the
several different growth factors released from macrophages and parenchyma of each kidney that has many round corpuscles and
other cells and from their binding sites in ECM proteoglycans. tubule cross sections
*Proliferating fibroblasts and newly sprouted capillaries produce Renal medulla - consisting mostly of aligned linear tubules and
new collagen-rich, well-vascularized tissue in the dermis called ducts
granulation tissue which undergoes remodeling and a more Renal pyramids - 8 – 15 conical structures in the human renal
normal vasculature is reestablished. medulla, with their bases meeting the cortex (at the
corticomedullary junction) and separated from each other by
extensions of the cortex called renal columns
Renal lobe - each pyramid plus cortical tissue at the base
Renal papilla - tip of each pyramid that projects into a minor
calyx that collects urine formed by tubules in one renal lobe
*Kidneys each contain 1-4 million nephrons

AMUAN, BERJAMIN, ERMITANIO, LARIOSA, RESURRECCION – BSMLS2B 4


histology
LECTURE / INSTRUCTOR: JURA SORNILLO JUNQUEIRA’S BASIC HISTOLOGY TEXT AND ATLAS

Nephrons – functional units of the kidneys that each consists of o From the juxtaglomerular corpuscles near the medulla,
a corpuscle and a long, simple epithelial renal tubule with three efferent arterioles do not form peritubular capillaries, but
main parts along its length instead branch repeatedly to form parallel tassel-like
*Major divisions of nephrons: bundles of capillary loops called the vasa recta
o Renal corpuscle - an initial dilated part enclosing a tuft of
capillary loops and the site of blood filtration, always located Vasa recta
in the cortex (L. recta = straight)
o Proximal tubule - a long convoluted part, located entirely in - penetrate deep into the
the cortex, with a shorter straight part that enters the medulla in association
medulla with the loops of Henle
and collecting ducts
o Loop of henle (nephron loop) - in the medulla, with a thin
*The cortex receives
descending and a thin ascending limb
over 10 times more
o Distal tubule - consisting of a thick straight part ascending blood than the medulla.
from the loop of henle back into the cortex and a convoluted
part completely in the cortex
o Connecting tubule - a short minor part linking the nephron
to collecting ducts
*Collecting tubules (from several nephrons) -> collecting
tubules -> collecting ducts (lager)
Cortical nephrons - are located almost completely in the cortex RENAL FUNCTION: FILATRATION, SECRETION, AND
Juxtamedullary nephrons - lie close to the medulla and have REABSORPTION
long loops of Henle.
BLOOD CIRCULATION
the kidney vasculature is large, well-organized, and closely
associated with all components of the nephron
o Renal artery divides into two or more segmental arteries at
the hilum
o Around the renal pelvis, these arteries branch further as the
interlobar arteries
Interlobar arteries - extend between the renal pyramids toward
the corticomedullary junction
o Interlobar arteries divide again to form the arcuate arteries
that run in an arc along this junction at the base of each
renal pyramid
o Smaller interlobular arteries (or cortical radial arteries)
radiate from the arcuate arteries, extending deeply into the
cortex Renal Function:
o From the interlobular arteries arise the microvascular o Filtration - water and solutes in the blood leave the vascular
afferent arterioles, which divide to form a plexus of capillary space and enter the lumen of the nephron
loops called the glomerulus o Secretion - substances move from epithelial cells of the
Glomerulus - located within a renal corpuscle where the blood tubules into the lumens, usually after uptake from the
is filtered surrounding interstitium and capillaries
o Blood leaves the glomerular capillaries via efferent o Reabsorption - substances move from the tubular lumen
arterioles across the epithelium into the interstitium and surrounding
o Efferent arterioles, which at once branch again to form capillaries
another capillary network, usually the peritubular capillaries Renal corpuscle – can be found at the beginning of each
Peritubular capillaries - profusely distributed throughout the nephron. They are about 200 µm in diameter and containing a
cortex tuft of glomerular capillaries

AMUAN, BERJAMIN, ERMITANIO, LARIOSA, RESURRECCION – BSMLS2B 5


histology
LECTURE / INSTRUCTOR: JURA SORNILLO JUNQUEIRA’S BASIC HISTOLOGY TEXT AND ATLAS

Glomerular (bowman) capsule - a double-walled epithelial Parietal layer (outer) - forms the surface of the capsule. It
capsule that surrounds a renal corpuscle consists of a simple squamous epithelium supported externally
Visceral layer (inner) - envelops the glomerular capillaries, by a basal lamina.
which are finely fenestrated Capsular (or urinary) space – can be found between the visceral
and parietal layer which receives the fluid filtered through the
capillary wall and visceral layer
Vascular pole – it is where the afferent arteriole enters and the
efferent arteriole leave
Tubular pole – it is where the proximal convoluted tubule (pct)
begins, and where simple squamous epithelium is changed into
simple cuboidal epithelium that continues and forms the
proximal tubule
*Capillaries of each glomerulus have a total length of
approximately 1 cm and are uniquely situated between two
arterioles—afferent and efferent—the muscle of which allows
increased hydrostatic pressure in these vessels, favoring
movement of plasma across the glomerular filter
Podocytes - stellate epithelial cells that composes the visceral
layer. It has several primary processes that extend and curve
around the length of glomerular capillary.
Pedicels (L. pedicllus = little foot) – are parallel, interdigiting
secondary processes from each primary processes. They cover
much of the capillary surface, in direct contact with the basal
lamina

Glomerular filtration rate (GFR) - is constantly regulated by


neural and hormonal inputs affecting the degree of constriction
in each of these arterioles. The total glomerular filtration area of
an adult has been estimated at 500 cm2 and the average GFR at
125 ml/min or 180 l/d
*The total amount of circulating plasma averages 3 l, it follows
that the kidneys typically filter the entire blood volume 60 times
Slit diaphragms – are elongated spaces or filtration slit pores every day.
that are about 25 to 30 nm wide that can be found between the Mesangial cells (Gr. mesos = in the midst + angion = vessel)
pedicels. They are modified and specialized occluding or tight - resemble vascular pericytes in having contractile properties
junctions composed of nephrins, other proteins, glycoproteins, and producing components of an external lamina.
and proteoglycans important for renal function *Mesangial cells are difficult to distinguish from podocytes, but
*Podocytes and capillary endothelial cells compose the often stain more darkly
apparatus for renal filtration Mesangium - surrounding matrix which fills interstices between
*The basement membrane between highly fenestrated capillaries that lack podocytes
endothelial cells and the podocytes is thick (300-360nm). This Functions:
separates the blood from the capsular space and forms by o Physical support of capillaries within the glomerulus
fusion of the capillary- and podocyte-produced basal laminae o Adjusted contractions in response to blood pressure
thus becoming the most substantial part of the filtration barrier. changes, which help maintain an optimal filtration rate

AMUAN, BERJAMIN, ERMITANIO, LARIOSA, RESURRECCION – BSMLS2B 6


histology
LECTURE / INSTRUCTOR: JURA SORNILLO JUNQUEIRA’S BASIC HISTOLOGY TEXT AND ATLAS

Three layers:
o single layer of small basal cells
on a very thin basement
membrane
o intermediate region that has one
to several layers of cuboidal or
low columnar cells
o superficial layer of large
bulbous or elliptical umbrella
cells, sometimes binucleated,
which are highly differentiated to
protect the underlying cells
against the potentially cytotoxic
o Phagocytosis of protein aggregates adhering to the effects of hypertonic urine
glomerular filter, including antibody-antigen complexes *Muscularis of the ureters
abundant in many pathological conditions moves urine toward the
o Secretion of several cytokines, prostaglandins, and other bladder by peristaltic
factors important for immune defense and repair in the contractions and produces
glomerulus prominent mucosal folds
URETERS, BLADDER, AND URETHRA when the lumen is empty
Ureters – transports urine from the renal pelvis to the urinary *Muscularis is thicker than
bladder. The walls of the ureters are similar to that of the calyces mucosa
and renal pelvis, with mucosal, muscular, and adventitial layers
and becoming gradually thicker closer to the bladder
Stratified Urothelium (transitional epithelium) – lines the
mucosa of the ureter.

AMUAN, BERJAMIN, ERMITANIO, LARIOSA, RESURRECCION – BSMLS2B 7


histology
LECTURE / INSTRUCTOR: JURA SORNILLO JUNQUEIRA’S BASIC HISTOLOGY TEXT AND ATLAS

Bladder: o spongy urethra - about 15 cm in length, is enclosed within


Umbrella cells - are especially well developed in the bladder erectile tissue of the penis and is lined by stratified
where contact with urine is greatest. These cells, up to 100 µm columnar and pseudostratified columnar epithelium, with
in diameter, have extensive intercellular junctional complexes stratified squamous epithelium distally
surrounding unique apical membranes. Female urethra - a 3- to 5-cm-long tube, lined initially with
Asymmetric unit membrane – can be found in the apical surface transitional epithelium which then transitions to nonkeratinized
of umbrella cells in which regions of the outer lipid layer appear stratified squamous epithelium continuous with that of the skin
ultrastructurally to be twice as thick as the inner leaflet. at the labia minora.
Uroplakins - assemble into paracrystalline arrays of stiffened *Middle part of the urethra in both sexes is surrounded by the
plaques 16 nm in diameter that causes the apical surface to look external striated muscle sphincter.
thick
*abundant membranous plaques, together with the tight CHAPTER 20: ENDOCRINE GLANDS
junctions, allow this epithelium to serve as an osmotic barrier *Endocrine cells produce hormones
protecting its cells and the cells of surrounding tissues from o Paracrine secretion - localized dispersal in interstitial fluid
hypertonic urine and preventing dilution of the stored urine. or through short loops of blood vessels, as when gastrin
made by pyloric G cells reaches target cells in the fundic
glands
o Juxtacrine secretion - a signaling molecule remains on the
secreting cell’s surface or adjacent extracellular matrix and
affects target cells when the cells make contact. Juxtacrine
signaling is particularly important in embryonic and
regenerative tissue interactions
o Autocrine secretion - cells may produce molecules that act
on themselves or on cells of the same type
PITUITARY GLAND OR HYPOPHYSIS
lies below the brain in a small cavity on the sphenoid bone
*Consists of two glands:
o Neurohypophysis -
retains many
histologic features of
Urethra - a tube that
brain tissue and
carries the urine
consists of a large
from the bladder to
part, the pars
the exterior.
nervosa, and the
*Urethral mucosa
smaller infundibulum stalk attached to the hypothalamus at
has prominent
the median eminence
longitudinal folds,
o Adenohypophysis - derived from the oral ectoderm, has
giving it a distinctive
three parts:
appearance in cross
 Pars Distalis - its main components are cords of well-
section
stained endocrine cells interspersed with fenestrated
Male urethra – it is longer than females and its two ducts for
capillaries and supporting reticular connective tissue.
sperm transport joins at the prostate gland.
 Pars Tuberalis - a smaller funnel-shaped region
*Three segments:
surrounding the infundibulum of the neurohypophysis;
o prostatic urethra - 3-4 cm long, extends through the
most of the cells of the pars tuberalis are
prostate gland and is lined by urothelium.
gonadotrophs.
o membranous urethra - a short segment, passes through an
 Pars Intermedia - contains basophils (corticotrophs),
external sphincter of striated muscle and is lined by
chromophobes, and small, colloid-filled cysts derived
stratified columnar and pseudostratified columnar
from the lumen of the embryonic hypophyseal pouch
epithelium

AMUAN, BERJAMIN, ERMITANIO, LARIOSA, RESURRECCION – BSMLS2B 8


histology
LECTURE / INSTRUCTOR: JURA SORNILLO JUNQUEIRA’S BASIC HISTOLOGY TEXT AND ATLAS

*Blood vessels of the hypothalamic hypophyseal portal system


are important in carrying peptide factors from hypothalamic
neurons to cells of the anterior pituitary where they control cell
secretion
*Most general staining methods
simply allow the parenchymal cells of
the pars distalis to be subdivided into
acidophil cells (A), basophils (B), and
chromophobes (C) in which the
cytoplasm is poorly stained. Capillaries *The pars intermedia (PI) is a
and sinusoids (S) in the second narrow region lying between
capillary plexus of the portal system. the pars distalis (PD) and the
pars nervosa (PN), with many
of its basophils (B) often
invading the latter

AMUAN, BERJAMIN, ERMITANIO, LARIOSA, RESURRECCION – BSMLS2B 9


histology
LECTURE / INSTRUCTOR: JURA SORNILLO JUNQUEIRA’S BASIC HISTOLOGY TEXT AND ATLAS

ADRENAL GLANDS
The adrenal (or suprarenal) glands are paired flattened
structures with a half-moon shape organs lying near the
superior poles of the kidneys, embedded in the pararenal
adipose tissue and fascia.
*Covered by a dense connective tissue capsule that sends thin
trabeculae into the gland’s parenchyma.
*The stroma consists mainly of reticular fibers supporting the
secretory cells and microvasculature
*Two concentric regions:
o Adrenal Cortex o Adrenal Medulla
- have characteristic features of steroid-secreting cells: - composed of large, pale-staining polyhedral cells arranged
acidophilic cytoplasm rich in lipid droplets, with central in cords or clumps and supported by a reticular fiber
nuclei network
- their cytoplasm shows an exceptionally profuse smooth - chromaffin cells
ER (SER) of interconnected tubules, which contain the -- are medullary parenchymal cells considered
enzymes for cholesterol synthesis and conversion of the modified sympathetic postganglionic neurons, lacking
steroid prohormone pregnenolone into specific active axons and dendrites and specialized as secretory cells
steroid hormones -- secrete either epinephrine or norepinephrine.
- has three zones: *Norepinephrine-secreting
 zona glomerulosa cells are also found in
- consists of closely packed, rounded or arched cords paraganglia (collections of
of columnar or pyramidal cells with many capillaries catecholamine-secreting cells
- mineralocorticoids, steroids that uptake f Na+, K+, adjacent to the autonomic
and water by cells of renal tubules - aldosterone, the ganglia) and in various viscera
major regulator of salt balance, which acts to stimulate Epinephrine - increases heart
Na+ reabsorption in the distal convoluted tubules, and rate, dilates bronchioles, and
stimulated by angiotensin II dilates arteries of cardiac and
 zona fasciculata skeletal muscle.
- consists of long cords of large polyhedral cells, one Norepinephrine - constricts
or two cells thick, separated by fenestrated sinusoidal vessels of the digestive system and skin, increasing blood flow
capillaries to the heart, muscles, and brain.
- polyhedral cells secrete glucocorticoids, especially PANCREATIC ISLETS
cortisol Pancreatic islets (islets of
- cortisol, affect carbohydrate metabolism by Langerhans) are compact
stimulating gluconeogenesis in many cells and spherical or ovoid masses of
glycogen synthesis in the liver, and suppresses many endocrine cells embedded
immune functions and can induce fat mobilization and within the acinar exocrine
muscle proteolysis--- stimulated by ACTH tissue of the pancreas.
 zona reticularis *Major islet cells:
- consists of smaller cells in a network of irregular o α or A cells secrete primarily
cords interspersed with wide capillaries glucagon and are usually located
- cells are usually more heavily stained than those of peripherally.
the other zones because they contain fewer lipid o β or B cells produce insulin (L.
droplets and more lipofuscin pigment insula, island), are the most
- produce cortisol but primarily secrete the weak numerous, and are located centrally.
androgens, including dehydroepiandrosterone (DHEA) o δ or D cells, secreting somatostatin,
that is converted to testosterone in both men and are scattered and much less
women--- stimulated by ACTH abundant.

AMUAN, BERJAMIN, ERMITANIO, LARIOSA, RESURRECCION – BSMLS2B 10


histology
LECTURE / INSTRUCTOR: JURA SORNILLO JUNQUEIRA’S BASIC HISTOLOGY TEXT AND ATLAS

Diffuse neuroendocrine system (DNES) - produce many of the C cells (parafollicular cell)
same polypeptides and neurotransmitter-like molecules, such - have a smaller amount of
as serotonin (5-hydroxytryptamine), also released by rough ER, large Golgi
neurosecretory cells in the CNS. complexes, and numerous
Enterochromaffin - cells of the DNES that are stained by small (100-180 nm in
solutions of chromium salts diameter) granules
Argentaffin cells - those stained with silver nitrate containing calcitonin
APUD cells - DNES cells secreting serotonin or certain other - secretion of calcitonin is triggered by elevated blood Ca2+
amine derivatives demonstrate amine precursor uptake and levels and it inhibits osteoclast activity
decarboxylation Production of Thyroid Hormone & Its Control
THYROID GLAND 1. Production of thyroglobulin
- located anterior and inferior to the larynx, consists of two lobes 2. Uptake of iodide from blood by na/i symporters (nis) in the
united by an isthmus thyrocytes’ basolateral cell membranes
- the only endocrine gland in which a large quantity of secretory 3. Iodination of tyrosyl residues in thyroglobulin
product is stored 4. Formation of t3 and t4
- synthesizes the thyroid hormones thyroxine (tetra- 5. Endocytosis of iodinated thyroglobulin by the thyrocytes
iodothyronine or T4) and tri-iodothyronine (T3), which help 6. Secretion of t4 and t3 at the basolateral domains of
control the basal metabolic rate in cells throughout the body, as thyrocytes
well as the polypeptide hormone calcitonin. *Thyroid hormones increase the number and size of
Thyroglobulin - a glycoprotein, and a precursor for the active mitochondria and stimulate mitochondrial protein synthesis,
thyroid hormones. helping to enhance metabolic activity.
Thyrocytes (follicular cells) *Thyroid hormones inhibit the release of TSH, maintaining levels
- squamous to low columnar of circulating T4 and T3 within the normal range.
in size, and controlled by TSH (thyrotropin)
thyroid-stimulating hormone - major regulator of the anatomic and functional state of thyroid
(TSH) from the anterior follicles
pituitary. - increases cell height in the follicular epithelium and stimulates
- exhibit organelles indicating all stages of thyroid hormone production and release
active protein synthesis and - secretion is increased by exposure to cold and decreased by
secretion, as well as heat and stressful stimuli.
phagocytosis and digestion.

AMUAN, BERJAMIN, ERMITANIO, LARIOSA, RESURRECCION – BSMLS2B 11


histology
LECTURE / INSTRUCTOR: JURA SORNILLO JUNQUEIRA’S BASIC HISTOLOGY TEXT AND ATLAS

PARATHYROID GLANDS *The resulting diurnal fluctuation


- four small ovoid masses in blood melatonin levels
e located on the back of induces rhythmic changes in the
the thyroid gland, usually activity of the hypothalamus,
embedded in the larger pituitary gland, and other
gland’s capsule. endocrine tissues that
Principal (chief) cells characterize the circadian (24
- endocrine cells of the hours, day/ night) rhythm of
parathyroid glands physiological functions and
Parathyroid hormone (PTH) - regulator of blood calcium levels behaviors.
targeting the osteoblasts, the distal convoluted tubules of the *Pineal landmarks are the
renal cortex, and the small intestines. concretions called corpora
PINEAL GLAND arenacea (brain sand)
- known as the epiphysis cerebri *Neural connections from the retina to pinealocytes allow
- a small, pine cone-shaped organ diurnal secretion of melatonin and rhythm in physiological
- regulates the daily rhythms of bodily activities activities.
- acts as a neuroendocrine transducer, converting sensory input
regarding light and darkness into variations in many hormonal
functions.
Pinealocytes - produce melatonin

AMUAN, BERJAMIN, ERMITANIO, LARIOSA, RESURRECCION – BSMLS2B 12


histology
LECTURE / INSTRUCTOR: JURA SORNILLO JUNQUEIRA’S BASIC HISTOLOGY TEXT AND ATLAS

CHAPTER 21: MALE REPRODUCTIVE SYSTEM *Testosterone secretion by


Testosterone is important for spermatogenesis, sexual interstitial cells is triggered
differentiation during embryonic and fetal development, and by the pituitary
control of gonadotropin secretion in the pituitary gonadotropin, interstitial
cell-stimulating hormone
(ICSH)
Seminiferous Tubules
- sperm are produced in the
seminiferous tubules at a
rate of about 2 × 108 per
day in the young adult
- each tubule is actually a loop linked by a very short, narrower
segment, the straight tubule, to the rete testis
- lined with stratified epithelium called germinal or
spermatogenic epithelium
- basement membrane of this epithelium is covered by fibrous
connective tissue, with an innermost layer containing flattened,
smooth muscle-like myoid cells
- efferent ductules connect the rete testis to the head of the
TESTES epididymis
- surrounded by a dense connective tissue capsule, the tunica *Germinal epithelium consists of two types of cells:
albuginea, which thickens on the posterior side to form the o Large nondividing Sertoli cells which physically and
mediastinum testis metabolically support developing sperm cell precursors
Testicular lobules o Dividing cells of the spermatogenic lineage - comprising
- pyramidal compartments which is subdivided by septa four or more concentric layers of cells in the germinal
- contains sparse connective tissue with endocrine interstitial epithelium, develop from progenitor cells to fully formed
cells (Leydig cells) sperm cells over a period of approximately 10 weeks
Seminiferous tubules Spermatogeneis
-sperm production occurs. - begins at puberty with
Tunica vaginalis proliferation of stem and
- consists of an outer parietal layer lining the scrotum and an progenitor cells called
inner visceral layer, covering the tunica albuginea on the anterior spermatogonia
and lateral sides of the testis Spermatogonia
Pampiniform venous plexus - occupy a basal niche in the
- surrounds testicular artery epithelial wall of the tubules,
- containing cooler blood from the testis, which draws heat from next to the basement
the arterial blood by a countercurrent heat-exchange system membrane and closely
Interstitial Tissue associated with Sertoli cell
- between the seminiferous tubules consists of sparse surfaces
connective tissue containing fibroblasts, lymphatics, and blood - Spermatogonia with dark,
vessels including fenestrated capillaries ovoid nuclei act as stem
Interstitial cells or Leydig cells cells
- during puberty, develop as polygonal cells with central nuclei o type A spermatogonia each undergo several unique clonal
and eosinophilic cytoplasm rich in small lipid droplets divisions that leave most of the cells interconnected as a
- produce the steroid hormone testosterone, which promotes syncytium
development of the secondary male sex characteristics o type B spermatogonia, comes after the type A
spermatogonia which have more spherical and pale nuclei

AMUAN, BERJAMIN, ERMITANIO, LARIOSA, RESURRECCION – BSMLS2B 13


histology
LECTURE / INSTRUCTOR: JURA SORNILLO JUNQUEIRA’S BASIC HISTOLOGY TEXT AND ATLAS

*Each of these cells then undergoes a final mitotic division to Sertoli cells
produce two cells that grow in size and become primary - tall “columnar” epithelial
spermatocytes cells, which nourish the
Primary spermatocytes spermatogenic cells and
- spherical cells with euchromatic nuclei divide the seminiferous
- has 46 (44 + XY) chromosomes, the diploid number, and a tubules into two (basal
DNA content of 4N. and adluminal)
Secondary spermatocytes compartments
- Homologous chromosomes separate in the first meiotic - adhere to the basal
division, which produces smaller cells lamina and their apical
- with only 23 chromosomes (22 + X or 22 + Y), but each still ends extend to the lumen
consists of two chromatids so the amount of DNA is 2N - contain abundant SER, some rough ER, well developed Golgi
- rare in testis sections; very short-lived cells that remain in complexes, numerous mitochondria, and lysosomes
interphase only briefly and quickly undergo the second meiotic - nuclei are typically ovoid or triangular, euchromatic, and have
division a prominent nucleolus, features that allow Sertoli cells to be
Spermatids distinguished from the neighboring germ cells
- division of each secondary spermatocyte separates the - form a blood-testis barrier within the seminiferous epithelium
chromatids of each chromosome and produces two haploid *Three general functions:
cells o Support, protection, and nutrition of the developing
- contains 23 chromosomes spermatogenic cells
- DNA per cell is reduced by half (1N) o Exocrine and endocrine secretion:
- haploid spermatids differentiate into sperm - androgen-binding protein (ABP) - concentrates
testosterone to a level required for spermiogenesis;
promoted by FSH
- inhibin - feeds back on the anterior pituitary gland to
suppress FSH synthesis and release
- müllerian-inhibiting substance (MIS) - causes
regression of the embryonic müllerian ducts; in the
absence of MIS these ducts persist and become parts
of the female reproductive tract
o Phagocytosis
INTRATESTICULAR DUCTS
- straight tubules (or tubuli recti), the rete testis, and the
efferent ductules, all of which carry spermatozoa and liquid
from the seminiferous tubules to the duct of the epididymis
Seminiferous Tubules(S), Straight Tubules(T), Rete Testis(R)

Spermiogenesis
- final phase of sperm production
- spermatids differentiate into spermatozoa (which are highly
specialized to deliver male DNA to the ovum)
*Divided into four phases:
o Golgi phase
o Cap phase
o Acrosome phase
o Maturation phase (not yet functional)

AMUAN, BERJAMIN, ERMITANIO, LARIOSA, RESURRECCION – BSMLS2B 14


histology
LECTURE / INSTRUCTOR: JURA SORNILLO JUNQUEIRA’S BASIC HISTOLOGY TEXT AND ATLAS

EXCRETORY GENITAL DUCTS Principal cells


- epididymis, the ductus (or vas) deferens, and the urethra - remove most of the water and residual bodies entering the
- transport sperm from the scrotum to the penis during epididymis with the sperm and secrete various products, greatly
ejaculation changing the fluid in which sperm are suspended
Epididymis *Changes within spermatozoa while passing through the
- long, highly coiled duct of epididymis include:
the epididymis, surrounded o development of competence for forward motility
by connective tissue, lies in o final modifications within the acrosome
the scrotum along the o reorganization of the cell membrane surrounding the sperm
superior and posterior head, including the addition of cholesterol and other
sides of each testis “decapacitation factors”, which block the acrosomal
- includes a head region reaction, a key event in fertilization
where the efferent ductules Ductus or Vas Deferens
enter, a body where sperm - long straight tube with a
cells undergo further subtle thick, muscular wall and a
modifications, and a tail relatively small lumen,
where sperm are stored leaves the scrotum and
until ejaculation continues toward the
Epididymal duct prostatic urethra where it
- lined with pseudostratified columnar epithelium consisting of empties
columnar principal cells, with characteristic long stereocilia - its mucosa is slightly
- surrounded by a thin, circular layer of smooth muscle cells, folded longitudinally
supplemented in the tail region with the addition of inner and - the lamina propria
outer longitudinal layers contains many elastic
fibers

AMUAN, BERJAMIN, ERMITANIO, LARIOSA, RESURRECCION – BSMLS2B 15


histology
LECTURE / INSTRUCTOR: JURA SORNILLO JUNQUEIRA’S BASIC HISTOLOGY TEXT AND ATLAS

- epithelial lining is pseudostratified with some cells having *Fluid from seminal vesicles typically makes up about 70% of
sparse stereocilia the ejaculate and its components include the following:
- muscularis consists of longitudinal inner and outer layers and o Fructose, a major energy source for sperm, as well as
a middle circular layer inositol, citrate, and other metabolites
- muscles produce strong peristaltic contractions during o Prostaglandins, which stimulate activity in the female
ejaculation, which rapidly move sperm along this duct from the reproductive tract
epididymis o Fibrinogen, which allows semen to coagulate after
- forms part of the spermatic cord, which also includes the ejaculation
testicular artery, the pampiniform plexus, and nerves Prostate gland
Accessory glands - a dense organ surrounding the urethra below the bladder
- produce secretions, which become mixed with sperm during - consists of a collection of 30-50 tubuloacinar glands
ejaculation to produce semen and are essential for reproduction embedded in a dense fibromuscular stroma in which smooth
- seminal vesicles (or glands), the prostate gland, and the muscle contracts at ejaculation
bulbourethral glands *Tubuloacinar glands are lined by simple or pseudostratified
columnar epithelium
*One clinically important product of the prostate is prostate-
specific antigen (PSA) which helps liquefy coagulated semen
for the slow release of sperm after ejaculation (elevated levels
of circulating PSA indicate abnormal glandular mucosa typically
due to prostatic carcinoma or inflammation)
- corpora amylacea, small spherical concretions present in the
lumens of many prostatic tubuloacinar glands containing
primarily deposited glycoproteins and keratan sulfate, may
become more numerous with age
- like the seminal vesicles, the prostate’s structure and function
depend on the level of testosterone.
*Arranged in three major zones around the urethra:
o transition zone occupies only about 5% of the prostate
volume, surrounds the superior portion of the urethra, and
contains the periurethral mucosal glands
o central zone comprises 25% of the gland’s tissue and
contains the periurethral submucosal glands with longer
ducts
o peripheral zone, with about 70% of the organ’s tissue,
contains the prostate’s main glands with still longer ducts
Bulbourethral glands (or Cowper glands)
- 3-5 mm in diameter, are located in the urogenital diaphragm
and empty into the proximal part of the penile urethra
-testosterone-dependent
-release a clear mucus-like secretion, which coats and lubricates
the urethra in preparation for the imminent passage of sperm
during erection
Seminal Vesicles
*Each gland has several lobules with tubuloacinar secretory
- folds are lined with simple or pseudostratified columnar
units surrounded by smooth muscle cells and lined by a mucus-
epithelial cells rich in secretory granules
secreting simple columnar epithelium
- lamina propria contains elastic fibers and is surrounded by
PENIS
smooth muscle with inner circular and outer longitudinal layers, *Penile urethra is lined with pseudostratified columnar
which empty the gland during ejaculation. epithelium

AMUAN, BERJAMIN, ERMITANIO, LARIOSA, RESURRECCION – BSMLS2B 16


histology
LECTURE / INSTRUCTOR: JURA SORNILLO JUNQUEIRA’S BASIC HISTOLOGY TEXT AND ATLAS

- In the glans, it becomes CHAPTER 22: FEMALE REPRODUCTIVE SYSTEM


stratified squamous Paired ovaries – produces the secondary oocyte and hormones
epithelium continuous with (progesterone, estrogen, inhibin and relaxin)
that of the thin epidermis Paired Uterine (Fallopian) tubes or oviducts – transports
covering the glans surface secondary oocytes and fertilized ovum to the uterus
- In uncircumcised men the Uterus - where embryonic
glans is covered by the Vagina – connects the uterus and cervix to the outside of the
prepuce or foreskin body.
*The corpora cavernosa are - this system produces the female gametes (oocytes), provides
each surrounded by a dense the environment for fertilization, and holds the embryo during
fibroelastic layer, the tunica its complete development through the fetal stage until birth.
albuginea OVARIES
*Three erectile tissues consist of many venous cavernous - each ovary is covered by a simple cuboidal epithelium, the
spaces lined with endothelium surface (or germinal) epithelium, continuous with the
*Central arteries in the corpora cavernosa branch to form mesothelium and overlying a layer of dense connective tissue
nutritive arterioles and small coiling helicine arteries capsule, the tunica albuginea like that of the testis.
*Penile erection involves blood filling the cavernous spaces in *Ovaries have a cortex and medulla – but the distinct is not
the three masses of erectile tissue always readily apparent.
o Erection is controlled by autonomic nerves in these o Medulla – vascular core and a region that contains loose
vascular walls connective tissue and blood vessels.
o Parasympathetic stimulation relaxes the trabecular smooth o Cortex – ovarian follicles in various stages of development
muscle and dilates the helicine arteries, allowing increased surrounded by connective tissue composed of fibroblasts
blood flow and filling the cavernous spaces and mesenchymal cells.
o This enlarges the corpora cavernosa and causes them to Ovarian Follicle – the fundamental functional unit of the ovary
compress the dorsal veins against the dense tunica - Ovarian follicles are made up of an oocyte surrounded by
albuginea, which blocks the venous outflow and produces follicular epithelial cells that play an essential role in
tumescence and rigidity in the erectile tissue promoting the survival and development of the oocyte.
o Beginning at ejaculation, sympathetic stimulation constricts - The follicular cells start out as a single layer, but proliferate
the helicine arteries and trabecular muscle, decreasing to form a multifunctional glandular epithelium cells the
blood flow into the spaces, lowering the pressure there, and granulosa layer.
allowing the veins to drain most blood from the erectile - Adjacent mesenchymal cells separated from the follicular
tissue epithelium by a basement membrane differentiate to form
another key element of the follicle,
o the theca folliculi made up to two layers
o the glandular theca interna
o and a supporting vascular layer, the theca externa.
Primordial Follicle
- Oocyte is immature (arrested in 1st meiotic prophase) and
is surrounded by a single layer of flattened follicular
epithelial cells.
- Primordial follicles tend to be
located toward the outer cortex
*Consists of three cylindrical masses of erectile tissue: of the ovary.
o corpora cavernosa - two of the erectile masses; dorsal - These follicles are formed
o corpus spongiosum - surrounds the urethra; ventral; At its during fetal life
end expands, forming the glans *Primary follicle includes
o glans unilaminar and multi-laminar
primary follicles

AMUAN, BERJAMIN, ERMITANIO, LARIOSA, RESURRECCION – BSMLS2B 17


histology
LECTURE / INSTRUCTOR: JURA SORNILLO JUNQUEIRA’S BASIC HISTOLOGY TEXT AND ATLAS

Unilaminar 1O Follicle *Follicular fluids contain high concentration of progesterone,


- Follicle-Stimulating hormone (FSH) is released during and androstenedione, and estrogens
puberty which promotes the development of the primordial *The follicular epithelium proliferates to form the stratified
follicle. follicular epithelium also termed as granulosa layer
- primordial follicle undergo mitosis and form a simple Granulosa cells are interconnected by gap junctions,
cuboidal epithelium around the growing oocyte. This follicle suggesting synchrony in function.
is now termed as the unilaminar primary follicle. - generate follicular fluid that accumulates to form the
follicular antrum.
- produce glycosaminoglycans (hyaluronic acid) and
proteoglycans (versican) creating an osmotic gradient
driving fluid movement from the vasculature of the theca
externa to the antrum.
- Creation of the antrum involves cellular remodeling within
the granulosa layer.
Multilaminar Secondary Follicle
- follicular cells here are now Pre-Ovulatory Follicle (Graafian or Mature Follicle)
termed Granulosa cells
- between the oocyte and the first
layer of granulosa cells, an
extracellular material accumulates
as the Zona pellucida
- stroma cells immediately outside
each growing primary follicle
differentiate to form the follicular
*As the antrum develops, the granulosa around the oocyte form
theca
a small hillock, the cumulus oophorus.
*Those granulosa cells that surrounds the zona pellucida make
Theca interna up the corona radiata, this part accompanies the oocyte when
- an endocrine tissue that leaving the ovary.
secretes Androstenedione *Cytokines released by the oocyte induce functional
which is converted by heterogeneity in the granulosa cells.
aromatase to estradiol (E2) *Oocyte completes first meiotic division, casts first polar body,
- differentiated and a critical enters meiosis II and arrests at metaphase (to continue
cooperation develops between following fertilization)
theca interna cells and *A mature follicle has thick thecal layers and normally develops
granulosa cells. from a primordial follicle over a period of about 90 days.
o LH stimulates production of androgens by theca interna Corona radiata and stalk cells - increase production of
cells hyaluronic acid while mural cells remain responsive to LH in
o FSH stimulates conversion of androgens to estrogens by preparation for their post-ovulation role in the corpus luteum.
granulosa cells, and the estrogens in turn stimulate Granulosa cells - produce a factor (oocyte maturation factor
granulosa cells to proliferate that inhibits the secondary oocyte from premature continuation
o de novo synthesis of estrogens by theca interna cells is of meiosis.
minimal.
Theca externa - more fibrous, compared to the theca interna, Ovulation
with fibroblasts and smooth muscle. - a surge in LH levels is essential for ovulation to take place and
*As the primary follicle grow, spaces appear between the must occur after both the theca interna cells and granulosa cells
granulosa layer as the cells secrete follicular fluid (liquor have expressed LH receptors, or the follicle will fall and undergo
folliculi). atresia.

AMUAN, BERJAMIN, ERMITANIO, LARIOSA, RESURRECCION – BSMLS2B 18


histology
LECTURE / INSTRUCTOR: JURA SORNILLO JUNQUEIRA’S BASIC HISTOLOGY TEXT AND ATLAS

Granulosa cells - secrete factors that promote weakening of the Three Phases of Ovarian Cycle
ovarian stroma to permit expansion of the growing follicle, with o Follicular Stage
greater activity nearing ovulation. - development of primordial follicle  mature follicle
Plasminogen activator - converts circulating plasminogen to - follicular phase of endometrium
plasmin (fribrinolysin), a trypsin-like protease that weakens the o Ovulatory Stage
surrounding connective tissue. - release of oocyte from mature follicle and capture by the
*Increased collagenase activity aided by local ischemia weakens oviducts
the connective tissue capsule at the point of contact with the o Luteal Phase
follicle. - residual follicular cells folds and becomes part of the
*Oocyte surrounded by corona radiata cells breaks free from the corpus luteum
follicle wall and is released to the surface of the ovary. - secretion/luteal phase of endometrium
UTERINE TUBE
Corpus Luteum - also called as fallopian tube
- produces the hormone progesterone in addition to small - supported by ligaments
amounts of estrogen and mesenteries
*After ovulation, the granulosa cells and theca interna of the - each tube opens into the
ovulated follicle reorganize to form a larger temporary endocrine peritoneal cavity near the
gland, the corpus luteum. ovary, with regions in the
*Under the influence of LH, cells of both granulosa cells and following sequence.
theca interna change histologically and functionally.
*The granulosa cells increase greatly in size and now called
granulosa lutein cells. These cells expand their role in
aromatase conversion of androstenedione into estradiol
*The former theca interna forms the rest of the corpus luteum
and is now called theca lutein cells.
Luteolysis - degeneration of the corpus luteum.
*The wall of the oviduct is made up of three layers:
o Mucosa
- highly folded lamina propria forms convoluted
channels lined by a simple columnar epithelium having
two cell types
- estrogen drives ciliogenesis and stimulates ciliary
activity, while progesterone has an opposite effect.
- ciliated cells have long, very active cilia that beat
towards the uterus
- peg cells secrete nutrients to support the oocyte and
factors that promote sperm capacitation
- lamina propia is a highly vascular loose connective
tissue that contains considerable smooth muscle,
Corpus Albicans particularly in the fimbriae. Contraction of these cells
*Ovarian follicles can undergo and movement of the fimbriae (and mucosal folds) is
degeneration at any stage of very active at ovulation.
development and often times o Muscularis
retain eosinophilic remnants of - made up of two (or more) interwoven layers of smooth
zona pellucida and basement. muscle, an inner circular/spiral layer and an outer
*The largest collagenous scars longitudinal layer; peristaltic contraction of the
in the ovary (corpus albicans) muscularis is directed toward the uterus
are derived from corpus lutea. o Serosa - lined by mesothelium

AMUAN, BERJAMIN, ERMITANIO, LARIOSA, RESURRECCION – BSMLS2B 19


histology
LECTURE / INSTRUCTOR: JURA SORNILLO JUNQUEIRA’S BASIC HISTOLOGY TEXT AND ATLAS

- the structural and functional changes involved are driven by


a variety of cytokines and ovarian hormones, with estrogen
and progesterone playing a major role.
- the endometrium (uterine mucosa) is composed of
uterine glands surrounded by a vascular lamina propria
(endometrial stroma)
- uterine glands are tubular type, commonly branched and
are lined by simple columnar epithelium.

Phases of the Menstrual Cycle:


o Menstrual phase
- The functionalis is shed, but the base of uterine glands
UTERUS is retained in the stratum basalis.
composed of two main portions, the body or corpus which is o Proliferative phase
lined by a mucosa (endometrium) capable of supporting - The torn endometrium is resurfaced by outgrowth of
implantation of a conceptus, and the cervix a specialized epithelial cells from the base of glands, accompanied
mucous gland that projects into the vagina. by proliferation of cells in the surrounding stroma.
*Layers of the Uterine wall: - The uterine glands lengthen and coil and the epithelial
cells accumulate glycogen
- The spiral (coiled) arteries grow toward the
functionalis.
o Secretory phase
- As the endometrium thickens the stroma becomes
edematous
- The uterine glands become swollen and tortuous, and
the epithelium releases glycoprotein-rich products into
the lumen.
- The spiral arteries elongate
o Menstrual Phase
Uterine Vasculature
- When pregnancy does not occur and the corpus
- the pattern of arterial supply to the endometrium is key to
luteum degenerates, estrogen and progesterone levels
the concept of cyclical renewal of the uterine mucosa fall triggering a series of events ultimately targeting the
(menstruation and the uterine cycle)
vasculature
- accurate arteries are located within the myometrium
- Prostaglandins released in the stroma trigger
- radial arteries originate in the myometrium and enter the
vasoconstriction of the spiral arteries.
basalis of the endometrium
- Endometrial stroma loses fluid, lymphocytes and
- straight (basal) arteries supply the endometrial basalis macrophages invade the stroma
- spiralis (coiled0 arteries extends from the basalis to the
- The spiral arteries open and close intermittently
functionalis of the endometrium
Ischemia  hypoxia  cell lysis  hemorrhage to
- these are heavily muscular vessels that grow during the
debride the functionalis
uterine cycle and play a key role in inducing ischemia at
menstruation.
CERVIX
Histological Changes During the Uterine Cycle - the lower, cylindrical part of the uterus
- a substantial portion of the endometrium (functionalis - differs histologically from the uterus
layer) is shed at menstruation leaving behind a layer - the mucosa of the narrow, inferior end of the uterus, the
(basalis of the endometrium) that undergoes resurfacing, cervix, does not change cyclically under hormone influence;
growth and maturation in preparation for implantation. at its external is columnar epithelium changes abruptly
stratified squamous.

AMUAN, BERJAMIN, ERMITANIO, LARIOSA, RESURRECCION – BSMLS2B 20


histology
LECTURE / INSTRUCTOR: JURA SORNILLO JUNQUEIRA’S BASIC HISTOLOGY TEXT AND ATLAS

o Endocervical Mucosa – lined with simple columnar EXTERNAL GENETALIA


epithelium - all covered by stratified squamous epithelium
o Exocervical Mucosa – lined with stratified non-keratinized o vestibule, a space whose wall includes the tubuloacinar
epithelium vestibular glands
o labia minora, folds of skin lacking hair follicles but with
numerous sebaceous glands
o labia majora, homologous and histologically similar to the
skin of the scrotum
o clitoris, an erectile structure homologous to the penis with
paired corpora cavernosa.
MAMMARY GLAND
- a compound tubuloalveolar gland consisting 15-25 separate
lobes having a lacteriferous duct that leads to a dilated
PLACENTA
lactiferous sinus at the nipple.
- site of exchange for nutrients, wastes, oxygen and carbon
- organs of milk production
dioxide gases between the mother and the fetus.
- consist of 15-25 lobes of
- also an endocrine organ, producing Hcg
compound tubuloalveolar
gland
- dense CT and adipose
tissue separates the lobes
- each lobe is a separate
gland with its own excretory
lactiferous duct
- these ducts emerge
independently in the nipple
*Histologic structure varies
according to: sex, age,
physiologic status
VAGINA o Mammary duct system
- the mucosa of the - lactiferous ducts are lined by a stratified cuboidal
vagina is lined by epithelium
stratified squamous - smooth muscle forms a sphincter guarding the
epithelium, surrounded opening of each lactiferous sinus at the nipple
by a muscularis. o Secretory units of the breast
o Mucosa - secretory elements called mammary alveoli form as
– lined with stratified outgrowths from small ducts that branch from the
non-keratinized lactiferous ducts and are composed of clusters of
squamous epithelium cuboidal cells (mammary gland epithelium)
– epithelial cells synthesize and accumulates glycogen responsible for milk production.
which is stimulated by the estrogen - myoepithelial cells are found closely associated with
– lamina propria is rich in elastic fibers mammary gland epithelium and play a role in
– has no glands; the mucus in the vagina is produced by stimulating the release of milk products into the lumen
the cervical gland and greater vestibular glands of the mammary gland.
o Muscularis – made up of smooth muscle o Mechanisms of secretion in the release of milk products
o Adventitia – made up of dense fibroelastic connective - the constituents of breast milk are released by several
tissue and contains extensive venous plexus, lymphatics, secretory mechanisms.
and nerves - proteins are packaged in membrane bound secretory
granules and released by regulated merocrine
secretion.
AMUAN, BERJAMIN, ERMITANIO, LARIOSA, RESURRECCION – BSMLS2B 21
histology
LECTURE / INSTRUCTOR: JURA SORNILLO JUNQUEIRA’S BASIC HISTOLOGY TEXT AND ATLAS

- lactose is released by constitutive exocytosis; the


major carbohydrate and energy source in milk
- lipids are released enclosed by a rim of cytoplasm,
apocrine secretion
- secretory IgA synthesized by adjacent plasma cells is
transported to the lumen via vesicular transcytosis
- in addition to these main constituents’ breast milk
contains factors (lysozyme, mucins, latoferrin) that
help regulate the intestinal flora of the infant
o Hormonal regulation of breast structure and function
- at puberty, estrogen stimulates development of
lactiferous ducts
- alveolar buds form and regress with each ovarian cycle
- in pregnancy, multiple hormones stimulate substantial
development of the lactiferous ducts and mammary
alveoli.
- during lactation, milk secretion is stimulated by
prolactin.
o Regulation of milk let-down during nursing
- release of milk from the breast involves feedback at the
level of the hypothalamus and pituitary.
- suckling stimulates the release of oxytocin which in
turn stimulates the myoepithelial cells associated with
secretory cells of mammary alveoli to contract
promoting secretion.
- afferent signals to the CNS trigger suppression of
release of prolactin-inhibitory hormone from the
hypothalamus, leading to an increase in prolactin levels
stimulating lactation.
- multiple neuroendocrine factors work to trigger
relaxation of the smooth muscle sphincter between
the lactiferous ducts and lactiferous sinuses.

AMUAN, BERJAMIN, ERMITANIO, LARIOSA, RESURRECCION – BSMLS2B 22


Module 6:
hïstøløgÿ Laboratory Notes
Reference: HISTO-GUIDE

SKIN Stratum Basale


• single layer of germinal cells resting on the
Covers the outer surface of basement membrane which is attached to the
\ the body and is the largest dermis.
organ. Skin and its
accessory structures (hair,
sweat glands, sebaceous
glands, and nails) make up
the integumentary
s y s t e m .

primary functions: Stratum Spinosum


• protect the body from the environment and;
• keratinocytes attached to each other by
• prevent water loss. desmosomes on spiny processes.

• Thick skin - covers the palms of the hands and the


soles of the feet. The outer keratin layer is
substantially thicker than in other parts of the body
and is lightly pigmented.
• Thin skin - covers the rest of the body and is
heavily pigmented.
Stratum Granulosum
• keratinocytes with numerous basophilic,
keratohyalin granules in their cytoplasm.

Stratum Lucidum
• highly refractive zone only seen in very thick skin.

Stratum Corneum
• thick layer of dead cells (squames) devoid of
EPIDERMIS nuclei and organelles.
• outer layer of stratified squamous keratinized
epithelium that is further divided into divided
into five strata (or layers).

dññçørñëlïø
DERMIS
• underlying layer of dense, irregular connective
tissue that contains other structures (such as
hair follicles and sweat glands)
Papillary Layer
• papillae that project into the dermis.
• connective tissue firmly attached to the
epidermis by the basement. membrane HYPODERMIS
• loose connective tissue with adipose tissue.

MELANOCYTES
Dermal Papillae
• producing cells located in the stratum basale.
• increase adhesion between the epidermis
• Paradoxically, melanocytes are unpigmented
and dermis.
because they do not store melanin.
• Melanin is transferred to other keratinocytes in
membrane-bound organelles (melanosomes).

Dermal Capillaries
• bring nutrients to the epidermis.

HAIR FOLLICLE & SEBACEOUS GLAND


• Common in thin skin. The sebaceceus glands are
large cells with a central nuclei and foamy cytoplasm.
These cells produce an oily, waxy substance called
Reticular Layer
sebum that is released onto the surface of the skin.
• dense irregular connective.
• less organized connective tissue that supports
the papillary and the epidermis

Eccrine Sweat Glands


• coiled tubular gland with simple or stratified
cuboidal epithelium (lightly stained) and duct
cells (dark staining)..
dññçørñëlïø
HAIR FOLLICLES
Glassy Membrane
• thick basement membrane that separates the
hair follicle from the dermis.

Hair Shaft
• cells grow from the hair bulb, die and lose their
cellular detail. The cortex is composed of keratinized
cells with melanin, while the medulla contains Hair Follicle
vacuolated cells. • hair shafts (which are absent in most
• Cuticle - squamous cells form the outermost layer follicles) are found at the center of hair
of hair. follicles. These structures are described from
the center outwards.

Internal Root Sheat


• only extends from the hair bulb to the level of
sebaceous glands. The cellular detail also becomes MECHANORECEPTORS
less defined. • a mechanoreceptor is a sensory receptor that
• Huxley's Layer - single or double layer of flattened responds to pressure or vibration. Two types are
cells. easily seen in skin from fingers or lips.
• Henle's Layer - outer single layer of cuboidal cells.
Meissner Corpuscles
• nerve endings in skin responsible for sensitivity to light
touch.
• Located in dermal papillae.
• Each corpuscle consists of an unmyelinated axon
meandering between Schwann cells arranged as
horizontal lamellae surrounded by a connective tissue
capsule.

Internal Root Sheat


• layers of cells continuous with the epidermis.

dññçørñëlïø
Pacinian Corpuscles
• nerve endings in skin responsible for sensitivity to
vibration and pressure.
• Large oval or spherical structures of 20 to 60
concentric lamellae located in the dense irregular
connective tissue (dermis) underneath the epithelium.
• Each corpuscle contains an inner bulb of an
unmyelinated axon within a fluid-filled cavity formed by
several lamellae of Schwann cells. Most of the
corpuscle is concentric lamellae separated by fluid.
The flattened cells that form these are fibroblasts
similar to those located in the endoneurium outside the
capsule.

dññçørñëlïø
Module 7:
hïstøløgÿ Lecture Notes
Reference: HISTO-GUIDE

CORTICAL LABYRINTHS
Urinary System is composed of • regions between renal corpuscles and medullary rays
the kidneys, ureters, urinary that contain proximal and distal convoluted tubules.
bladder, and urethra. Its main
function is the production,
storage, and expulsion of urine.

kidney MEDULLARY RAYS


Kidneys have several functions: • projections of tubules between the cortex and medulla
1. Excretion - elimination of water-soluble metabolic that contains straight tubules and collecting ducts.
wastes and foreign substances as urine
2. Regulation - maintain an appropriate fluid volume
and concentrations of various electrolytes in the
body fluids, maintain normal blood pressure, and
maintain the pH of blood
3. Endocrine - secretion of hormones
• Renin - regulation of blood pressure
• Erythropoietin - stimulates production of red
blood cells
• Vitamin D - regulation of calcium levels MEDULLA
• Lighter inner region
• Kidneys filter blood and produce urine. Unlike the
• Pyramids - equal to the number of lobes and form
human kidney which is multilobed (10 to 12 lobes)
conical structures whose base faces the cortex and
separated by renal columns (cortical tissue that
their apex form the renal papilla. Urine passes through
extends alongside the margin of pyramids in the
the minor calyx which is cup-shaped structure that is
medulla), the monkey kidney is unilobular.
an extension of the renal pelvis.
CORTEX
• darker outer region

RENAL PELVIS
• funnel-shaped origin of the ureter.
RENAL CORPUSCLE
• spherical structures that form ultrafiltrate from blood. Renal papilla

dññçørñëlïø
ARCUATE ARTERY Parietal Layer - simple squamous epithelium that lines
• branches of interlobular arteries that form an arcade the outer wall of the capsule.
over the pyramids at the junction of the cortex and Visceral Layer - podocytes cover the glomerular
medulla. capillaries. These cells have large ovoid nuclei.
Bowman's Space - the space between the parietal and
visceral layers that receives the ultrafiltrate.

GLOMERULUS
• branches of interlobular arteries that form an arcade
over the pyramids at the junction of the cortex and
medulla.

HILIUM
• concave surface with a deep fissure in which vessels
enter and exit the kidney.

Glomerular Capillaries - supplied by an afferent


arteriole and drained by an efferent arteriole.
Podocytes - cover the surface of the glomerular
capillaries and form narrow (~25 nm) filtration slits.
NEPHRON These cells have large ovoid nuclei.
• The nephron is the functional unit of the kidney. Each Mesangial Cells - large cells with irregularly shaped
nephron includes a filter (renal corpuscle), and a nuclei that have phagocytic and contractile function.
single, long tubule (renal tubule) through which the
filtrate passes before emerging as urine
RENAL CORPUSCLE VASCULAR POLE
• spherical structures with an average diameter of 200 • where the afferent and efferent arterioles enter and
µm distributed throughout the cortex. exit the glomerulus.

BOWMAN’S CAPSULE URINARY POLE


• encloses the glomerulus • where the ultrafiltrate exits Bowman's space and a
proximal convoluted tubule begins.

dññçørñëlïø
CONNECTIVE TISSUES • Urine is unchanged after it leaves the kidney.
• Connective tissue surrounds nephrons, blood vessels FETAL KIDNEY
and lymphatics. There is less connective in the cortex • During the development of the kidney, additional renal
compared to the medulla. corpuscles form in the outer cortex as the kidney
CAPSULE grows. Those found deeper in the inner cortex are
more mature
• thin layer of dense irregular connective tissue.
CORTEX
• Darker outer of each lobule

CORTEX
• connective tissue supports nephrons, blood vessels
and lymphatics. Macrophages and fibroblasts are Renal Corpuscles - consist of Bowman's capsule and a
prevalent. glomerulus.
Mature Corpuscles - in the inner cortex.

MEDULLA
Developing Corpuscles - in the outer cortex in various stages
• connective tissue supports ducts and tubules forming
of completion.
the renal interstitium. The most prevalent connective
tissue cell type is myofibroblasts.

Cortical Labyrinth - region between renal corpuscles and


medullary rays.
BLOOD SUPPLY PROXIMAL CONVOLUTED TUBULE
• each kidney is supplied by a renal artery. • their eosinophilic cross-sections are abundant near
renal corpuscles. They have a simple cuboidal to
Interlobar Arteries - branches of the renal artery within the columnar epithelium with an irregular, often stellate
renal sinus that enter the renal columns. lumen.
Arcuate Arteries - branches of interlobar arteries that arch
over the base of pyramids at the junction of the cortex and
medulla.

Interlobular Arteries - branches of arcuate arteries that


transverse the cortex to provide an afferent arteriole for each
renal corpuscle.
dññçørñëlïø
Peritubular Capillaries - destination of efferent arterioles of
each renal corpuscle depends on its location.
Adventitia - loose connective tissue with blood
DISTAL CONVOLUTED TUBULE vessels, nerves and adipose cells.
• their cross-sections have a simple cuboidal epithelium
with a uniform lumen and indistinct boundaries
between cells.

URETER
• Ureter transport urine from the kidney to the bladder.
It is lined with an epithelium that is impermeable to
water and ions. Peristaltic contraction of the smooth
muscle moves urine from the kidney to the bladder.
Like the bladder, it is lined by transitional epithelium
(urothelium). URINARY BLADDER
Transitional Epithelium (Urothelium) - consists of two to • The urinary bladder is a muscular sac that stores
urine, allowing urination to be infrequent and
three cell layers in the upper ureter with up to ten cell layers
voluntary. It is lined by transitional epithelium
near the bladder.
(urothelium), and has a thick layer of smooth muscle.
Bladder is an expandable vessel for the storage of
urine. It is lined with an epithelium that is impermeable
to water and ions.

LIKE THE URETERS, THE BLADDER IS COMPOSED


OF FOUR CONCENTRIC LAYERS.
Umbrella Cells - upper layer of cells that change shape
depending on the distention of the ureter (relaxed) Transitional Epithelium (Urothelium) - consists of three to five
cell layers.

Lamina Propria - thick layer of dense irregular connective


tissue rich in collagen and elastic fibers Umbrella Cells - the upper layer of cells that change shape
depending on the distention of the bladder.

Muscularis Externa - irregular arrangement of smooth


muscle in two layers (inner longitudinal and outer circular) in Lamina Propria - thick layer of dense irregular connective
the upper ureter or three layers (inner longitudinal, middle tissue rich in collagen and elastic fibers.
circular and outer longitudinal) near the bladder

dññçørñëlïø
Muscularis Externa - irregular arranged smooth muscle that URETHRA
forms an inner longitudinal, middle circular and outer • The urethra is a thin, fibromuscular tube that begins at
longitudinal layers. the lower opening of the bladder and extends through
the pelvic and urogenital diaphragms to the outside of
the body, called the external urethral orifice. The
urethra also connects the to the ductus deferens in
males, for the ejaculation of sperm.

Outer Layer of Connective Tissue - most of the bladder is


covered externally by adventitia with parts of its superior
surface covered by serosa of the peritoneum.

• Adventitia - loose connective tissue with blood


vessels, nerves and adipose cells.

MALE URETHRA (PARTS)


Prostatic part descends through the prostate and can
dilate to change significantly in size.
Membranous part exists between the prostate and the
beginning of the penis and is the section targeted if a
catheter needs to be inserted.
• Serosa - composed of a surface layer of mesothelium Spongy part traverses the penis to end at the external
supported by loose irregular connective tissue. urethral orifice.

TRANSITIONAL EPITHELIUM (OR UROTHELIUM)


• is a specialized stratified epithelium found in the lower
urinary tract. It rapidly adapts to distention and
contraction by changing from a taller to thinner
epithelium. Umbrella cells are highly dynamic cells at
the luminal surface.

RELAXED (NON-STRETCHED
• The transitional epithelium (urothelium) has several
layers of cells and large, dome-shaped umbrella cells FEMALE URETHRA
on its surface. (They are called umbrella cells
• The female urethra is a tube 4.5 cm long, areas of
because they cover several underlying epithelia cells.)
pseudostratified columnar epithelium in the middle
and lined with stratified squamous epithelium in the
distal parts.
• The mid part of the female urethra is surrounded by
an external striated voluntary sphincter.

RELAXED (NON-STRETCHED
• The transitional epithelium has become thinner. The
umbrella cells have become elongated and flattened.

dññçørñëlïø
Module 8:
hïstøløgÿ Laboratory Notes
Reference: HISTO-GUIDE

PITUITARY GLAND
• situated at the interface between the brain and the
rest of the body and secretes numerous hormones
affecting many aspects of physiology.
PINEAL GLAND
• secretes melatonin which modulates sleep patterns

THYROID GLAND
• secretes hormones that primarily influence the
metabolic rate, protein synthesis and calcium
metabolism

The Endocrine System PARATHYROID GLANDS


• secretes parathyroid hormone that causes increase
• is composed of glands that synthesize and secrete
Ca2+ blood levels
products, called hormones, directly into the blood rather
than through a duct. ADRENAL GLANDS
• secretes a variety of hormones including adrenaline
Hormones and the steroids aldosterone and cortisol
• are transported throughout the body where they PANCREATIC ISLETS (OR ISLETS OF LANGERHANS)
influence only those cells that have receptors for that • secrete hormones that regulate glucose and lipid
hormone. metabolism
OVARY
• Proteins/peptides (e.g., insulin, growth hormone) • secrets the steroids estrogen and progesterone that
• Lipid derivatives - derived from cholesterol or fatty regulate changes in menstrual cycle and during
acids (e.g., steroids, eicosanoids) pregnancy.
• Amino acid derivatives - derived from tyrosine or TESTIS
tryptophan (e.g., epinephrine, melatonin, thyroid • secretes the steroid testosterone that regulate the
hormones) development of male characteristics.
Many other tissues have isolated single cells or groups of
Endocrine - released into blood and act at long distances
cells with endocrine functions.
Paracrine - diffuse a short distance through tissue fluids to
nearby cells •
Autocrine - act on the same cell that produced the hormone • Pituitary
Endocrine Glands • The pituitary gland (hypophysis) is often called the
"master gland" of the body because it produces
• secrete their hormones into the blood where they are hormones that regulate other endocrine glands. It
transported to other tissues. They bind with has two major parts: anterior pituitary and posterior
receptors on target cells and alter their function. pituitary.

dññçørñëlïø
NEGATIVE FEEDBACK LOOPS
The anterior pituitary maintains homeostasis by using
negative feedback to regulate hormone secretion.

• The hypothalamus signals the pituitary gland to


release more of a pituitary hormone (e.g., ACTH).
• The pituitary hormone acts on another endocrine
gland (e.g., adrenal cortex) to release additional
hormones (e.g., cortisol) that produce physiologic
actions.
• These hormones also act on the hypothalamus Pars Distalis - comprises most of the anterior lobe
and/or anterior pituitary to inhibit the release of the (~75%) and contains five types of endocrine cells.
pituitary hormone (e.g., ACTH).
Chromophils - stain with H&E and secrete hormones.
Overall, a negative feedback loop means that when the level
of a pituitary hormone is high, further secretion of the
hormone is inhibited. When the level of the hormone is low,
the secretion of the hormone is stimulated.
ANTERIOR PITUITARY
• The anterior pituitary (adenohypophysis) secretes six
hormones. Each hormone is secreted by a different
cell type, except for FSH and LH which are secreted
by the same cell type. Acidophils - stain pinkish-red with H&E.

• stimulates the adrenal cortex to produce steroids


Mineralocorticoids (e.g., aldosterone)
• act on the kidney to regulate ion balance and
increase blood pressure
Glucocorticoids (e.g., cortisol)
• stimulate gluconeogenesis, suppresses immune • Somatotrophs - growth hormone (GH).
responses, and modulates CNS function
• Mammotrophs (Lactotrophs) - prolactin.
Basophils - stain bluish-purple with H&E.
• stimulates the thyroid to produce thyroxine (T4) and
triiodothyronine (T3 which influence the basal
metabolic rate and protein synthesis

Follicle-stimulating hormone (FSH)


• Testis - stimulates spermatogenesis and inhibin
production
• Ovary - stimulates development of ovarian follicles
and estrogen production
• Corticotrophs - ACTH.
Luteinizing hormone (LH) • Thyrotrophs - TSH.
• Testis - stimulates secretion of testosterone • Gonadotrophs - FSH and LH.
• Ovary - stimulates ovulation and progesterone Chromophobes - stain poorly with H&E and do not secrete
production hormones.

Prolactin - stimulates milk production in mammary glands

Growth hormone (GH) - effects by direct action of GH and


indirect actions through GH-induced production of insulin-like
growth factor-1 (IGF-1) by the liver; major promoter of body
growth
dññçørñëlïø
Sinusoidal Capillaries - extensive network that receives Pituicytes - most nuclei belong to glial cells.
hormones from acidophils and basophils.
Herring Bodies - dilations of axons filled with neuro-
secretion vesicles.

Pars Intermedia - thin remnant (<2%) at interface between


the anterior and posterior lobes that contains numerous
colloid (protein)-filled cysts (Rathke's cysts; #1 and #2).
Thyroid
• The thyroid gland is a bilobed endocrine gland. It is
unique in that it stores its hormones bound to an
extracellular pool of protein (colloid).

Capsule - enclosed by a thin layer of connective tissue.

Rathke's cysts

Trabeculae - connective tissue extends inwards from the capsule to


partially outline irregular lobes and lobules.

POSTERIOR PITUITARY
- secretory follicles constitute the functional units of the gland.
• Posterior pituitary (neurohypophysis) only secretes
two hormones. Each hormone is synthesized in cell Thyroid Follicles - spherical follicles of varying size (50 to 500 µm) in
which thyroid hormones are stored.
bodies of neurons in the hypothalamus and released
from axon terminals in the posterior pituitary.

- increases water
reabsorption in the kidney
Oxytocin
• Stimulates contraction of smooth muscle in the
uterus during childbirth
• Stimulates ejection of milk by the mammary gland Colloid - lumen of each follicle is filled with the gel-like mass called colloid.
It is mostly the protein thyroglobulin (pink) and bound thyroid hormones
(triiodothyronine and tetraiodothyronine (or thyroxin)). The clear space
around the colloid is a shrinkage artifact.

dññçørñëlïø
Follicular Cells - follicles are lined by a simple cuboidal to Trabeculae - connective tissue extends inwards from the
columnar epithelium depending on functional activity. capsule to partially outline irregular lobes and lobules.
Secrete thyroid hormones when active.

Capillaries - a rich network surrounds each follicle.

Chief Cells - the majority of cells in the parathyroid.


• Small cells (5 to 8 µm diameter) with dark nuclei and
thin rim of lightly stained cytoplasm.
• They secrete parathyroid hormone (PTH).

Parafollicular Cells - small numbers of larger cells


located at the periphery of follicles that secrete calcitonin.
They stain poorly with H&E making identification difficult.

Oxyphil Cell- larger cells (8 to 12 µm diameter) with dark


nuclei and strongly eosinophilic cytoplasm because of
numerous mitochondria. They appear after the first decade
of life and are thought to be non-secretory cells

Parathyroid
• Parathyroid glands secrete parathyroid hormone
(PTH) in response to low blood levels of calcium.
PTH secretion causes the release of calcium from
bones by stimulating osteoclasts, inhibition of
osteoblasts, and increased reabsorption of calcium
in the kidney.

Clear Cells - larger cells (8 to 12 µm diameter) with dark


Capsule - enclosed by a thin layer of connective tissue. nuclei and a watery, clear cytoplasm.

Adipose Cells increase with age.


dññçørñëlïø
Adrenal Glands Zona Fasciculata - middle zone (80%) of two-cell wide
• Adrenal glands produce a variety of hormones that
vertical cords that secrete cortisol. The cells have a
help regulate metabolism, blood pressure, response
central nucleus and lipid filed ("foamy") cytoplasm.
to stress, and other essential functions.
• located on the top of each kidney

Capsule - enclosed by a thin layer of connective tissue.

Zona Reticularis - inner zone (7%) of one-cell wide


anastomosing rows that secrete precursors of
testosterone. The cells have a central nucleus and
Afferent Blood Vessels - penetrate the capsule and branch eosinophilic cytoplasm often with lipofuscin pigment.
into sinusoids that supply the cortex and medulla.

Cortex - cells that synthesize and secrete steroid hormones.


lipofuscin pigment

Sinusoidal Capillaries - rich network of blood vessels.

Zona Glomerulosa - outer zone (15%) of glomerular-like


clusters of cells and secrete aldosterone. The cells have a
central nucleus and lipid filled ("foamy") cytoplasm.

dññçørñëlïø
Pancreatic Islets
• Pancreatic islets (or islets of Langerhans) are
'islands' of endocrine cells located within the
pancreas. They secrete hormones (insulin and
glucagon) important in the regulation of glucose in
blood.

These "islands" of endocrine cells (or islets of


Langerhans) are lighter staining than the exocrine cells by
H&E.

Chromaffin Cells - modified postganglionic sympathetic


neurons that secrete catecholamines (epinephrine or
norepinephrine).

ENDOCRINE PANCREAS
• The pancreas is a mixed exocrine and endocrine
gland. Most of the pancreas (~95%) is exocrine cells
that secrete digestive enzymes into the duodenum.
Scattered throughout the pancreas are clusters of
only a few cells to several thousand cells (2 to 5%)
that secrete hormones into the blood that regulate
glucose.
Ganglion Cells - infrequent sympathetic ganglion cells.

• The endocrine cells occur as irregular cords


surrounded by fenestrated capillaries.

(Note the size of an islet cannot be inferred from its cross-


section in a single section. Is the cross-section from the
Medullary Vessels - large veins that drain the organ. top, middle, or bottom of ellipsoid islets?) Islets contain four
major types of endocrine cells:

Beta Cells - most common and secrete insulin.


Alpha Cells - second most frequent and secrete glucagon.
Delta Cells - secrete somatostatin.
PP Cells - secrete pancreatic polypeptide.
• These cell types cannot be distinguished by H&E.
However, they can by electron microscopy or
immunohistochemical staining for the hormones.

dññçørñëlïø
Module 9: hïstøløgÿ Laboratory Notes
Reference: HISTO-GUIDE

Cortex - outer region that is the site of oocyte


development.

Ovarian Follicles - oocytes surrounded by one or more


layers of cells.
Female Reproductive System Primordial Follicles - oocytes arrested in development are
• The organs of the female reproductive system are the located in the outer cortex.
ovaries, oviducts, uterus, vagina, placenta, and
mammary glands. They work together for the production
of female gametes (oocytes), fertilization, support of the
developing fetus, delivering it to the outside world, and
nutrition of the newborn.

Ovary
• The ovaries are responsible for the production of an
oocyte (oogenesis) and secretion of female sex • Primary Oocyte - large (25 to 30 µm), round to
hormones (estrogen and progesterone). When it releases oval cells with a vesicular nucleus.
a mature ovum, it travels down the oviduct to the uterus.
Capsule - covers the outer surface.

• Zona Pellucida - layer of glycoproteins between


the oocyte and granulosa cells. It is visible a thin,
• Germinal Epithelium - the surface is covered by a
eosinophilic band in many follicles.
simple cuboidal epithelium.

• Tunica Albuginea - layer of dense irregular connective


• Follicular Cells - single layer of flattened cells that
tissue that supports the epithelium.
surround each oocyte.

dññçørñëlïø
Oviduct
• The oviducts (uterine tubes; fallopian tubes) are Mucosa - exhibits thin longitudinal folds that project into the
fibromuscular tubes that transport an ovulated ovum from lumen.
the ovary to the uterus. Fertilization usually takes place in
the oviduct.

thin
Infundibulum with Fimbria - funnel-shaped segment open to longitudinal
the peritoneal cavity with fringed extensions that extend folds
toward the ovary.

Simple Columnar Epithelium - consists of two types of cells:

fringed • Ciliated Cells - the cilia wave towards the uterus to


help propel ovum or fertilized zygote to the uterus.
extensions • Peg Cells - secrete fluid that provides nutrients for the
ovum or fertilized zygote.

Lamina Propria - contains blood vessels and nerves.

Ampulla - longest region and site of fertilization.

Muscularis - consists of an inner circular or spiral layer and an


outer longitudinal layer whose peristaltic contractions help
propel the ovum or fertilized zygote to the uterus.

Isthmus - narrow region near the uterus.


Intramural Part - short region that passes through the
uterine wall and opens into the uterus.

dññçørñëlïø
Serosa - composed of a simple cuboidal epithelium (or Endometrial Stroma - the underlying lamina propria is
mesothelium) supported by a thin layer of connective highly cellular (stellate cells, macrophages and
tissue. lymphocytes).

Uterine Glands - during the menstrual cycle the surface


epithelium invaginates into the stroma to form simple
Uterus tubular glands lined with mostly non-ciliated secretory
• The uterus is a hollow organ in which the fetus develops. cells.
The uterine wall is composed of three layers:

Endometrium - specialized mucosa that undergoes marked


changes during the menstrual cycle.

Basal Layer (or stratum basalis) - lower third of the


mucosa that is retained during menstruation and
regenerates the functional layer.

Functional Layer (or stratum functionalis) - the upper two


thirds of the mucosa that develops glands and is lost during
menstruation.

This specimen is from the early secretory phase of the


menstrual cycle because the uterine glands are mostly
straight and only slightly dilated.
Simple Columnar Epithelium - ciliated columnar and non- Glycogen stored in the base of the epithelial cells is
ciliated secretory cells. characteristic of early secretory glands. (They appear
empty because glycogen was extracted during sample
preparation.)

dññçørñëlïø
Myometrium - composed of three indistinct layers of smooth Straight Arteries - supply the basal layer.
muscle.

Inner and Outer Layers - mostly longitudinal bundles of


smooth muscle.
Middle Layer (or stratum vasculare) - thickest layer of mostly
circular or spiral bundles of smooth muscle with numerous blood
vessels. Spiral (or Coiled) Arteries - pass through the basal layer
and supply the functional layer.

Perimetrium - covered by an outer serous layer or visceral


peritoneum that is continuous with the broad ligament.

Terminal Capillaries - dilated (or ectatic) capillaries that


arise from spiral arteries.

The endometrium contains a unique system of blood vessels


that undergo marked changes during the menstrual cycle

Arcuate Arteries - 6 to 10 branches of the uterine artery that


encircle the uterus in the myometrium.

Because the two layers of the endometrium have separate


blood supplies, the basal layer is unaffected by the loss of
blood to the functional layer during menstruation.

Cysts - remnants of former uterine glands.

Radial Arteries - branches of arcuate arteries that ascend


into the endometrium and give rise to:

dññçørñëlïø
Uterine (menstrual) Cycle
• The uterine cycle is divided into three phases based
on changes in the inner surface of the uterus
(endometrium):

Menstruation (days 1 to 4)- in the absence of implantation, the


cells die off, shed and pass out as menstrual bleeding.
Proliferative phase (days 5 to 14) - grows and proliferates into a
thick, blood vessel-rich lining.
Secretory phase (days 15 to 28) - secretions rich in glycogen to
support development of the embryo.
Stratified Squamous Non-Keratinized Epithelium
continuous with the lining of the vagina.
Cervix
• The cervix is the lower end of the uterus that opens into
the vagina. During menstruation, it allows the passage of
menstrual fluid from the uterus. In childbirth, it widens
(dilates) to allow passage of the baby from the uterus to
the outside world.

Mucosa
• unlike the rest of the uterus, it is not lost during
menstruation.

Endocervix
• Estrogen promotes the storage of glycogen in the
• forms the wall of the cervical canal. middle and upper layers of the epithelium.
• Simple Columnar Epithelium - mostly mucus-secreting .Cervical Glands - relatively few glands are found and are
cells continuous with the lining of the body of the similar to those in the endocervix.
uterus.

• Cervical Glands - branched glands of mucus-secreting


cells located in the lamina propria.
Transformation Zone
• abrupt junction between the mucus-secreting
columnar epithelium of the endocervix and the
squamous epithelium of the ectocervix.

Ectocervix (or exocervix)


• part of the cervix that protrudes in the vagina (portio
vaginalis) and contains the opening of the uterus
(external os).

dññçørñëlïø
Nabothian Cysts Lamina propria - dense irregular connective tissue rich
in collagen and elastic fibers.
• develop as stratified squamous epithelium grows over
mucus-secreting simple columnar epithelium and
entraps large amounts of mucus.

Muscularis
Cervical Wall • composed of two indistinct layers of smooth muscle.
• composed of dense connective tissue rich in both An inner circular layer and a much thicker, outer
collagen and elastic fibers. Unlike the rest of the longitudinal layer.
uterus, it contains little smooth muscle.
Adventitia
• composed of an inner layer of dense connective
tissue and an outer layer of loose connective tissue

• There are no Meissner or Pacinian corpuscles in


the vaginal wall (unlike skin). (The vagina is often
confused with the esophagus. However, it does not
have a muscularis mucosae or glands.)

Vagina Placenta
• The vagina is a fibromuscular tube that connects the • The placenta develops from both fetal (chorion) and
cervix of the uterus to the vestibule of the external maternal (decidua basalis) tissues. This allows the
genitalia. It is kept moist by mucus produced by exchange of nutrients and waste between the fetal
cervical glands. and maternal circulations.
The vaginal wall is composed of three layers: • The placenta is composed of 15 to 20 regions
called cotyledons. This specimen is a portion of a
cotyledon from late pregnancy.
Mucosa
• contains numerous transverse folds (or rugae).
• Stratified Squamous Non-Keratinized Epithelium Villi
Estrogen promotes the storage of glycogen in the middle and • projections of the fetal chorion that extend into
upper layers of the epithelium. lacunae in which maternal blood flows. Exchange
between the two circulations occurs through the
villus wall.

As epithelial cells are exfoliated, glycogen is released and


metabolized by bacteria producing lactic acid. This lowers the dññçørñëlïø
pH in the lumen and inhibits the growth of pathogens.
- consists of two types of
Decidual Cells
cells:
• Cytotrophoblasts - cuboidal cells with pale staining • clusters of large round or oval cells.
cytoplasm and euchromatic nuclei. They are absent
by mid to late pregnancy so are not seen in this
specimen.
• Syncytiotrophoblasts - multinucleated cuboidal cells
with microvilli.

Fetal and maternal blood do not mix in the placenta.


Umbilical Cord
• The umbilical cord connects the fetus to the
placenta.
- mesenchymal connective tissue forms the core of Amniotic Epithelium
villi and contains fetal capillaries and venules. • the exterior surface is covered by a simple
cuboidal epithelium.
Blood Vessels
• exchange oxygen, nutrients and waste
products between the fetus and the
placenta.
Two Umbilical Arteries - carry deoxygenated
blood from the fetus to the placenta.

Placental Arteries
• develop from arteries in the endometrium to supply
maternal blood to the lacunae.

Single Umbilical Vein - delivers oxygenated blood


from the placenta to the fetus. It has a thick layer of
circular smooth muscle unlike adult veins.

Basal Plate (Stratum Basalis)


• the part of the uterus to which chorionic villi are
anchored. This specimen contains only a portion of
the basal plate.

Wharton's Jelly
• the blood vessels are embedded within a
matrix of embryonic mucous connective
tissue.

dññçørñëlïø
• Mostly ground substance (primarily hyaluronic acid • Lactiferous Duct - each lobe is drained by a single
and chondroitin sulfate) with a low abundance of lactiferous duct that opens into the nipple. It is
collagen or reticular fibers. lined by a double layer of cuboidal or columnar
cells surrounded by a sheath of connective tissue
with myoid cells.

• Mesenchymal Cells - near the remnant of the


allantois are many stellate and spindle-like Lobules - enclosed by a thin layer of connective tissue.
(fusiform) mesenchymal cells. However, spindle- Intralobular Ducts - lined by one or two layers of
shaped mesenchymal cells are indistinguishable cuboidal cells surrounded by a thin layer of connective
from resting fibroblasts. tissue.

Terminal Ductules - branches of intralobular ducts lined


with cuboidal secretory cells.

Mammary Gland
• Mammary glands are compound, tubulo-
alveolar glands whose structure changes
depending on the reproductive status of
females.

Lactating Mammary Gland - during pregnancy the


mammary gland undergo morphologic and functional • During pregnancy, they differentiate into
maturation. secretory alveoli that produce milk
Alveoli - grow and expand during pregnancy and
lactation.

Simple Epithelium - cuboidal or columnar secretory cells


• Lobes - 15 to 20 lobes separated by septae of that synthesis and secrete milk.
connective tissue with adipose cells. Milk - the eosinophilic material within the alveolar
lumen and ducts is proteins from secreted milk.
Intralobular Stroma - loose connective tissue with few
adipose cells. dññçørñëlïø
Seminiferous Tubules
• each lobule contains 1 to 4 highly-coiled
seminiferous tubules lined by a germinal
epithelium that is the site of sperm
production.

Male Reproductive System


• The male reproductiveMa
system consists of the
testes, conducting tubules and ducts (epididymis,
vas deferens, ejaculatory ducts), accessory sex
glands (seminal vesicles, prostate, and Sertoli Cells - large, columnar cells that extend the
bulbourethral glands), and the penis. These organs full thickness of the germinal epithelium.
work to together to produce male gametes (sperm)
and the other components of semen.
Testis
• The testis are ovoid organs responsible for sperm
production (spermatogenesis).

Tunica Albuginea
• capsule of thick connective tissue.
• Irregular, euchromatic nucleus with a single,
prominent nucleolus.
• Blood-testis Barrier - these cells separate
the basal epithelial compartment (of
spermatogonia) from the luminal
compartment (of spermatocytes, spermatids
and sperm).

Lobules Spermatogenesis - the process by which


• pyramid shaped lobules separated by septae of spermatogonia develop into sperm. Many dividing
connective tissue that extend inward from the cells are seen in the germinal epithelium.
capsule.

• Spermatogonia - single layer of germ cells


resting on the basement membrane.
• Primary Spermatocytes - arise from
spermatogonia and cross from the basal
epithelial to luminal compartment of the
germinal epithelium.
septae
• Secondary Spermatocytes - arise from
primary spermatocytes and rapidly divide
(rarely visible). dññçørñëlïø
Spermatids - arise from secondary spermatocytes and Straight Tubules (or Tubuli Recti) - short, terminal section
undergo spermiogenesis to transform into sperm. of each seminiferous tubule lined only by Sertoli cells.

Rete Testis - straight tubules empty in an anastomosing


• Small, spherical cells (8 µm or less) with labyrinth lined by a simple cuboidal or columnar
intensely stained nuclei near the lumen. epithelium.
• Embedded in the cytoplasm of Sertoli cells.

Sperm - released into the lumen.


• Small, pointed head with a long tail (~60 µm
long).

Epididymis
• The epididymis is a single, coiled duct (~250 µm in
diameter and ~4 meters in length) in which sperm
undergo maturation (motility and capacitance). Sperm
are stored in the tail of the epididymis.
Pseudostratified Columnar Epithelium
Leydig Cells (or Interstitial Cells) • has a smooth luminal surface (unlike the "wavy" or
• found in the connective tissue (or interstitium) "saw-toothed" appearance of the efferent ductules.
between seminiferous tubules.

• Principal Cells - tall columnar cells (~75 to 80 µm)


• Large, round cells (20 to 30 µm diameter) with with numerous modified, long microvilli called
vesicular nuclei and eosinophilic cytoplasm. stereocilia extend from their luminal surface.
• Usually occur in clusters of 5 to 30 cells.
• Secrete testosterone.
Mediastinum
• region in which seminiferous tubules converge
and sperm exits the testis.

• Basal Cells - small, round stem cells resting on the


basement membrane.

dññçørñëlïø
Muscular Layer Muscular Layer
• gradually increases in thickness through the • consists of three layers (inner longitudinal,
length of the duct. middle circular and outer longitudinal) of
smooth muscle similar to the tail of the
epididymis.

Head and Body - consists of a single, circular layer of


smooth muscle. • Sperm is transported through the vas
• Spontaneous, rhythmic peristaltic contractions deferens by peristaltic contractions of the
move sperm through the duct. smooth muscle.
Tail - consists of three layers (inner longitudinal, middle Seminal Vesicle
circular and outer longitudinal) of smooth muscle. • The seminal vesicles are unbranched, highly-
• Neural stimulation induces contraction and forces coiled tubular glands. Their secretions make up 60
percent of the volume of the semen. This fluid is
sperm from the epididymis into the vas deferens
high in fructose that acts as the main energy
Stroma source for sperm outside the body.
• the single, coiled duct is embedded in connective Muscosa
tissue. • forms highly convoluted folds that appear as
cross bridges ("honey combed" or "lace-like")
in cross-sections.

Ductus Deferens
• The ductus deferens (or vas deferens) is a muscular
Pseudostratified Columnar Epithelium - although
tube connecting the epididymis with the urethra in the classified as a pseudostratified epithelium, the nuclei
prostate. are located at the base of the cells.

Duct • Simple Columnar Cells - Golgi apparatus is


• resembles that of the epididymis except the luminal well-developed in these secretory cells and
surface has longitudinal folds. It is lined with a often appears as lightly-stained region
pseudostratified columnar epithelium. adjacent to the nucleus.
• Basal Cells - small, round stem cells resting
on the basement membrane.
• Lumen - many are filled with an eosinophilic
substance.

Lamina Propria - connective tissue rich in collagen


and elastic fibers and smooth muscle.

• Principal Cells - tall columnar cells (~75 to 80 µm) with


numerous modified, long microvilli called stereocilia
extend from their luminal surface.
• Basal Cells - small, round stem cells resting on the dññçørñëlïø
basement membrane.
Muscular Layer Perialveolar Connective Tissue (lamina propria) - dense
• consists of an inner circular and outer longitudinal irregular connective tissue with interlaced bundles of
layers of smooth muscle. smooth muscle.

• The columnar epithelial cells secrete a clear,


slightly alkaline fluid into the urethra through short
ducts.

The columnar epithelial cells secrete a mucoid substance high


in fructose for sperm maturation that comprises 70% of the
seminal fluid.

Contraction of the smooth muscle during ejaculation forces the


seminal fluid into the ejaculatory duct.
Penis
Prostate • The penis is composed of three columns of erectile
• The prostate is composed of compound tubuloalveolar tissue bound together by the tunica albuginea.
glands that contributes a slightly alkaline fluid to
semen. These secretions help neutralize the acidity of Tunica Albuginea
the vagina, prolonging the lifespan of sperm. • sheath of dense irregular connective tissue that
surrounds the erectile bodies.
Parenchyma
• consists of 30 to 50 tubuloalveolar glands.

Glandular Epithelium - mostly simple columnar and cuboidal


epithelial cells but may have regions of pseudostratified
columnar epithelium.

Corpora Cavernosum
• a pair of vascular channels located dorsally within
the penis.

Basal Cells - small, round stem cells resting on the basement


membrane.
Corpora Amylacea - concretions of prostatic fluid in the lumen
of some glands in older men.
• Often have concentric lamellar appearance.
• They gradually increase in number and size with age.

Walls and trabeculae consist of connective tissue and


smooth muscle.
• Smooth Muscle - abundant in trabeculae and
relaxes during erection.
• Helicine Arteries - normally coiled, but straighten
and dilate on erection.

Stroma
• Capsule - dense irregular connective tissue with
interlaced bundles of smooth muscle. (Not seen in this
specimen.)
dññçørñëlïø
• Blood Sinuses - labyrinth lined with endothelial cells and
become filled with blood during erection.

• Central Artery (or Deep Artery of the Penis) - each


corpus contains a single, central artery.

Corpus Spongiosum
• located ventrally within the penis.

Erectile Tissue - similar in structure to the corpora


cavernosum but does not become as turgid on erection.
Urethra - lined with a pseudostratified columnar epithelium.

Mucous Glands of Littre and Ducts - secrete a clear mucus on


erection.

dññçørñëlïø
HISTOLOGY CYCLE 1 MIDTERM 84/100
MODULE 1
Question 1
Detecting certain tissues involves using the correct stains. What stain among the
choices below could be best in detecting cellular deposits of glycogen in tissue
samples?
Response: Periodic-acid Schiff reaction
Score: 1 out of 1 Yes

Question 3
Which among the following statements about enzymes used in enzyme histochemistry
below is CORRECT?
Response: Peroxidase promotes the oxidation of substrates with the transfer of
hydrogen ions to hydrogen peroxidase.
Score: 1 out of 1 Yes

Question 6
Studying fresh, unstained cells and tissue samples are practiced in some laboratories.
What is the appropriate Microscopy to be used in such methods?
Response: Phase contrast microscopy
Score: 1 out of 1 Yes

Question 8
What process of tissue preparation involves the infiltration of reagents that would give
the tissue a translucent appearance?
Response: Clearing
Score: 1 out of 1 Yes

Question 9
Which among the following statement is INCORRECT about the Transmission Electron
Microscopy?
Response: Focused electron beam is moved sequentially from point to point across the
specimen’s surface.
Score: 1 out of 1 Yes

Question 12
What is an instrument used for sectioning tissue blocks of paraffin-embedded tissues
for light microscopy?
Response: Microtome
Score: 1 out of 1 Yes

Question 13
This process of preparation of tissues involves the gradual removal of water in different
concentrations of Alcohol to remove water from tissue block.
Response: Dehydration
Score: 1 out of 1 Yes

Question 14
What is a method for localizing cellular structures using a specific enzymatic activity
present in those structures?
Response: Enzyme Histochemistry
Score: 1 out of 1 Yes

Question 15
What are minor structural abnormalities not present in the living tissue that is brought
by procedure of preparation?
Response: Artifacts
Score: 1 out of 1 Yes

Question 16
Jean, a medical laboratory science student, was performing the lab activity of tissue
preparation. Midway in the process, while doing the next step which was Clearing, the
tissue sample was soaked in the clearing agent and it turned into milky white in a few
minutes. What was likely the cause of the error?
Response: The tissue sample was not completely dehydrated, and the water present in
the tissue formed a cloudy solution with the clearing agent.
Score: 1 out of 1 Yes

Question 17
A Medical Technologist assigned in the Histopathology section was tasked to preserve
a tissue to be examined by the Pathologist. Upon finishing the preparation, it was
examined but was immediately rejected by the Pathologist due to the significant amount
of shrinkage on the tissue cells with hollow spaces in some parts. What likely was the
cause of the problem?
Response: The tissue sample was not properly preserved by the fixative thus losing its
tissue structure.
Score: 1 out of 1 Yes

Question 24
This method uses a highly specific of an antigen and its antibody to identify and localize
many specific proteins.
Response: Immunohistochemistry
Score: 1 out of 1 Yes

MODULE 2
Question 1
Among the given statements, which is TRUE about the different classifications of
neurons?
Response: Bipolar neuron has one and axon and comprises sensory neurons of the
retina.
Score: 1 out of 1 Yes

Question 2
What are cells that extend may processes that wrap around a CNS axon?
Response: Oligodendrocytes
Score: 1 out of 1 Yes

Question 3
A Laboratory instructor in Histology wanted to her students to examine the
endoneurium and perineurium that surrounds the Schwann cells for a laboratory activity.
What tissue slide could be used to demonstrate a preferred image?
Response: Peripheral nerve tissue slide
Score: 1 out of 1 Yes

Question 4
Which of the following statements about the Elastic cartilage is not TRUE?
Response: Perichondrium is not usually present and both chondrocyte and
chondroblast is present.
Score: 1 out of 1 Yes

Question 5
Which among the following statement is CORRECT about the Cartilage?
Response: It lacks blood vessels thus it is surrounded by the perichondrium for oxygen
and nutrient supply.
Score: 1 out of 1 Yes

Question 6
These are very large cells specialized for storage of triglycerides.
Response: Adipocytes
Score: 1 out of 1 Yes

Question 7
These fibers are found in delicate connective tissue of many organs, to which mainly
consists of collagen type III are known to be argyrophilic.
Response: Reticular fibers
Score: 1 out of 1 Yes

Question 8
A student was given a slide to examine in a laboratory activity. The tissue has a pinkish
color with cells aligning as they branched off with a transverse line where two cells
connect together. Some cells contain an unstained portion near the nucleus. What is the
purpose of the transverse line in between the cells?
Response: The transverse lines are composed many desmosomes and fascia adherens
junctions which provide strong intercellular adhesion during the cells’ constant
contraction.
Score: 1 out of 1 Yes

Question 9
This is a thin extracellular layer of specialized proteins found at the basal surface of all
epithelia that serves as semipermeable filter for substances.
Response: Basement membrane
Score: 1 out of 1 Yes

Question 10
These are very small channels in the central canals of osteons that delivers nutrients to
the osteocytes.
Response: Canaliculi
Score: 1 out of 1 Yes

Question 11
When a surgeon makes an incision in a muscle tissue, in what order would the
physician encounter these structures?
(1) epimysium, (2) skeletal muscle, (3) perimysium, (4) muscle fiber, (5) Endomysium,
(6) fascicle
Response: 1-2-3-6-5-4
Score: 1 out of 1 Yes

Question 12
In a laboratory activity, the laboratory instructor focused a slide containing the whitefish
blastula. A student was chosen to identify if a cell was undergoing phase of mitosis.
Looking under the microscope, the cell was smaller compared to the other cells with its
nucleus and nucleoli present and the centrosome unseen on its cytoplasm. What is the
correct interpretation of the phase to which the cell was undergoing and why?
Response: The cell was not undergoing the mitotic phase since the chromatid pairs
were not present and the chromosomes were still enclosed inside the nucleus and the
centrosome was not prominent.
Score: 1 out of 1 Yes

Question 13
Which of the following characteristic is unique to cardiac muscle?
Response: Often branched with intercalated discs
Score: 1 out of 1 Yes

MODULE 3
Question 1
Examination of a normal peripheral blood smear reveals a cell much larger than an
erythrocyte with a size of approximately 12 μm in diamter with a spherical nuclei. There
cells are < 20% of the total leukocytes. Which of the following cell types is being
described?
Response: Lymphocyte
Score: 1 out of 1 Yes

Question 2
Which of the following is NOT TRUE about pericytes?
Response: Secrete many ECM components and form their own basal lamina, which
fuses with the basement membrane of the endothelial cells
Score: 0 out of 1 No

Question 3
The simple columnar epithelium that covers the lymphoid nodules of Peyer patches
includes large epithelial _____________ with apical microfolds rather than the brush
border typical of the neighboring enterocytes
Response: Microfold (M) cells
Score: 1 out of 1 Yes

Question 4
This composed almost entirely of splenic cords (of Billroth) and splenic sinusoids and is
the site where effete RBCs in blood are removed.
Response: Splenic Red Pukp
Score: 1 out of 1 Yes

Question 5
Which tunic of an artery contains an endothelium?
Response: Tunica intima
Score: 1 out of 1 Yes

Question 6
Vasa vasorum serves as a function analogous to that of which of the following?
Response: Arterioles
Score: 0 out of 1 No

Question 7
The most abundant leukocyte with polymorphic, multilobed nuclei, and faint
cytoplasmic granules that contain many factors for highly efficient phagolysosomal
killing and removal of bacteria.
Response: Neutrophil
Score: 1 out of 1 Yes

Question 8
What type of tissue lines blood vessels?
Response: Stratified squamous epithelium
Score: 0 out of 1 No

Question 9
It has a thin paracortical region between cortex and medulla with high endothelial
venules (HEV).
Response: Lymph Nodes
Score: 1 out of 1 Yes

Question 10
It is the thickest tunic which consists of mainly cardiac muscle with its fiber arranged
spirally around each heart chamber.
Response: Myocardium
Score: 1 out of 1 Yes

Question 11
Which among the following is the main function of the thymus?
Response: Induction of central tolerance, which along with regulatory T cells prevents
autoimmunity.
Score: 1 out of 1 Yes

Question 12
The major function of this cell is to kill helminthic and other parasites. This cell can also
modulate local inflammation.
Response: Eosinophil
Score: 1 out of 1 Yes

Question 13
Which among the following type of tonsil is situated in the in the posterior wall of the
nasopharynx, is covered by pseudostratified ciliated columnar epithelium, and has a thin
underlying capsule.
Response: Pharyngeal tonsil
Score: 1 out of 1 Yes

Question 14
Which of the following is a distinct structure found specifically in the liver, spleen, and
bone marrow?
Response: Sinusoidal capillaries
Score: 1 out of 1 Yes

Question 15
Which of the following statements about the structural plan of blood vessels is/are
correct? I. Elastic arteries are the largest blood vessels in the body and include the
pulmonary trunk and aorta with their major branches. II. Large veins have a thick tunica
adventitia with numerous longitudinal smooth muscle bundles, which help to force blood
to flow toward the heart. III. Small-sized and medium-sized veins, particularly in the
extremities, does not have valves. IV. Venules and small veins have very thin walls and
few valves.
Response: I & II only
Score: 0 out of 1 No

Question 16
Shortly after her birth a baby is diagnosed with a mutation in the erythropoietin receptor
gene which leads to familial erythrocytosis (familial polycythemia). During the seventh to
ninth months of fetal development, the primary effect on her red blood cell production
was in which of the following?
Response: Bone marrow
Score: 1 out of 1 Yes

Question 17
Which of the following is NOT a distinguishing feature between larger veins and
arteries?
Response: The tunics in veins are not as clearly delimited as are the tunics in arteries
Score: 0 out of 1 No

Question 18
Many immune-related cellular activities are often impaired in aged patients. Which
lymphoid organ(s) normally develop less functionality and increasing amounts of
adipose tissue with age?
Response: Axillary lymph nodes
Score: 0 out of 1 No

Question 19
What is the earliest stage at which the specific granulocyte types can be distinguished
from one another?
Response: Myelocyte
Score: 1 out of 1 Yes

Question 20
The adventitia of this type of artery has no vasa vasorum with 3-10 layers of smooth
muscle in its tunica media.
Response: Small arteries
Score: 1 out of 1 Yes

Question 21
Which leukocyte is the second most abundant in a peripheral smear of blood?
Response: Lymphocytes
Score: 1 out of 1 Yes

Question 22
Which structure is partly encapsulated and covered by nonkeratinized stratified
squamous epithelium?
Response: Thymic (Hassal’s) corpuscle
Score: 0 out of 1 No

Question 23
Which structure would be most heavily labeled by an immunohistochemical method
targeting the CD8 surface antigen?
Response: Paracortex
Score: 1 out of 1 Yes

Question 24
Which of the following statements about the immune system is/are correct? I. Adaptive
Immunity is acquired gradually by exposure to microorganisms, is more specific, slower
to respond, and an evolutionarily more recent development than innate immunity. II.
Antimicrobial agents by leukocytes are a part of adaptive immunity III. Skin is
considered as a part of the first line of defense IV. Phagocytosis plays an important role
in adaptive immunity
Response: I & III only
Score: 1 out of 1 Yes

Question 25
Larger agranulocyte with distinctly indented or C-shaped nuclei. A precursor of
mononuclear phagocyte system.
Response: Monocyte
Score: 1 out of 1 Yes

Question 26
Which of the following statements about the MALT is/are correct? I. Peyers Patches is
found in the jejenum of small intestines II. Tonsils are small, regular masses of lymphoid
tissue in the mucosa of the posterior oral cavity III. Most of the immune cells in the
MALT are dispersed diffusely in the endothelium iv. The simple cuboidal epithelium that
lines the lymphoid nodules of Peyer patches include large epithelial M cells with apical
microfolds
Response: All of the choices are correct
Score: 0 out of 1 No

Question 27
When looking at the spleen, what are the invaginations of the capsule into the splenic
parenchyma called?
Response: Trabeculae
Score: 1 out of 1 Yes

Question 28
This consists of lymphoid tissue surrounding the central arterioles as the PALS and the
nodules of proliferating B cells in this sheath.
Response: Splenic White Pulp
Score: 1 out of 1 Yes

Question 29
What tissue is directly associated with and extends into the heart valves?
Response: Myocardium
Score: 0 out of 1 No

Question 30
What are the large aggregates of lymphatic tissue in the ileum?
Response: Peyer's patches
Score: 1 out of 1 Yes

Question 31
Which layer of the gastrointestinal tract contains the gut associated lymphatic tissue?
Response: Mucosa
Score: 1 out of 1 Yes

Question 32
Which cell type has cytoplasmic granules that contain heparin and histamine?
Response: Basophil
Score: 1 out of 1 Yes

Question 33
Which of the following cytokines act as mitogen for megakaryoblasts and their
progenitor cells?
Response: Thrombopoietin
Score: 1 out of 1 Yes

Question 34
A medical technology student examined a blood smear stained with supravital stain
under a compound microscope and noticed a cell almost the same size with that of RBC
yet with some granulation in its cytoplasm with no nucleus. Which stage of
erythropoiesis do you think the student was able to see?
Response: Polychromatophilic erythroblast
Score: 0 out of 1 No

Question 35
This type of capillary has a sieve-like structure that allows more extensive molecular
exchange across the endothelium. This is found in organs where molecular exchange
with blood is important, such as endocrine organs, intestinal walls, and choroid plexus.
Response: Fenestrated capillary
Score: 1 out of 1 Yes

Question 36
67. Which of the following statements about the layers of the heart is/are correct? I.
Epicardium is composed of mesothelium and a thicker layer of connective tissue. II.
Cardiac muscle fibers found in myocardium require high oxygen supply. III. Endothelial
cells form a permeability barrier between blood and the interstitial tissue.
Response: All of the choices
Score: 1 out of 1 Yes

Question 37
These are pale-staining fibers, larger than adjacent contractile muscle fibers, with
sparse, peripheral myofibrils and much glycogen.
Response: Purkinje fibers
Score: 1 out of 1 Yes

Question 38
Which of the following cytokines act as mitogen for all erythroid progenitor and
precursor cells?
Response: Erythropoietin
Score: 1 out of 1 Yes

Question 39
All of the following is true about hemopoiesis, EXCEPT:
Response: The red bone marrow produce blood and hemopoietic cells. While the yellow
bone marrow is filled with adipose tissue that exclude most hemopoietic cells.
Score: 0 out of 1 No

Question 40
Which function is carried out by all lymphoid tissues and organs?
Response: Filtration of blood
Score: 0 out of 1 No

Question 41
Capillary beds are supplied preferentially by one or more terminal arteriole branches
called
Response: Postcapillary venules
Score: 0 out of 1 No

Question 42
What do you call the simple squamous epithelium that lines the blood vessels?
Response: Endothelium
Score: 1 out of 1 Yes

Question 43
Individuals with Marfan syndrome have mutations in the fibrillin gene and commonly
experience aortic aneurisms. What portion of the arterial wall is most likely to be
affected by the malformed fibrillin?
Response: Tunica media
Score: 1 out of 1 Yes

Question 44
In which of the following is a venous portal system found?
Response: Liver
Score: 1 out of 1 Yes

Question 45
What is the brachial artery?
Response: Muscular artery
Score: 1 out of 1 Yes

Question 46
Which leukocyte usually has a bi-lobed nucleus?
Response: Eosinophil
Score: 1 out of 1 Yes

Question 47
It involves cytoplasmic change dominated by synthesis of proteins for the azurophilic
granules and specific granules.
Response: Granulopoiesis
Score: 1 out of 1 Yes

Question 48
Which of the following can be used to describe megakaryocytes?
Response: Process of dynamic cell projections from which one type of formed element
is released
Score: 1 out of 1 Yes

Question 49
Which description is true of continuous capillaries?
Response: Most common in both brain and muscle
Score: 1 out of 1 Yes

Question 50
What is the connective tissue sac surrounding the heart?
Response: Pericardium
Score: 1 out of 1 Yes

MODULE 4
Question 1
Which of the following papillae is the largest?
Response: Vallate papillae
Score: 1 out of 1 Yes

Question 2
A patient with Liver Cirrhosis has problems with absorption of fat-soluble vitamins and
digestion of fats. Which function of the liver has been affected?
Response: Production of steroid hormones
Score: 0 out of 1 No

Question 3
Which of the following enteroendocrine cells inhibits gastric acid secretion?
I. D cells
II. I cells
III. Mo cells
IV. K cells
V. G cells
VI. S cells
VII. EC cells
Response: II, IV, VI
Score: 1 out of 1 Yes

Question 4
A 30-year old male was diagnosed with autoimmune gastritis in which the parietal cells
were destroyed by autoantibodies. Which of the following will happen if there was
destruction of parietal cells?
Response: Increased in gastric pH
Score: 1 out of 1 Yes

Question 5
These secretory epithelial cells of the gastric glands produce hydrochloric acid and are
round or pyramid in shape with one central round nucleus.
Response: Parietal cells
Score: 1 out of 1 Yes

Question 6
Which of the following organs of the gastrointestinal tract absorbs water and
electrolytes and forms indigestible material into feces?
Response: Large intestine
Score: 1 out of 1 Yes

Question 7
All of the following organs are composed of serosa, EXCEPT:
Response: Anal canal
Score: 1 out of 1 Yes

Question 8
These are absorptive cells that are tall columnar with oval nuclei found basally.
Response: Enterocytes
Score: 1 out of 1 Yes

Question 9
It is a mass of striated muscle covered by mucosa, which manipulates ingested
material during mastication and swallowing.
Response: Tongue
Score: 1 out of 1 Yes

Question 10
Which of the following layers of the digestive tract contains denser connective tissue
with larger blood and lymph vessels and the submucosal plexus of autonomic nerves?
Response: Submucosa
Score: 1 out of 1 Yes

Question 11
All of the following enteroendocrine cells are located in the small intestine, EXCEPT:
Response: D cells
Score: 1 out of 1 Yes

Question 12
A patient presents symptoms with frequent urination and unintended weight loss. He
was diagnosed with Type 1 Diabetes, an autoimmune disorder, which cells of the
pancreas are affected?
Response: Beta cells
Score: 1 out of 1

MODULE 5
Question 1
A 45-year old male was diagnosed with emphysema which is a chronic lung disease caused by
cigarette smoking. Which of the following would be observed to the respiratory epithelium of the
cigarette smoker?
Response: Pseudostratified ciliated epithelium becomes stratified squamous epithelium
Score: 1 out of 1 Yes

Question 2
Which of the following organs of the respiratory tract conducts air?
I. Alveolar ducts
II. Larynx
III. Bronchi
IV. Bronchioles
V. Alveoli
VI. Trachea
VII. Nasal cavity
Response: I, III, IV, VI
Score: 0 out of 1 No

Question 3
Which of the following is the function of vibrissae?
Response: Filter out particulate material from inspired air
Score: 1 out of 1 Yes

Question 4
Which of the following regions of the respiratory tract contains c-shaped rings of hyaline
cartilage, with smooth muscle in the posterior opening of each?
Response: Trachea
Score: 1 out of 1 Yes

Question 5
These cells are columnar with broad, cylindrical apexes containing nuclei and narrower bases.
Response: Alveolar cells
Score: 0 out of 1 No

Question 6
These cells possess numerous dense core granules and are part of the diffuse neuroendocrine
system.
Response: Kulchitsky cells
Score: 1 out of 1 Yes

Question 7
All of the following are cells of respiratory epithelium, EXCEPT:
Response: Brush cells
Score: 0 out of 1 No

Question 8
Which of the following tissues lines the trachea?
Response: Pseudostratified epithelium
Score: 0 out of 1 No

Question 9
Infant respiratory distress syndrome is the leading cause of death in premature babies due to
incomplete differentiation of type II alveolar cells which results in a deficit of surfactant and
difficulty in expanding the alveoli in breathing. How will you differentiate type II alveolar cells
from type I alveolar cells?
Response: Type II alveolar cells have occluding junctions that prevent the leakage of tissue
fluid into the alveolar space.
Score: 0 out of 1 No

Question 10
Which of the following regions of the respiratory tract comprises respiratory epithelium?
I. Trachea
II. Most areas of nasal cavities
III. Larynx
IV. Nasopharynx and posterior oropharynx
V. Superior areas of nasal cavities
Response: I, II, III
Score: 1 out of 1 Yes

Question 11
These cells are characterized ultrastructurally by unique cytoplasmic lamellar bodies, large
granules with closely stacked layers of membrane involved in surfactant synthesis.
Response: Type II alveolar cells
Score: 1 out of 1 Yes

Question 12
All of the following are functions of Clara cells, EXCEPT:
Response: Apoptosis of cells
Score: 1 out of 1 Yes

Question 13
Which of the following regions of airway lacks supporting cartilage and glands?
Response: Bronchioles
Score: 1 out of 1
HISTOLOGIC TECHNIQUES 11. What is the best approach to identify and
1. What reagent in indirect immunohistochemistry is localize a specific protein in the tissues?
the label or tag incorporated? Response: Immunohistochemistry
Response: Secondary antibody 12. Which enzyme’s activity in the mitochondria is
2. Storage deposits of glycogen can be detected in detected with Cytochemistry?
tissues using what method? Response: Dehydrogenases
Response: Autoradiography 13. What biomolecule within the tissue serves as a
3. A tissue section was subjected to H&E staining target for hybridization?
showing acidophila. If these cells are examined Nucleic acid
using TEM which of these organelles are abundant
and responsible for acidophilia? CELL AND TISSUES
Response:Microtubules 1. Which among the ff. Tissue type is affected
4. At the operating room the circulating nurse called by an individual suffering from a sprained
the histopathology laboratory. The surgeon wanted elbow?
to know if he needs to remove only the breast mass a. Reticular tissue
or do mastectomy (removal of the whole breast). As b. Fibrocartilage
soon as the medical technologist received the c. Elastic cartilage
specimen he fixed the specimen with 10% formalin, d. Hyaline cartilage
is the action of the Medical technologist correct? 2. Which of the ff. Is the function of the
Response: No, because you can examine the basement membrane?
tissue in fresh preparation a. Modification of protein
5. Which of the following is not an advantage of the b. Molecular filtering
Frozen section? c. Excitability
Response: None of the choices d. Active ion transport
6. Tissue biopsy to diagnose metabolic diseases 3. What best describes a sebaceous gland?
that involve intracellular accumulation of cholesterol a. Compound tubuloacinar
phospholipids and glycolipids are best stained with b. Simple branched acinar
which dye? c. Compound tubular
Response: Sudan black d. Unicellular
7. Fixation preserves tissue structure and 4. The goblet cell may be found intermixing
morphology. Dehydration removes water before with other epithelial cells in which organs?
infiltration. a. Trachea
Response: First statement is true, the second b. Jejunum
statement is false c. Duodenum
8. What color does a tissue or cell rich in d. AOTA
polysaccharides impart when stained using 5. Which among the following is associated
Periodic Acid Schiff reaction? with intramembranous ossification?
Response: Purple or magenta (muna bi a. Development of the skull
nakabutang sa libro te choose your fighter na b. Formation of phalanges
lang sa duwa hehe) c. Growth of Ribs
9. In which of the following techniques was a vital d. Lengthening of femur
dye used to identify the hematopoietic stem cells? 6. Venipuncture was performed on a patient.
Response: Cell tissue & culture Which among the ff tissue types was the
10. A student is required to submit a tissue section first to be affected by the insertion of the
of the skin routinely processed. Which is normally needle?
not seen after routine tissue processing? a. Squamous epithelium
Response: exactly the same features as in the b. Dense irregular CT
body c. Areolar
d. Adipose 13. Which of the ff supports the cells and
7. What type of cartilage is tough as a cushion contains fluid nutrients to the cells and
or sometimes as shock absorbers carrying away their wastes and secretory
preventing damage during tension or products?
stress? a. Cell membrane
a. Hyaline b. Intracellular matrix
b. Fibrocartilage c. Cytoplasm
c. Elastic d. Extracellular matrix
d. AOTA 14. What happens to the nucleolus during
8. A tissue of adrenal gland was processed mitosis and meiosis?
with H&E. cells in the zona fasciculata a. Reappears during prophase,
appear washed out and spongy due to the disappears during telophase
accumulation of cholesterol. Electron b. Present inside the cell in all stages
microscopy of these cells will show c. Disappears during metaphase,
abundance of which organelles? reappears during interphase, and
a. SER present in prophase
b. Secretory organelles d. Disappears during prophase,
c. Golgi bodies reappears during late telophase,
d. RER present only in interphase
9. In the emergency room, an obese patient 15. A tissue of the epididymis is examined by
complained of difficulty of breathing and microscopy and revealed epithelial cells
chest at the left of his sternum. He was lining the lumen. Which protein is
diagnosed with ischemia. What tissue was responsible for the apical structure seen on
damaged? the epithelium?
a. Cardiac muscle a. Tubulin
b. Skeletal muscle b. Keratin
c. Smooth muscle c. Desmin
d. Bone d. Actin
16. What mesenchymal cells are responsible for
10. Which of the ff. organelles maintain the regenerating muscle cells?
shape of dendrites and axons? a. Myoblast
a. Microtubules b. Myocyte
b. Centrioles c. Myotubes
c. Microfilaments d. Satellite cell
d. Intermediate filament
11. A bacteria enters the nervous system. CIRCULATORY SYSTEM
Which among the ff cells will respond and
protect the surrounding tissue? 1. What type of immunity involves the actions
a. Dendritic cell of T lymphocytes?
b. Satellite cells Response: Adaptive immunity
c. Microglia 2. A 6-yr old boy was bitten by a dog two days
d. Ependymal cell earlier. His right hand is lacerated between
12. What intermediate filament protein is found the thumb and index finger. Upon
in the cytoplasm of squamous cells? examination of the doctor, small but
a. Keratin painless swellings inside the right elbow and
b. Laminin armpit are noted. The doctor explained that
c. Myosin these swellings are active lymph nodes in
d. Actin
response to the infection. Which statement 13. A sample of the glomerulus from an autopsy
explains this phenomenon? shows many pores closed by diaphragms
Response: Formation of germinal with a continuous external lamina on the
centers in the cortex outer surface of the endothelial cell. This
3. Which of the following statements describes sample will show abundance of what
the mesothelium? structure?
Response: All of the above Response: Fenestrated capillaries
4. What is defined as capable of mounting an 14. Which of the following cytokine is involved
immune response? in thrombopoiesis?
Response: Immunocompetent I.EPO III.IL
5. Examination of a bone marrow II.TPO IV.IL-3
sample showed granulocyte forming Response: II and IV
cells exhibiting development of 15. Which description is true only of secondary
specific granules. What cell type is lymphoid organs?
being described? Response: Contains crypts
Response: Myelocyte 16. What are long cells oriented lengthwise that
6. In what layer of the lymph node high line the sinusoids in the red pulp?
endothelial venules (HEV) can be seen? Response: Stave cells
Response: Paracortex 17. Malarial parasites invade RBC. Which
7. The skeleton of the heart consists of what immune defense will be seen in this type of
type of tissue? infection?
Response: Dense CT in the Response: Phagocytosis in the spleen
interventricular septum 18. What is the characteristic feature of a lymph
8. Which of the following tissues cannot be node?
found in the epicardium? Response: Medullary sinuses separate
Response: cardiac muscle the cords
9. Which among the following inclusion 19. What occurs in an allergen-antibody
fragments may be present in the response?
reticulocyte? Response: Mast cells release histamine
Response: RNA 20. What MALT is a blind evagination of the
10. What is known as a decrease in size and cecum with its lamina propria and
activity of the thymus during adolescence? submucosa filled with lymphoid follicles
Response: Involution Response: Peyer’s patches
11. Catherine had a streptococcal throat 21. A 4-year old boy had been treated with
infection and was treated immediately. After several ear infections in the past year.
6 months, the same infection occurred. Which organ is most active for cellular
However, the signs and symptoms were immunity?
less severe and she recovered after 3 days. Response: Thymus
What mechanism is responsible for her fast 22. What is the selection process in which
recovery? survival of thymocytes is dependent on the
Response: The previous infection ability to bind to MHC molecule?
induced production of Memory T and B Response: Negative selection
cells as part of adaptive immunity 23. Which of the following is true of pericytes?
12. A patient is suffering from a severe allergic Response: associated with basal lamina
reaction. Which among the following of capillary endothelium
leukocytes is responsible for this condition? 24. Which structure is partly encapsulated and
Response: Basophils covered by nonkeratinized stratified
squamous epithelium?
Response: Palatine tonsil peanuts had a complete blood count. Which
25. which among the following organs is of the blood cells will have a high count
necessary for the stimulation of red blood when viewed under the microscope?
cell production? Response: Eosinophil
Response: Kidney 37. A child was brought to the clinic for fever &
26. Which of the following best describes malaise. In the presence of a bacterial
sinusoids? infection, which formed element of the blood
Response:Discontinuous capillaries will be increased?
27. Which of the following structures of the Response: PMN
lymph nodes serve as the site for storage of 38. What is a unique feature of mucosa
developing B cells? associated lymphoid tissue?
Response: Cortex Response: Presence of crypts and M
28. Which of these cells are involved in cells
adaptive immunity? 39. The monocyte is derived from which cell?
a. T-helper cells a. Myeloid stem cell
b. Kupffer cell b. Lymphoid stem cell
c. Paneth cell c. Progenitor cell
d. Neutrophil d. Pluripotent stem cell
29. Where does the lymphoblast matures and 40. What protein is absent in the plasma/serum
develops to become naive B cells? when a blood sample is transferred to a test
Response: Spleen tube without anticoagulant?
30. A 40 year old man involved in a vehicular a. Albumin
accident arrived at the hospital. A blood b. Globulin
transfusion was requested due to blood c. Fibrinogen
loss. How will the bone marrow respond to d. Gammaglobulin
prevent further blood loss? 41. Anemia, a condition of having low
Response: The red marrow will undergo concentration of RBC may be due to which
thrombopoiesis of the following conditions?
31. Which among the following was the first a. Iron deficiency
layer pierced during venipuncture? b. Chronic kidney disease
Response: Tunica adventitia c. Bleeding
32. As an initial response to the dog bite, the d. AOTA
boy’s adaptive immune response is 42. Which among the ff mostly forms the middle
activated and will produce which antibody? layer of the muscular artery?
Response: IgM a. Loose CT
33. Which of these organs have no lymphoid b. Simple squamous
follicles? c. Elastic fibers
Response: bone marrow d. Smooth muscle
34. Which cell type gives rise to both memory 43. Which process occurs during granulopoiesis
and effector cells and is primarily associated but not during erythropoiesis?
with humoral immunity? Nucleus becomes increasingly lobulated
Response: B cells 44. Which among the following is an example of
35. Which of the following is true about tunica innate immunity?
adventitia? 1.) Protective covering of the skin
Response: contains autonomic nerve 2.) Cells circulating in the blood
fibers in large vessels 3.) Lymph nodes filtering lymph from the
36. A patient having swelling, redness and blood
itchiness on his face after eating a lot of
4.) Antibodies developed from prior main mode of transmission. Which among
exposure to an infection the following lymphoid structures will most
a. 1,2,3,4 likely be activated?
b. 2,4 a. Bone marrow
c. 1,3 b. Thymus
d. 1,2,3 c. Tonsils
45. What are unique epithelial cell specialized d. Lymph node
for transcytosis of particles and
microorganisms RESPIRATORY SYSTEM
a. Dendritic cell 1. A teenage girl who is known asthmatic
b. Microfold cell present at the ER with cough and difficulty of
c. Helper T-cell breathing. On examination, wheezing is heard.
d. Plasma cell Which of the following cell types and their location
46. What type of cell produces antibodies is correctly matched to a function it may perform
having a clock-faced appearance nucleus? during an asthma attack?
a. NK cells Response: Smooth muscle in bronchioles-
b. Macrophage bronchoconstriction
c. B cell Sa book muni ang answer: Mast cells in
d. T cell BALT, bronchoconstriction and edema
47. Which of these cells do not have 2. Which of the following cells is capable of
cytoplasmic projections? phagocytising lost erythrocytes and airborne
a. Megakaryocyte particulate matter?
b. Reticulocyte Response: Alveolar macrophage/ dust cells
c. Dendritic cells 3. Respiratory bronchioles is the region before
d. None of the choices the start of the respiratory process
48. Which part of the aorta will be affected by a Terminal bronchioles do have few opening to
defect in the production of Type II collagen? alveoli but it is not part of the respiratory portion.
a. Tunica media Response: Second Statement is correct and
b. Aortic valve First statement is false
c. Tunica intima 4. What cell is characterized by having a
d. Tunica externa dendrite end which detects odoriferous
49. What is the largest lymphoid organ in the substances?
body with abundant macrophages which Response: Olfactory neuron
destroys foreign substances, 5. Which feature involved in protection is absent
microorganisms and abnormal cells present in the digestive tract?
in the blood? Response: Cilia
a. Spleen 6. Which of the following is responsible for
b. Thymus producing phlegm causing productive cough and
c. Tonsils difficulty of breathing?
d. Bone marrow Response: goblet cells
50. In what stage of Erythropoiesis does 7. Which of the following increases in proportion to
hemoglobin starts to appear? the respiratory tract from trachea going to
a. Polychromatophilic erythroblast bronchioles?
b. Proerythroblast Response: Cartilage answer ya pro not
c. Basophilic erythroblast sure
d. Orthochromatophilic erythroblast 8. Which of the following is not part of the
51. The COVID-19 outbreak has been declared blood-air barrier?
with inhalation of droplets in the air as the Eliminate this choice: Endothelial cell
(Components of blood-air barrier: 2 or 3 a. Large intestine
highly attenuated thin cells lining the alveolus; b. Stomach
the fused basal laminae of these cells and the c. Small Intestine
endothelial cells of the capillaries; the thin d. Esophagus
capillary endothelial cells)
9. Which of the following correctly describes 4. In which type of cells is bile being produced?
Type II alvseolar cells? Response: Hepatocyte
Response: Cuboidal cells with rounded
nuclei 5. What type of cell in the liver stores Vitamin A and
other fat soluble vitamins?
10. The pleura that covers the lung is made up of Response: Hepatic stellate cells (or lto cells)
what tissue
Response: dense regular connective tissue 6. A 52-year old man was diagnosed with carcinoid.
The enteroendocrine cells producing this disorder
11. A premature neonate (30 weeks old) was differ from goblet cells in which of the following?
delivered through a Cesarean section. What would Response: The direction of release of
likely be the cause if the neonate experience infant secretion
respiratory distress syndrome?
Response: The bronchial tree lacks elastic 7. Which of the following will most likely result in the
and reticular fibers that supports during reduction in number of Paneth cells?
respiration Response: Increased number of intestinal
(based sa ginsearch ko due to insufficient bacteria
surfactant in the lungs) 8. Which description is true of the gallbladder?
12. Which among the following provides ELIMINATE THIS CHOICE: Absorbs bile
protection against damage to the vocal cords? 9. 60 year old Ramon was diagnosed with liver
Response Elastic CT cirrhosis. What changes will be seen in the liver?
Response: None of the choices
fibrosis and proliferation of fibroblasts
DIGESTIVE SYSTEM and hepatic stellate cells occur beyond the
1. Which part of the Esophagus muscularis is portal areas. The excessive connective tissue
a combination of both skeletal and smooth may disrupt the normal hepatic architecture and
muscle? interfere with liver function. (halin ni sa book)
a. Middle portion 10. Which of the following layers is affected when
b. Any of the choices you have diarrhea?
c. Lower third portion Response: mucosa
d. Upper portion 11. A patient was diagnosed with pernicious
anemia, a condition that prevents maturation of
2. Which best describes the bile canaliculi? RBC due to deficiency of vitamin B12. Which
a. Lumens are entirely sealed by junctional among the following cells may have been affected?
complex Response: Parietal cell
b. Part of portal triad
c. Bordered directly by endothelial cells 12. Certain antibiotic therapies slow replacement of
d. Surrounded by hepatic sinusoids cells lining the small intestine. This may cause the
3. An unlabeled microscopic slide with tissue was loss of what tissue?
examined showing the mucosa and submucosa Response: Simple columnar epithelium
poorly preserved. The muscularis is well-stained
showing striated muscle fibers. The slide most
likely shows a biopsy of which region of the GI tract
INTEGUMENTARY SYSTEM 8. A 65 year old woman complains to lose her
sense of touch especially when she runs
1. There are tissues found in the deeper layer her fingers tip on a smooth surface. Few
of the DERMIS except; tests were done and no abnormal findings
A. Sebaceous glands were seen. The physician explained it was
B. Nerves due to her age. On the basis of histology,
C. Adipose what caused her loss of sensation?
D. Dense Irregular CT A. Stratum corneum thicken making
light touch insensible
2. What star shaped cells are found mainly in B. Neural activity slows down due to
the stratum spinosum and have a significant increase number of neurons
role in the skin’s immunological reactions? C. AOTA
RESPONSE: Langerhan Cells D. Meissner corpuscles decline in
number
3. Upon performing venipuncture, what layer
of the epidermis is the first to be affected? 9. What layer consists of loose connective
RESPONSE: S. corneum tissue that binds the skin loosely to adjacent
organs?
4. A woman who frequently goes to the beach RESPONSE: Hypodermis
and maintained her suntanned, healthy look 10. Which of the following layers of the skin
developed a blue-violet painless lump on prevent water loss by producing lipid-rich
the left shoulder. Biopsy was done and the lamellar granules?
pathologist diagnosis is Merkel Cell RESPONSE: S. granulosum
carcinoma. What other cell type might also
be affected by the UV radiations? 11. When performing routine venipuncture,
A. Keratinocytes in stratum granulosum which layer of the skin contains the veins
B. Fibroblast in papillary layer needed for blood collection?
C. Basal stem cell RESPONSE: Dermis
D. All of the choices
12. Which of the following components of the
5. What is composed of a single row of epidermis provide sealant between cells?
cuboidal stem cells that underwent cell RESPONSE: Glycolipids and lipids
division to continuously produce
keratinocytes? 13. What is the mechanism of secretion of
RESPONSE: NOTA apocrine sweat glands?
RESPONSE: Merocrine
6. Which among the following skin tissue
allows the skin to retain its shape regardless 14. Which encapsulated mechano sensory
of stretching and movements? receptors of the dermis detect tension and
RESPONSE: Dense irregular connect torque?
tissue on the reticular layer. RESPONSE: Ruffini corpuscles

7. There describes keratinocytes, EXCEPT: 15. Which among the layers is found only in the
RESPONSE: Stem cells thick skin?
RESPONSE: S. lucidum
16. Cold air came in contact with the skin and 7. Which tissue is found in both male and female
produced tiny bumps. What is responsible urethra?
for this phenomenon? A. Stratified squamous
RESPONSE: Contraction of arrector pili B. Stratified columnar
muscle C. Smooth muscle
D. Pseudostratified columnar
17. A tissue section collected from a healing
wound was examined by an intern. He 8. The urothelium is made up of these cells except:
noticed proliferation of spindle-shaped cells A. Low columnar cells
and cuboidal cells in the stages of mitosis. B. Basal cells
How will he report this tissue section? C. Low cuboidal cells
RESPONSE: Fibroblast proliferates and basal D. None of the choices
cells undergo reepithelialization
9. Which of the following segments of the male
URINARY SYSTEM urethra is correctly matched?
1. Cells of the distal convoluted tubules that RESPONSE: Spongy urethra - Stratified
come in contact with the arteriole becomes squamous epithelium
which structure?
RESPONSE: Macula densa 10. The collecting system of the renal tubules are
made up of which cells?
2. Which of the following is not a blood vessel? RESPONSE: Cuboidal to columnar cells with
A. Vasa recta distinct cell membranes
B. Peritubular capillaries
C. Glomerulus 11. Uroplakins are an important feature in which
D. None of the choices part of the urinary system?
A. JGA
3. A 15 year old male with kidney disease B. Renal pyramids
underwent biopsy that confirms the damage C. Bladder mucosa
disrupts normal glomerular filtration. Which of the D. Collecting ducts
following structures will be abnormal in the TEM of
this boy’s biopsy? 12. What tissue is located around the renal capsule
which protects the kidney from mechanical shock?
RESPONSE: Glomerular basement membrane RESPONSE: Dense CT

4. Which of the following structure and function is 13. An MLS student examined the glomerulus and
correctly paired? noticed an increased number of darkly stained cells
RESPONSE: Principal cells - water filtration resembling vascular pericytes. What is your
conclusion regarding this biopsy?
5. What structures passes through the hilum of the RESPONSE: Increased mesangial cells due to
kidney? bacterial glomerulonephritis
RESPONSE: All of the choices
14. Which of these organs are lined by the
6. A 50 year old man sent a urine sample to the urothelium?
laboratory for routine urinalysis. The Medical A. Renal calyx
Technologists on duty tested 4+ on the protein B. Urinary bladder
parameter. Histologically Speaking what would C. Prostatic urethra
allow increase protein in the urine? D. All of the choices
RESPONSE: Damaged filtration membranes
15. Blood in the arcuate artery flows next to which
vessel? 23. Which of the following cells are rich in
A. Interlobular artery aquaporins that are specific channels for water?
B. Afferent arteriole RESPONSE: Principal cells
C. Interlobar artery
D. Efferent arteriole 24. What epithelium comprises the thick ascending
limb of the Loon of Henle?
16. The muscularis of the urinary bladder is best RESPONSE: Simple cuboidal
described by which of the following?
A. Seen distinctly at the neck of the bladder 25. A 45 year old developed kidney stones from
B. Three well-delineated layers calcium oxalate crystals. In which of the following
C. Has an inner, outer and longitudinal layers structures in the renal medulla begins this stone
D. All of the choices formation?
RESPONSE: Collecting tubules
17. Which gives the urethra its distinctive
appearance in a cross-section? 26. Which of the following is not a function of the
A. Stratified columnar epithelium mesangium?
B. Large longitudinal folds in the mucus a. Secretion of cytokines
C. Fibromuscular tube b. None of the choices
D. All of the choices c. Physical support of the capsule
d. Phagocytosis
18. Which structure delivers urine directly to the
minor calyx? 27. The blood filtration unit of the kidney consists of
A. Connecting tubule the following EXCEPT?
B. Duct of Bellini RESPONSE: Mesangial cells
C. Collecting tubule
D. Collecting duct 28. Which of the following is not normally found in
the filtrate?
19. Which area of the nephron facilitates active a. Amino acid
reabsorption of sodium? b. Glucose
RESPONSE: LOOP OF HENLE c. All of the choices
d. RBC
20. Which cell comprises the visceral layer of the
Bowman’s capsule? 29. The ultrastructure of the PCT is characterized
RESPONSE: Mesangial cell by which of the following?
a. Numerous vesicles
21. The slit diaphragms are characterized by which b. All of the choices
of the following? c. Prominent brush border
a. Forms the filtration membrane d. Membrane invaginations
b. All of the choices
c. Contains nephrons 30. The umbrella cell is characterized by which of
d. Specialized occluding junctions the following?
a. Large & bulbous
22. What hormone is produced by Juxtaglomerular b. Superficial
cells during hypotension or hyponatremia? c. All of the choices
a. Renin d. Binucleated
b. ACE
c. Angiotensin 31. Which of the following statements is true of
d. Aldosterone medullary rays?
a. They contain PCT 5. Which statement is true about
b. They do not extend to renal cortex hypothalamic-hypophyseal tract?
c. They contain arched collecting tubules A. All of the choices
d. They may lie at center of renal tubule B. Superior hypophyseal arteries supplying
blood to the neurohypophysis
32. A renal biopsy examined showed infiltrate C. Plexus of capillaries that carries the
containing immune cells among tubules having hormones
cells with prominent brush borders. Which of the D. bundle of axons that courses into the
following structures is being described? neurohypophysis
RESPONSE: PCT
6. Which cell is a modified sympathetic
33. What is the process by which the nephron postganglionic neuron?
removes water and solutes from the tubular fluid A. Chromophil
and returns them to the circulating blood? B. C cell
RESPONSE: Glomerular filtration C. Chromophobe
D. Chromaffin cell
ENDOCRINE SYSTEM
7. What cell secretes melatonin?
1. Which blood vessel forms the A. Pinealocyte
hypothalamic-hypophyseal portal system? B. Herring bodies
A. All of the choices C. Melanocyte
B. Superior hypophyseal arteries D. Astrocyte
C. Inferior hypophyseal arteries
D. Primary plexus 8. A student skipped breakfast to study for his final
examination. He felt hungry and tired from lack of
2. In hypersecretion of the thyroid hormone, sleep but pushed on. Which among the following
follicular cells are cuboidal in shape. hormones may be active given this situation?
in hyposecretion of the thyroid hormone, C cells 1. Cortisol
are flattened. 2. Epinephrine
A. Both statements are incorrect 3. Glucagon
B. Only the first statement is correct 4. Insulin
C. Both statements are correct
D. Only the second statement is correct A. 1,3
B. 1,2,3
3. To maintain uterine contractions, what endocrine C. 1,2,3,4
tissue will release its hormone? D. 2,4
A. Pars nervosa
B. Pars intermedia 9. A man drove fast in order to be on time for his
C. Pars tuberalis laboratory check-up. Upon testing. One hormone
D. Pars distalis was elevated in the hormone panel. In what tissue
is this hormone secreted?
4. Secretion, chemical modification & storage, A. Adrenal medulla
reuptake and digestion of proteins occur in what B. Thyroid follicles
structure? C. Anterior pituitary
A. Thyrocytes D. Pinealocytes
B. Melanocyte
C. Pituicyte 10. Which among the following zones of the
D. Chief cells adrenal cortex is responsible for the reabsorption of
sodium ions?
A. Zona reticularis 17. Modified aldehyde fuchsin stain shows these
B. Adrenal medulla cells with cytoplasmic granules that are deep
C. Zona glomerulosa brownish purple. What hormone is secreted by
D. Zona fasciculata these cells?
A. Glucagon
11. A 3 year-old boy had just been recently B. Insulin
diagnosed with Diabetes Type 1. Which among the C. Somatostain
following endocrine structures may be affected? D. PP
A. Delta cells
B. Beta cells 18. What appears as rounded clusters of cells
C. Alpha cells embedded within the pancreatic acini?
D. PP cells A. Pancreatic duct
B. Medulla
12. Which among the following glands perform in C. Cortex
tandem in the regulation of the calcium ions in the D. Islets of langerhans
body?
A. Adrenal cortex and adrenal medulla 19. A tumor was incised in the thyroid gland. Tissue
B. Thyroid gland and parathyroid gland sections convict of proliferation of large cells which
C. Anterior pituitary gland and posterior are stained less intensely by H & E. which of the
pituitary gland following occurred due to the presence of these
D. Pancreatic A cells and pancreatic B cells cells?
A. Increase metabolism due to intense
13. What is the parenchyma of the thyroid gland? production of thyroid hormones
A. Parafollicular cells B. Hypocalcemia induced by too much
B. None of the choices secretion of calcitonin
C. Follicles C. All of the choices
D. Thyrocyte D. Elevated iodide uptake and thyroglobulin
synthesis
14. In which part of the pancreatic islets are B cells
located? 20. Which of the following statements correctly
A. Head of pancreas describes the Pineal Gland?
B. None of the choices A. Abundant acidophilic cells with short
C. Periphery processes
D. Central B. Presence of variously sized concretions of
sodium and potassium salts
15. Which part is made up of nervous tissue? C. Produces hormones regulating the
A. Pars intermedia circadian rhythm
B. Infundibulum D. None of the choices
C. All of the choices
D. Pars distals

16. A 10 year old girl measures 2 feet and 3 inches


tall. Which of the following cells are responsible for
her dwarfism?
A. Somatotrophs
B. Lactotrophs
C. Gonadotrophs
D. Corticotrophs
REPRODUCTIVE SYSTEM
9. A man with pituitary gonadotrophic tumor caused
1. While studying a germ cell line, a researcher hypersecretion of FSH. Which condition will be
noted cells in metaphase have 45 chromosomes. exhibited?
From within the male genital tract would you expect RESPONSE: Sperm production is decrease
the cells to originate?
RESPONSE: Basal lamina of seminiferous 10. MAria gave birth to a baby girl. HEr doctor
tubule advised breastfeeding her baby as soon as
possible. Which histologic picture is associated with
2. A seafarer submitted a semen sample for lactation
analysis. Under the microscope morphology is RESPONSE: Enlarged tubulo-alveolar ducts
normal by spermatocytes seem to move slowly. with secretion
Which statement explains this condition?
RESPONSE: Seminal vesicle provides little 11. Which of the following glands are tubuloacinar
amount of fructose in morphology pruoducing mucus that coats and
lubricates the urethra?
3. Which of the following is correctly matched RESPONSE: Cowper’s gland
during Spermatogenesis
RESPONSE: Maturation Phase- shedding of 12. What is the site of physiologic exchanges
residual body between the mother that consists of a chorion and
a decidua basalis?
4. Which among the following structures contain the RESPONSE: Placenta
oocyte?
RESPONSE: Follicle 13. Which of the following is the tissue covering of
the testes?
5. A 29-year old man complained of testicular pain RESPONSE: Dense CT
and a burning sensation during urination. If the pain
is caused by inflammation at the channel between 14. Which of the following cells produces
the testis and epididymis. What is the tissue of the testosterone?
part that is affected? RESPONSE: Leydig cells
RESPONSE: Simple cuboidal epithelium
15. Interns are examining a prepared stained slice
6. A 28 year old woman is 16 weeks pregnant. She of an ovary. What will they observe under the
was rushed to the emergency room because of microscope if they are focusing on the cortex?
abdominal blood spotting and pain in her lower RESPONSE: Simple cuboidal epithelium
abdomen. Blood was drawn and progesterone and
HCG levels were discovered below normal range.
What histologic changes will occur due to the
decreased level of hormones?
RESPONSE: Functional layer sloughing off

7. What cells support and protect developing


spermatogenic cells?
RESPONSE: Sertoli cells

8. Which among the following male reproductive


organs is primarily responsible to produce fructose
as an energy source?
RESPONSE: Seminal vesicles
------MODULE 1------ The micrograph above shows an osteoclast in an empty
space. What do you call the empty space found in the bone
When moving the slide to the left, the image moves to which
and cartilage that contains an individual cell? LACUNA
direction? - to the Right
What phase of mitosis is seen in the image? PROPHASE
Locate the part of the microscope used to adjust the distance
between the eyes for better viewing. – A (diopter) What staining of nuclei is describes if a tighter packed of DNA
is seen? HETEROCHROMATIN
A type of microscope that uses the differences in the phase
of light reflected by a specimen. – Phase-contrast Seen is a cross section of a peripheral nerve. At what color of
Microscope the arrow is perineurium pointed? YELLOW ARROW

Identify the location where a hole through which the A deficiency of vitamin C in the diet can lead to scurvy. Above
transmitted light reaches the stage. – B (Aperture?) is a histologic micrograph of a patient with scurvy, which of
the following best describes the connective tissue involved?
The total magnification of a microscope when using a high-
WEAKENED COLLAGEN AND CONNECTIVE TISSUE
power objective. Given that the ocular lens alone has 10x
magnification power. – 400x What color of the arrow points to the component of the cell
that is stained by a positively charged dye? YELLOW ARROW
Artifacts are structural abnormalities that are not often seen
in living tissue. Pointed by an arrow is an example of what What type of epithelium lines the villi of the jejenum?
artifact? – Cracks SIMPLE COLUMNAR EPITHELIA

This type of artifact can create artificial spaces between cells Identify what epithelia are seen on the duct’s surface of the
and other tissue components. Such spaces ……. An example esophagus?
of this is seen in the micrograph below. –
STRATIFIED CUBOIDAL EPITHELIA
Shrinkage/Precipitates
Above is a micrograph of a developing bone. Which of the
Locate the part of the microscope that moves the stage up
following is true based on the micrograph shown?
and down quickly when using LPO. – D (coarse)
SHOWS THE MOST COMMON TYPE OF CARTILAGE
A histotechnologist was tasked to prepare a tissue to be
(HYALINE) WITH A FEW PERICHONDRIUM
examined by the pathologist. Upon finishing the
preparation, it was examined but was immediately rejected Ma. Dothea, a 4-year-old patient is suffering from a high
by the pathologist due to the artifact shown below. What fever for 3 days associated with bone pain and bleeding. The
likely was the cause of the problem? – The fixative isn’t physician ordered a dengue test and CBC. Laboratory results
properly buffered or thoroughly removed…… it may also showed a decrease in platelet count and tested positive for
occur of the dye is improperly prepared Dengue IgM. Which of the following WBC is expected to rise
during the infection?
Here is a micrograph of a cardiac muscle with a magnification
of 4x. Identify how this tissue was sectioned. –

Referring to the micrographs below, which is true about


histology sections? – All of the choices are correct

What evidence can you present for the blood vessel below
sectioned in a transverse or cross-section? – The plane is
round and it discloses the correct size and shape of the
blood vessel (?)

--------MODULE 2--------

In patients with scleroma, the


immune system triggers other
cells to produce too much
collagen. This extra collagen,
which causes hardening and
thickening. Which of the
following cells produce the
collagen?
Which of the following epithelial tissue given is paired with 2.) What is the thickest histologic layer of the
its correct definition? organ shown in the micrograph? Myocardium

3.) Which of the following arrows is pointed into


a monocyte? Blue arrow

4.) Why is the cytoplasm of the cell shown in the


micrograph is basophilic? Large number of azurophilic
Identify the function of the cell shown in the micrograph. granules

LIPIDS ARE STORED AS A SINGLE, NON-MEMBRANE BOUND


DROPLET IN THESE CELLS

A medical technologist is staining a peripheral blood smear


to be read by a pathologist. What do you think is the effect
of a basic dye to the red blood cells?

THE BASIC DYE WILL GIVE NO EFFECT TO THE RBCs 5.) All of the following describes this organ
except? The red pulp contains the PALS
The micrograph above is a part of connective tissue which
takes part in:

PHAGOCYTOSIS

What does the image above (Labelled 3) represent?

LAYERS OF BONE DEPOSITED IN THE SPACES REMAINING 6.) What is the immediate precursor of the cell
AFTER PRIMARY OSTEON FORMATION encircled? Polychromatic erythroblast
The nervous system has a variety of specialized receptors.
Which of the following images is correctly paired to its
definition?

BOTH IMAGE A & B

Which of the following best describes the micrograph shown 7.) Which of the following is correctly matched?
above? Red arrow: Proerythroblast

ALL OF THE CHOICES

--------MOD 3-------------

1.) Which of the following is true regarding the


encircled structure in the micrograph? Region of small
lymphocytes
13.) Which of the following correctly describes this
micrograph of an artery from an individual with
8.) What type of blood vessel is shown in the
atherosclerosis? None of the statement is correct
micrograph? Large vein

14.) Which of the following is true about this cell?


9.) Which of Plasma cells: secrete large quantities of proteins
(antibodies)
the following correctly describes this cell? Contains major
basic proteins and is a phagocytic cell

15.)Which of the following arrows point to a region where


it is populated mainly by T cells? RED ARROW
10.)

Which of the following is false regarding this micrograph?


Individual endothelial cells here have large perforations
without diaphragm

11.) The structure is found in what lymphoid 16.) Identify the blood cell shown in the picture.
MONOCYTES
organ? Thymus

12.) What can you predict when you see the


abundance of these cells under a microscope? The 17.)Which of the following is true with cells in the
patient is suffering from acute inflammation micrograph? ANG MAY LIGHT

GRAY IN COLOR
18.)What is the structure pointed in the arrow? DOHLE
BODIES

22.)Identify the cell below. BASOPHIL

19.)Identify the lymphoid organ shown in the micrograph.


TONSIL

23.)The most abundant tissue element forming the tunic of


the aorta pointed with the arrow is: ELASTIC FIBERS

20.)Which of the following is true about this blood cell? ALL


OF THE CHOICES

24.)Which of the following blood cells are involved in


adaptive immune system?

21.)One of the following is false about the cells shown in


the micrograph. CONSIDERED AS TRUE CELLS THAT

CONTAIN NUCLEI AND CELL Ans.

ORGANELLES 25.)The tunic pointed with the arrow is composed of what


type of tissue? Dense irregular CT
26.)This precursor cell matures into what blood cell?
NEUTROPHIL

4. Why is the structure pointed by an


arrow below significant?
ANS: IT PROVIDES MUSCLE MOTOR INNERVATION
OF THE MUSCULARIS MUCOSAE EXTERNA. FOR
THESE NERVE CELLS…

27.)Which is true of the structure of the

micrograph? THESE ARE SECONDARY NODULES WHERE B


CELLS MATURE INTO PLASMA CELLS.

5. What structure us pointed at by the


arrows in the micrograph? ANS:
PANETH CELLS

6. Refer to the micrograph below to


distinguish which among the
statements is false about the
micrograph
ANS: THE SMOOTH MUSCLE FIBERS WILL PRODUCE
RHYTHMUV MOVEMENT OF THESE STRUCTURES…

------- Module 4 -------

1. The mucosa of the stomach is lined


with that epithelium ANS: SIMPLE
COLUMNAR EPITHELIUM 7. All of the following tissues are visible
in the micrograph except
ANS: KERATINIZED STRATIFIED SQUAMOUS
EPITHELIUM

2. What is the structure seen at the


center of the micrograph ANS:
CENTRAL VEIN

8. The micrograph shows what


structure of the duodenum ANS:
BRUNNERS GLAND

3. All the following are visible in tge


micrograph except? ANS: NONE OF
THE CHOICES
9. Which of the following structures is
seen in the micrograph? ANS:
INTESTINAL CRYPTS OF LIEBERKUHN

10. As seen on the micrograph below


describe the submucosa of the
esophagus
ANS: IT IS LINED WITH DENSE IRREGULAR
MODULE 5
CONNECTIVE TISSUE. IT IS ABUNDANT UN MUCUS
GLANDS. THE DUCTS HERE ARE MOSTLY Identify the to tissues shown in the micrograph. – Stratified
STRATIFIED CUBOIDAL. Squamous Nonkeratinezed and pseudostratified columnar
epithelium
(note: basaha niyo tarong kay may same sentence
daan feel ki nagmali lang si maam) Which of the following doesn’t possess a hyaline cartilage? –
Brochiole

A pulmonary infection like tuberculosis is characterized pathologically by


necrotizing granulomas…... What type of tissue is seen in a patient
suffering from tuberculosis? – Produces scar tissue of collagenous fiber
instead of normal tissue formation

All of the following are functions of the cells visible in the micrograph,
except? – Apoptosis of cells

11. Choose the best statement in A biopsy result of a patient working in a gold mine…... Which of the
comparing chief cells from parietal as following is responsible for the discoloration? – Discoloration is due to
shown on the micrograph ANS: CHIEF the abundance of alveolar phagocytes

CELLS HAVE BASAL NUCLEI. PARIETAL Identify the tissue and organ shown in the micrograph. – Elastic
CELLS HAVE CENTRALLY LOCATED Cartilage; Epiglottis
NUCLEI What cell types are found in the micrograph above? – Ciliated
Pseudostratified Columnar with Goblet cells

What epithelium is shown in the micrograph? – Respiratory Epithelium

A 45-yo male was diagnosed with emphysema which is a chronic lung


disease caused by cigarette smoking. Which of the following would be
12. Amedical student on a rotation in the observed in the micrograph of a cigarette smoker? – Pseudostratified
pathology department was given an ciliated epithelium becomes stratified squamous epithelium
unlabelod tissue slide (shown below) Infant respiratory distress syndrome is the leading cause of death……
to whom she was tasked to kdeny by How will you differentiate type II alveolar cells from type I alveolar cells?
gastroenterologist from the cancer – Type II alveolar cells have lamellar bodies which contain closely
patient he is attending. As the slacked parallel membrane lamellae
medical student evaluated the tissue These non-ciliated cuboidal cells found in the bronchiole epithelium and
slide, she found that f was the are thought to be involved in absorption and secretion. – Basal cells
esophagus. Based on the
micrograph, what was the data that
the medical student used to make
the conclusion? ANS: ALL OF THE
CHOICES
The arrow in the micrograph indicate: Keratohyalin These structures can be found in what layer of the skin?
granules of stratum granulosum Reticular layer of the dermis

This part of the hair contains basal epithelial cells that


Refer to the micrograph to choose whether statements are
produce progenitor cells for the matrix of a new hair bulb.
true or false. Both statements are TRUE
Green Arrow

Here is a micrograph of the sebaceous glands wherein its The structure in the micrograph can be found in: Dermal
secretion is expelled by: The contraction of erector pili papillae
muscles

Which of the following arrow points to melanin-producing


cells? Yellow Arrow What is the function of the structure shown in the
micrograph? Detect firm touch

Which of the following statements correctly describes the


structure in the micrograph. I & IV

Which of the following statements is/are correctly


described in the micrograph? II & III

This is a thin layer which is composed of loose CT, with


types I &II collagen fibers. Fibroblast and scattered mast
cells, dendritic cells and leukocytes. Papillary layer of the
dermis
This part of the epidermis is where synthesis of keratin and
other proteins takes place. B ?
What region of the kidneys is seen in the
What is the function of the cells pointed by the
micrograph?
arrow?
ANSWER: RENAL MEDULLA
ANSWER: THEY COVER AND PROTECT THE
UNDERLYING EPITHELIAL CELLS

Between two images, which mucosal layer of


the bladder is in relaxed state? Identify the structure indicated by the black
ANSWER: IMAGE A arrow

ANSWER: ARCUATE ARTERY

Identify the region where water reabsorption in


response to the hormone vasopressin and
sodium reabsorption occurs.
Name the cell that is indicated by the question ANSWER: C
mark.

ANSWER: PODOCYTE

Compare the 1st tubule and the 2nd tubule

ANSWER: 1ST TUBULE IS NEAR THE RENAL


The following are functions of this structure
CORPUSCLE; 2ND TUBULE IS AWAY FROM THE
present in the kidneys, EXCEPT.
RENAL CORPUSCLE
ANSWER: IT DRAINS URINE INTO THE
NEPHRONS
What are the factors that decreases the
filtration of the structure with the black line?

ANSWER: ALL OF THE ABOVE


What structure in the corticomedullary junction Urinary
is seen in the micrograph? 1. The diagram shows the point of
entry for renal artery, renal vein,
ANSWER: ARCUATE ARTERY and ureter - true
2. How will you describe the
arrangement of smooth muscles
present as three layers in the
muscularis of the ureters? - inner
longitudinal, middle circular, outer
longitudinal
3. Which statements is/are correct
about the type of epithelium shown
in the micrograph? - it is found only
What connective tissue lines the capsule of the in the ureter and the bladder
4. Compared to the blood plasma
kidney?
osmolarity (~300 mOsm), what is the
ANSWER: DENSE IRREGULAR CONNECTIVE osmolarity of the interstitial fluid
within the indicated areas - iso-
TISSUE
osmotic
5. Which of the indicated tubules is
generally the MOST permeable to
water? - E
6. What cell in the glomerulus has a
large ovoid nucleus and forms a
narrow slit - PODOCYTE
7. In what region of the kidney is the
medullary rays located? - cortex
Evaluate the difference between relaxed and 8. Seen in the micrograph the blue
distended state of the urinary bladder. colored fiber lining the outer layer of
the bowman's capsule is called -
ANSWER: TRANSITIONAL CELLS BECAME COLLAGEN FIBER
SQUAMOUS IN A DISTENDED STATE
duct

9.
10. Identify the structure indicated by the
black arrow -medullary collecting 11. The structure beneath the epithelium
of this mucosal layer is called -
LAMINA PROPRIA
ENDOCRINE

5.)
1.) All of the following are types of the cells
Describe the predominant cell seen in the indicated in the micrograph, EXCEPT?
micrograph. SECRETES THE SOMATOTROPHS
PARATHYROID HORMONE

6.)
Identify the majority of cells in the
parathyroid. CHIEF CELLS
2.)
FOLLICULAR CELLS

7.)
3.) The regulation of blood glucose is a typical
Which definition describes the cells seen in example of negative feedback. The
the micrograph? DILATIONS OF AXONS structure illustrated in the micrograph has
FILLED WITH NEURO-SECRETION two major cell types that synthesize 2
VESICLES different hormones which work together to
maintain blood glucose levels in a narrow
range. The function of one hormone is to
INCREASE blood glucose. What is the
stimulus for the secretion of that hormone?
A LOW BLOOD GLUCOSE LEVEL

4.)
What do you call the cells that secrete
catecholamines that are present in the
adrenal medulla? CHROMAFFIN CELLS

8.)
Which of the following is shown in the
micrograph? CHIEF CELLS
13.)
9.) Evaluate the following micrographs of the
The regulation of blood glucose is a typical endocrine organs; select which is
example of negative feedback… The INCORRECTLY paired to the hormones
function of one hormone is to DECREASE they secrete. PARATHYROID HORMONES
blood glucose. What is the stimulus for the
secretion of that hormone? A HIGH BLOOD
GLUCOSE LEVEL

14.)
Select the cell that is correctly paired with
the hormones they secrete. CELL A-
GLUCAGON
10.)
What protein-filled components are seen in
the interface between the anterior &
posterior lobes of the pituitary gland?
RATHKE’S CYSTS

15.)
Enclosed in yellow circles, simple cuboidal
to columnar cells seen are what specific
cells of the thyroid glands? FOLLICULAR
CELLS
11.)
Evaluate which is INCORRECTLY labeled.
CHROMAFFIN CELLS

16.)
Which of the following zones is shown in the
micrograph? ZONA RETICULARIS

12.)
All of the following cells are visible in the
micrograph, EXCEPT? NONE OF THE
CHOICES
17. What structure in the pancreas contains
most of the endocrine cells?

BASOPHIL
ISLET OF LANGERHANS
21.which of the following is indicated in the
18. Differentiate the cells of the structure a
micrograph?
with structure b.

Structure A: paraFollicular cell is active


Structure B: paraFollicular cell is inactive

19. which of the following is indicated in the


micrograph?
ACIDOPHIL

22. which of the following zones secretes


the precursor of testosterone, the cells have
a centrak nycleus and eosinophilic
cytoplasm?

PARAFOLLICULAR CELL

20. Which of the following is indicated in the


micrographs? LAYER D
23.all of the following are hormones 26.differentiate correctly the two cells
produced by the structure presented in the pointed?
micrograph, except:

NOREPINEPHRINE
Blue is basophil that scretes acth; yellow
24.based on the micrograph, what would be is an acidophil that secretes gh
the physiological result of hypoactivity of the
layer of cells within bracket? 27. all zones of the adrenal cortex are seen
in the micrograph, except:

ADDISON’S DISEASE
ZONA MASCULARIS
25. which of the following is indicated in the
micrograph? 28. what type of connective tissue lines the
capsule of the thyroid gland?

FIBROELASTIC CONNECTIVE
TISSUE
OXYPHIL CELLS
29. all of the following cells are visible in the 32. which of the following is indicated in the
micrograph, except: micrograph?

CHROMOPHOBES CHROMOPHOBES

30. what cells are most abundant in the pars 33.all of the following are types of the cells
distaliss of the anterior pituitary gland? indicated in the micrograph, excpt:

SOMATOTROPHS SOMATOTROPHS

31. which of thw following zones is shown in


the micrograph?

ZONA GLUMERULOSA
1. The structure in the micrograph can be 5. Which of the following is a benign
found in ANS: TAIL OF THE EPIDIDYMIS cervical lesion and represent mucinous
cysts due to (…)
ANS: PICTURE

2. Which of the following cell types


commonly have prominent lipid
droplets in their cytoplasm? ANS:
PICTURE

6. What is the secretory product of the


epithelium shown in the micrograph?
ANS: GLYCOPROTEIN

3. Identify the structure shown in the


micrograph ANS: RETE TESTIS

7. Identify the function of the muscle layer


of the mammary gland ANS: THE
CONTRACTION OF THIS LAYER ALLOWS
4. Which of the following correctly THE ERECTION OF THE NIPPLE
describes the organs in the micrograph?
ANS: D. secretes fluid that is composed
of fructose, sodium, fibrinogen, and
prostaglandins
8. Which of the following statements is 11. Which statement best describes the
true based on the micrograph below? micrograph below. ANS: THE SLIDE
ANS: STATEMENT 1 IS FALSE. SHOWS A COLUMNAR EPI W
STATEMENT 2 IS TRUE MICROVILI THE SECRETES ESTROGEN
(…)

9. The structure in the micrograph is 12. Which of the following statements best
found in what organ ANS: SEMINAL supports the cell pointed by the black
VESICLES arrow? ANS: THESE CELLS ARE
REMOVED FROM THE OOCYTE WHEN
OVULATION OCCURS

10. What would be the effect of the


parasympathetic stimulation with the
structures in the micrograph? ANS:
DILATE, INC BLOOD FLOW AND FILLING
CAVERNOUS SPACES
13. Which of the following statements is
true regarding the structure in the
micrograph? ANS: I & III
14. What arrow points to a glycoprotein 18. The structure pointed with the arrow is
layer (…) ANS: BLUE ARROW taken from the testis which is also seen
in what female reproductive organ?
ANS: OVARY

15. Identify the structure shown in the 19. Which of the following arrows point to
micrograph ANS: MUCOUS GLAND the cell that acts as a blood-testis
barrier? ANS: YELLOW ARROW

16. Which organ in the micrograph can you 20. Which of the following is responsible for
find the interstitial cell of Leydig? ANS: supplying vital proteins to the oocyte
A and leaves the ovary with the oocyte
during ovulation? ANS: PICTURE

17. Which of the following arrows points to


a spermatid ANS: BLUE ARROW
REPRODUCTIVE SYSTEM PT.2

Which of the following is false regarding theb structure


shown in the micrograph? Consist of folds that are lined
with SSE cells rich in secretory granules
The metaplasia seen in the micrograph could be in what part of
the cervix? Endocervix/Cervical Canal

Which of the following is true regarding the cells shown in


the micrograph? Both statement are true

Identify the cell pointed with the yellow arrow. Sertoli


Cells

Which of the following incorrectly describes the struicture


in the micrograph? III only
A 40 y/o male desires to undergo bilateral vasectomy which
is a most common method of male contraception. This
procedure involves excising a portion of which organ shown
in the micrograph? D Which of the following structures grow and expand during
pregnancy and lactation? C

All the following are accessory organs of the male


reproductive system, except? C

The micrograph below is part of the endometrial layer of


the uterus. Describe the changes seen. There are presence
of monocytes in the lamina propria and indicates severe
bacterial infection

Identify the phase shown in the micrograph. In this phase,


spiral arteries intermittently close causiong ischemia.
After several days, the spiral arteries remain constricted
and the lack of oxygen leads to necrosis

Which of the following best describes the micrograph


below? (or above haha) The cells will develop into corpus
albicans

What portion of the uterus is retained during


menstruation? B
---------INTEGUMENTARY SYSTEM------- The skin has no sweat glands.

What consists of loose connective tissue that binds the False


skin loosely to the adjacent organs? These are tissues found in the deeper layer of the
Subcutaneous Tissue DERMIS except:

It has a pale-staining, rounded cell body with irregular ANG MAKITA SA DIRI NGA LAYER ARE MGA:
cytoplasmic extensions.
dense irregular connective tissue surrounding hair
Melanocyte follicles,
What is the layer of the skin that contain the veins sebaceous glands and sweat glands,
needed for blood collection?
nerves,
Dermis
deep plexus of blood vessels extending into
What are branched acinar cells embedded in the dermis subcutaneous layer
with holocrine secretion?
It is a dendritic cell found in stratum spinosum.
Sebaceous Glands
Langerhans Cell
What describes keratinocytes?
Eccrine sweat glands empty into hair follicles

False
Cells that produce the pigment for dark hair are located
in where? Which of the following components of the epidermis
provides sealant between adjacent cells?
Hair Matrix
Glycolipids and lipids
The presence of what tissue typifies the papillary dermis?
What cells derive from precursors originating in the bone
More superficial layer of dermis; marrow and function as antigen-presenting cells in skin?
composed of areolar connective tissue; Langerhans Cells
forms dermal papillae; What typifies reticular dermis but not the papillary
contains subpapillary vascular plexus dermis?

What are the encapsulated mechanosensory receptors Dense Irregular CT


of the dermis that detects tension and torque? What best characterizes sebaceous glands?
Ruffini Corpuscles It releases its contents via holocrine secretion
What is the tissue that allows the skin to retain its shape Myoepithelial cells aid in the secretory process of what?
regardless of stretching and movement?
Eccrine Sweat Glands
Dense irregular CT
A 52y/o woman presents with severe blistering over her
Intercellular bridges (desmosomes) are characteristic of buttocks. Analysis of her serum demonstrates the
what stratum? presence of antibodies which by immunohistochemical
Spinosum (?) techniques stain material located at the basement
membrane of the epidermis in a biopsy of her skin. The
underlying biological mechanism of her skin disorder
involves an abnormality in where?

Hemidesmosomes
A 64 y/o woman, who has alays been proud of her Important target cells of leptin are found in which
suntanned, healthy look, is referred to a dermatologist organ?
with a blue-violet, painless, 1.5-cm lump in the skin of her
Hypothalamus
left shoulder. The lump is firm and cannot be moved, and
has grown very rapidly over the past few weeks. The The hormone-sensitive lipase in the cells of adipose
mass is removed surgically and the pathologist diagnoses tissue acts primarily on what substrate?
it as a Merkel cell carcinoma. If the UV radiation to which
her skin was exposed affected he Merkel cells, what Triglycerides
other cell type sharing the same specific epidermal layer Applied to adipocytes, the term "multilocular" refers to
might also be affected? which of the following?
Basal stem cells for keratinocytes The large number of small cytoplasmic lipid droplets
A 37 y/o woman presents with a suspected in terms of adipocytes, multilocular refers to the large
Schwannoma. The radiology report indicates “a soft number of small cytoplasmic lipid droplets.
tissue mass to the right of L1 t the level of the L to L2
neural foramen.” The neurologist presses the base of a Fully differentiated white adipocytes are large cells,
vibrating 128 cps tuning fork to the skin of the patient’s typically having diameters of approximately what size?
right and left thighs and asks her to describe the 100 μm
sensation. She asks the patient to close her eyes and then
to tell her whether the tuning fork is vibrating or not. Ten days after birth a full-term newborn boy develops
With that instrument the doctor is primarily testing the firm, erythematous nodules and plaques over his trunk,
function of which sensory receptors? arms, buttocks, thighs, and cheeks. His mother's
pregnancy was complicated by placenta previa and his
Lamellated (Pacinian) Corpuscles airway was cleared of aspirated meconium immediately
Which of the following separates the hair follicle from after birth. A biopsy of subcutaneous tissue shows
the connective tissue of the dermis? necrosis within the brown adipose tissue. What
metabolic activity is liable to be affected in this patient?
Glassy Membrane
Oxidation of fatty acids for thermogenesis
White adipocytes are derived developmentally from
what precursor cells? A 44-year-old African-American woman visits her family
physician for a physical examination at the urging of her
Mesenchymal cells husband. She has no current complaints and is taking no
What are the relatively large particles formed in the medications. She is allergic to erythromycin. She works
intestinal epithelial cells and rich in ingested lipids? as a software developer and lives with her 52-year-old
husband and 12-year-old daughter. She is a nonsmoker
Chylomicrons and drinks an occasional glass of wine when she and her
husband go out to dinner. She is involved in no regular
What substance, released from the adrenal gland and
exercise. Her mother is 66 and suffers from type II
some autonomic neurons, increases lipolytic activity in
diabetes, hyperlipidemia, and hypertension and had a
white adipocytes?
myocardial infarction last year. The patient's father died
Norepinephrine of a stroke last year at the age of 72. On examination,
the patient's blood pressure is 155/100 mm Hg, pulse
What is the most important form of lipid storage in both
84, weight 215 lb (increased from 180 lb 3 years ago),
white and brown adipocytes?
and height 5 ft. 7 in. In this patient, during the period of
Triglycerides weight gain which one of the following responses would
be most likely in her white fat?

Increased release of norepinephrine from nerve


terminals near white adipocytes
In addition to providing body insulation and maintaining A 16-year-old girl on the high school athletic swim team
energy homeostasis, adipose tissue is considered to be is concerned about the effects of water on her skin. She
an important organ for which of the following biological asks you whether she should apply creams or lotions to
functions? waterproof her skin before entering the pool. You
explain to her that the skin itself serves as an effective
Endocrine secretion
water barrier to protect the body. Which of the
During a dermatopathology conference, you are asked following components of the skin is most important in
about differences between thick and thin skin. Which of establishing an effective water barrier?
the following epidermal layers is a unique feature of
Cell and lipid envelopes
thick skin?
You are invited to talk to elementary school students
Stratum lucidum
about your interest in biology and medicine. One of the
The parents of a 2-year-old girl are concerned about students asks you to explain why his best friend has
their daughter's white scaly skin. Physical examination dark skin. Which of the following biological processes is
reveals coarse, fish-like scales on the child's forearms. responsible for darker pigmentation of the skin in those
The patient's father is afflicted with the same condition of African descent?
(ichthyosis vulgaris). A skin biopsy from this child would
Slower degradation of melanin
most likely show which of the following histopathologic
findings? A 28-year-old white woman returns home following a 4-
week summer vacation at the beach. Her friends and
Excessive thickening of the stratum corneum
family comment that her skin has become much darker
in color. Which of the following biological processes
accounts for this change in skin coloration?
A 2-day-old neonate cries violently whenever she is
breast-fed. Physical examination of the neonate reveals Increased production of melanin
numerous blisters in the oral mucosa and on the skin.
A 48-year-old man complains of "poison ivy." Physical
Microscopic examination of a skin biopsy reveals
examination reveals skin rashes and blisters on his
separation between upper and lower portions of basal
upper limb and trunk. Which of the following cells
cells in the epidermis. The results of immuno-labeling
internalize and present "poison ivy" antigens to
assays are negative (i.e., they do not show evidence of
lymphatic cells of the immune system in this patient?
immune complex deposition in the skin). Which of the
following is the most likely diagnosis? Langerhans cells

Epidermolysis bullosa simplex The skin is richly innervated by sensory nerve fibers and
contains a variety of sensory receptors. Which of the
A 64-year-old farmer presents with multiple patches of
following epidermal cells is closely associated with
discoloration on his face. Biopsy of lesional skin reveals
terminal nerve fibers and serves as a mechanoreceptor
"actinic keratosis." This pathologic condition represents
in the skin?
abnormal maturation of which of the following skin
cells? Merkel cell

Keratinocytes A 4-year-old girl is rescued after becoming lost outdoors


during a winter snowstorm. Physical examination
indicates that the child is fine, except for exhaustion
and a few skin sores related to hypothermia. Which of
the following biological processes was chiefly
responsible for conserving this child's core body
temperature during her winter trek?

Opening of arteriovenous anastomoses


A 32-year-old Asian man notices that he is losing hair Which of the following statements about eccrine sweat
and is concerned about becoming bald. You talk to him glands is true?
about the mechanisms that regulate hair growth. Cells
They have a narrow duct lined by a stratified cuboidal
in which of the following regions of the hair follicle are
epithelium
directly responsible for the growth of hair?
Which of the following statements about hair follicles is
Hair matrix
true?
A 12-year-old girl suffers second-degree burns on her
They are present in thin skin but not in thick skin
left arm while helping her father in the kitchen.
Regenerative stem cells that migrate extensively to Which of the following statements concerning skin
resurface this patient's skin wound originate from which melanocytes is true?
of the following cutaneous structures?
They synthesize a pigment that protects against
External root sheath damage caused by UV radiation
A 15-year-old girl presents with severe acne. Physical Which of the following is an appendage of skin?
examination reveals many discrete papules and
pustules on her face and neck. Acne is a chronic Nail
inflammatory disorder that principally affects which of Stimulation of sympathetic nerves to brown fat would
the following cutaneous structures? lead to each of the following EXCEPT
Sebaceous glands increased intracellular triglyceride
During a 3rd year clinical rotation, you are asked to All the following are TRUE regarding white adipose
discuss the biological functions of the skin. tissue cells (adipocytes) EXCEPT
Thermoregulation is the primary function of which of
the following epidermal appendages? they increase in number during normal weight gain in
the adult
Eccrine sweat glands
The intercellular spaces in the stratum granulosoum of
Intercellular bridges are characteristic of which of the the epidermis contain lipid containing sheets that are
following layers of the epidermis? impermeable to water. This material is released from
Stratum spinosum which of the following?

Which of the following statements concerning the Lamellar bodies


stratum granulosum is true? A 35-year-old female presented with a blister in the sole
It contains keratohyalin granules. of her right feet. A skin biopsy was done and was
reported that there was a separation of the epidermis
Which of the following statements about Langerhans from the dermis, what likely structure is disrupted?
cells is true?
Hemidesmosomes
They play an immunological role in the skin.
Which of the following is best described as an
Meissner corpuscles are present in which of the elongated, encapsulated receptor found in the dermal
following regions of the skin? pegs of glabrous skin and is especially abundant on lips
and fingertips?
Dermal papillary layer
Meissner's corpuscle
Which of the following statements concerning thin skin
is true?

It contains hair follicles


Pain receptors in the skin are typically classified as
which of the following?

Free nerve endings

Which of the following best describes an expanded tip


tactile receptor found in the dermis of hairy skin that is
specialized to detect continuously applied touch
sensation?

Merkel's disc

Vibratory sensation is dependent on the detection of


rapidly changing, repetitive sensations. The high-
frequency end of the repetitive stimulation scale is
detected by which of the following?

Pacinian corpuscles
A 15 y/o male with kidney disease underwent biopsy that
-----URINARY SYSTEM---- confirms the damage disrupts normal glomerular
Distal is smaller than proximal renal tubule filtration. What structures will be abnormal in the TEM of
this boy’s biopsy?
Distinguish rbc in tubule by the nucleus (?)
Glomerular Basement Membrane
What is a fibromuscular tube with large longitudinal
folds around the lumen? What layer cannot be found in the ureter?

Urethra Submucosa/middle layer

What are the darkly stained cells resembling vascular What segments of the male urethra is correctly
pericytes in the glomerulus? matched?

Mesangial Cells Spongy urethra - Stratified Squamous Epithelium

What organ is lined by the urothelium? What epithelium composes the thick ascending limb of
the loop of Henle?
Urinary Bladder
Simple Cuboidal Epithelium
What part of the nephron facilitates active reabsorption
of sodium? The mucosa of the urinary bladder folds extensively and
umbrella cells decrease their surface area. This condition
Proximal Tubule indicates an ____ bladder.
Uroplakins are important feature of what cell? EMPTY BLADDER
Umbrella Cells The blood filtration unit of the kidney consists of the
What cells are rich in aquaporins that are specific following EXCEPT:
channels of water? Mesangial cells
Principal Cells Which of the following statements is true of medullary
A renal biopsy examined showed infiltrate containing rays?
immune cells among tubules having cells with prominent They may lie at center of renal lobule
brush borders. What is being described?
Blood in the renal arcuate arteries flows next into which
PCT vessels?
What is the process by which the nephron removes Interlobular Arteries
water and solutes from the tubular fluid and returns
them to the circulating blood? What cell is modified smooth muscle cell that secretes
renin?
Tubular Reabsorption
Juxtaglomerular Cells
What cell comprises the visceral layer of the Bowman’s
capsule? Epithelial cell membrane domains containing many
stiffened plaques of protein are an important feature in
Podocytes which part of the urinary system?
What structure and function are correctly paired? Bladder mucosa
Lacis cells-cytokine secretion
An immunohistochemical technique using antibodies Which type of epithelium lines the thick ascending limb
against aquaporins to stain a section of kidney would be of the loop of Henle?
expected to stain cells in which structures most
Simple cuboidal
intensely?
Which cell is a modified smooth muscle cell that
Collecting Ducts
secretes renin?
What type of epithelium lines the prostatic urethra?
Juxtaglomerular cells
Transitional (urothelium)

A 14 y/o patient presents in the nephrology clinic with Epithelial cell membrane domains containing many
fatigue, malaise, anorexia, abdominal pain, and fever. stiffened plaques of protein are an important feature in
She reports a loss of 6 lb in the past 2 months. A renal which part of the urinary system?
biopsy is prepared for light microscopy, and an infiltrate
Bladder mucosa
containing lymphocytes, plasma cells, and eosinophils is
found among tubules having cells with prominent brush An immunohistochemical technique using antibodies
borders. What statement correctly pertains to these against aquaporins to stain a section of kidney would be
epithelial cells? expected to stain cells in which structures most
intensely?
The primary site for the reduction of the tubular fluid
volume Collecting ducts
A 45 y/o man presents with nephrolithiasis or kidney What type of epithelium lines the prostatic urethra?
stones. The process of calcium oxalate stone formation
as seen in this patient begins with Randall plaques found Transitional (urothelium)
in the basement membranes of which structure that is A 14-year-old patient presents in the nephrology clinic
only found in the renal medulla? with fatigue, malaise, anorexia, abdominal pain, and
Collecting Ducts fever. She reports a loss of 6 lb in the past 2 months.
Serum gamma globulin and the immunoglobulins IgG,
A 15 y/o male presents with hematuria, hearing loss, lens IgA, and IgM are all elevated. Her serum creatine is 1.4
dislocation, and the onset of cataracts. Genetic analysis mg/dL (normal 0.6-1.2 mg/dL) and urinalysis of glucose
reveals a mutation in the COL4A5 gene. Transmission EM and protein are 2+ on a dipstick test, confirmed by
examination of a renal biopsy confirm that the disorder laboratory at 8.0 g/dL and 0.95 g/dL, respectively. A
has affected a component of the renal corpuscles in renal biopsy is prepared for light microscopy, and an
which damage disrupts normal glomerular filtration. infiltrate containing lymphocytes, plasma cells, and
Which structure would most likely be abnormal in the eosinophils is found among tubules having cells with
TEM of this patient’s biopsy? prominent brush borders. Which one of the following
statements correctly pertains to these epithelial cells?
Glomerular Basement Membranes
The primary site for the reduction of the tubular fluid
Blood in the renal arcuate arteries flows next into which
volume
vessels?
A 45-year-old man presents with nephrolithiasis or
Interlobular arteries
kidney stones. The process of calcium oxalate stone
Which cell type comprises the visceral layer of Bowman formation as seen in this patient begins with Randall
capsule? plaques found in the basement membranes of which
one of the following structures found only in the renal
Extraglomerular mesangial (or Lacis) cells
medulla?

Collecting ducts
A 15-year-old male presents with hematuria, hearing Which type of epithelium lines the urinary bladder?
loss, lens dislocation, and the onset of cataracts.
Transitional
Genetic analysis reveals a mutation in the COL4A5 gene.
Transmission EM examination of a renal biopsy confirms The basal lamina of the glomerular filtration barrier is
that the disorder has affected a component of the renal cleared of cellular debris and antigen-antibody
corpuscles in which damage disrupts normal glomerular complexes by the phagocytic activity of which cell
filtration. Which one of the following structures would types?
most likely be abnormal in the TEM of this patient's
biopsy? Mesangial cell

Glomerular basement membranes Which term refers to cortical tissue between the
medullary pyramids?
Which feature is characteristic of the renal cortex but
not of the renal medulla? Renal lobes

Contains the medullary rays Which type of epithelium lines the thick ascending limb
of the loop of Henle?
Which vessels lie at the border between the renal
cortex and medulla? Simple cuboidal

Arcuate arteries and veins Which type of epithelium lines the proximal convoluted
tubule?
The efferent arterioles of the juxtamedullary renal
corpuscles connect glomerular capillaries with which Simple cuboidal
vessels? Which cell type is the modified smooth muscle cell that
Vasa recta secretes renin?

Blood in the arcuate arteries next flows into which Juxtaglomerular cell
vascular channels? Which type of epithelium lines the thin loop of Henle?
Interlobular arteries Simple squamous

Which cell type comprises the visceral layer of


Which structures comprise a renal lobule? Bowman's capsule?
Medullary ray and all nephrons that empty into it Podocyte
Which feature is characteristic of proximal but not of Which feature is characteristic of cortical, but not
distal convoluted tubules? juxtamedullary, nephrons?
Exhibits a brush border Loops of Henle have long, thick descending limbs
Which feature is characteristic of the thin loops of Henle Which type of epithelium comprises the parietal layer of
but not of the vasa recta? Bowman's capsule?
Ascending portion is impermeable to water Simple squamous
Which type of epithelium lines the prostatic urethra? Voluntary control of urine flow in males involves neural
Transitional stimulation of which structure?

Which feature is characteristic of the internal, but not of External urethral sphincter
the external, urinary sphincter?

Consists of smooth muscle


---------ENDOCRINE SYSTEM-------- ______ and _____ are the cells that secretes hormones
to regulate calcium secretion.
Hypothalamus is not a gland but a region the brain
Parathyroid Hormone
Hypothalamus is made up of interconnecting neurons
Follicles are the parenchyma of the thyroid gland with
It is not a gland bcos it is not made by epithelial tissues these cels called thyrocytes.

ADH is produced by the hypothalamus in the brain and The pinealocyte secretes a hormone that generally slows
stored in the posterior pituitary gland at the base of the metabollic activity at night
brain.
Cells that secrete the hormone responsible for
ADH is normally released by the pituitary in response to reabsorption of sodium is located in the zona
sensors that detect an increase in blood osmolality glomerulosa
(number of dissolved particles in the blood) or decrease
A man drove fast and furious. The hormone that is likely
in blood volume.
to be elevated is produced by the _____ cells of the
ADH is synthesized by neurons in supraoptic _____
nuclei/nucleus in hypothalamus
Chromaffin cells ,Adrenal medulla
Nuclei – PNS – gongranucleun (?)
(hindi ko sure kay dira daan ga produce ang adrenalin)
The synthesized products are then transported down the
The ____ is the thick middle layer of large, spongy-
axons of these neurons into the posterior pituitary
looking cells secreting cortisol
(neurohypophysis)
Zona Fasciculata
Precursor: Forerunner
The ____ produces ADH and Oxytocin
- an inactive substance converted to an active one
(such as an enzyme, vitamin, or hormone) Posterior Pituitary (Produces both)
Thyroglobulin - a precursor protein present in thyroid Supraoptic Nucleus (produces ADH)
gland, from which hormones are synthesized
Paraventricular Nucleus (produces oxytocin)
What part of the hypophysis is not glandular in origin?
Pituicytes are glial cells that resemble astrocyted in the
PARS NERVOSA neurohyphysis
Modified aldehyde fuchsin stains what cells of the The thryrocytes’ shape range from ___ to ___
pancreas with cytoplasmic granules that are deep
brownish purple? Squamous to low columnar

granules in the peripheral α cells To maintain uterine contraction, what part of the pars
nervosa will release its hormones?
__ cells lowers blood sugar and are located at the _____
of the Islets of Langerhans? Herring bodies

B cells, center Bella is 12 years old but measures 3 feet tall. Her
condition is due to hyposecretion of?
Prolactin is synthesized by the lactotrophs in the anterior
of pituitary gland. somatotropic cells

Which of the following accurately describes


glucocorticoids?

Are produced in response to stimulation by ACTH


Pregnant women who have begun labor but in whom A 45-year-old corporate executive presents with the
this process is no longer progressing are often given an primary com plaint of “always being tired.” She
IV injection of Pitocin to stimulate uterine contractions comments that she has been tired for 4 months even
and facilitate parturition. Pitocin is a trade name of a though she is sleeping more. She complains of being
hormone produced in what endocrine tissue? unable to finish chores at home and “dragging at work.”
She indicates that she is often constipated and is
Pars nervosa
intolerant of cold. She is continuously turning the
What hormone is produced in response to decreased thermostats in the house and office to higher
blood calcium levels temperatures, to the dismay of family and coworkers.
She also complains that her skin is very dry; use of
PTH lotions and creams have not helped the dryness. A
Addison disease (or adrenal cortex insufficiency) is a biopsy of her thyroid gland shows dense lymphocytic
disorder, usually autoimmune in origin, which can cause infiltration with germinal centers throughout the
degeneration and cell loss in the adrenal glands. parenchyma and a battery of further tests is carried out,
Fludrocortisone is a mineralocorticoid used to treat which also suggest thyroiditis. Which of the following
Addison patients. Response to this drug indicates that results of blood tests would be most likely in this
which region of the adrenal glands was involved in the patient?
disease? Elevated TSH levels
Zona glomerulosa A 9-year-old girl, the youngest of four daughters, is
A glucagonoma is a malignant tumor consisting of what taken to the pediatrician by her mother, who indicates
cells that for at least 4 months the child has seemed
“hyperactive,” unable to sleep soundly because “she
A or α cells says her room is too hot,” and no longer able to
Secretion, chemical modification and storage, reuptake, concentrate in school. Upon questioning, the mother
and digestion of a protein occur in epithelial cells of also remembers that her daughter’s periods also began
what endocrine tissue? within the past few months. Blood tests indicate high
levels of estrogen-related hormones and cortisol. Which
Thyroid gland of the following tentative diagnoses is consistent with
all of these symptoms?
Secretion in what neuroendocrine cell is controlled
directly by neural activity and involves a hormone that A benign tumor involving cells in the adenohypophysis
generally slows metabolic activity at night?

Pinealocyte

Some mammalian endocrine tissues or cells can be


experimentally transplanted to other well-vascularized
sites (such as the oral mucosa) in genetically similar
hosts and the tissue’s function continues normally and
with proper regulation. The pars distalis is not a good
candidate for such transplantation studies for which
one of the following reasons?

The vascular wall of the superior hypophyseal arteries


is unique.
-----REPRODUCTIVE SYSTEM----- What is made up of tortuous tubes that secretes fluid
containing fructose?
Important functions of accessory glands, are semen
production, lubrication, and keeping the sperm cells Seminal Vesicles
moving and alive What consists of chorion & decidua basalis?
Ectocervix is the part of cervix where the papsmear is Placenta
performed. (Stratified squamous epithelium)
What gland is producing mucous that coats & lubricates
Endocervix is the upper part of the cervix that is lined by the urethra?
a simple columnal epithelium, that contains mucous-
secreting cell Bulbourethral Glands

During the Luteal Phase, the follicle that burst and What cells produce testosterone?
released the egg (during ovulation) develops into a Leydig
small yellow structure, or cyst, called the corpus
luteum. What cells support & protect developing spermatogenic
cells?
The corpus luteum secretes progesterone and estrogen
that cause the uterine lining, or endometrium, to Sertoli
thicken and be able to nourish a fertilized egg.
While studying the germ cell line, the cells seen are in
Lipid milk is release through the _____mechanism from metaphase & contain 46 chromosomes. From which
the secretory _______ part did the cells come from?

Apocrine, Alveoli Seminiferous tubules

What changes will occur when progesterone levels is Which of the following accurately describes
decreased? spermiogenesis?

Functional layer sloughs off Involves cytodifferentiation of early spermatids

What contains the oocyte A man with a pituitary gonadotrophic tumor causing
hyposecretion of FSH is most likely to exhibit which
Which of the following occurs during the luteal phase? condition?
Corpus Luteum Formation Low sperm count
What is the stage of ovarian follicle development Interstitial cells of Leydig have an important function in
characterized by initial period of fluid accumulation?
male gamete production. Because of this function,
Secondary Follicle which of the following organelles is abundant within
these cells?
The ______ is made up of ______capsule that covers
the testis Smooth endoplasmic reticulum

Tunica Albuginea, Dense CT While studying a germ cell line developed from a
patient’s tes ticular biopsy, the researcher notes that
Fertilization occurs in the ____ lined by ____ colchicine-treated cells blocked in metaphase have 46
epithelium. chromosomes. From which of the following regions of
Ampulla, simple columnar the male genital tract would you expect these cells to
have originated?

At the basal lamina of the seminiferous tubule


Which of the following organs is normally characterized Which stage of ovarian follicle development is
by the accumulation of corpora amylacea with characterized by an initial period of follicular fluid
increasing age accumulation?

Prostate Secondary follicle

Within the male reproductive tract, stereocilia project Which of the following is characteristic of granulosa
from cells lining which of the following regions lutein cells?

Epididymis Secrete progesterone

As sperm pass through the male genital ducts, proteins Which of the following hormones is primarily
and low molecular-weight products are added from responsible for inducing ovulation?
several sources producing semen. Which of the
Luteinizing Hormone LH
following provides a nutritive, fructose-rich secretion?
Which feature is characteristic of the corpora albicans
Seminal vesicles
but not of atretic follicles?
A 20-year-old man contracts cholera during a long tour
Resemble large collagenous scars
of military duty in a remote, completely undeveloped
region. After a 5-day period of severe diarrhea and Endometrial glands are typically most fully developed
treatment, he gradually recovers and slowly returns to and filled with product during which day(s) or phase of a
work. He is married 3 years later but after a few years of woman’s menstrual cycle?
trying to conceive a child, semen analysis reveals that
his sperm are few in number and malformed, and blood Days 15-28
tests show a high titer of antibodies against sperm Which feature is characteristic of the endometrium’s
antigens. The causative agent of cholera, Vibrio cholera, basal layer but not of its functional layer?
secretes a toxin that interferes with tight (occluding)
junctions. What cells in the gonad are the likely target Contains cells that replace the surface epithelium after
of this toxin in the male reproductive system? menstruation

Sertoli cells Most lipid in milk is released from cells by which


mechanism?
A 29-year-old man presents with testicular pain and a
burning sensation during urination. Tests reveal the Apocrine secretion
presence of Neisseria gonorrhea and penicillin is A 33-year-old woman with an average menstrual cycle
prescribed. Gonorrhea often produces acute or chronic of 28 days comes in for a routine Pap smear. It has been
inflammation of the testes and frequently involves the 35 days since the start of her last menstrual period, and
channels that connect the testis to the epididymis. a vaginal smear reveals clumps of basophilic cells. As
What is the name of these channels? her physician you suspect which of the following?
Efferent ductules There will be detectable levels of hCG in her serum and
A 39-year-old man undergoing an extensive series of urine.
tests for infertility is found to have a genetic mutation
preventing formation of a functional synaptonemal
complex during meiosis, causing almost complete
failure of sperm formation. Which cells would be
directly affected by this mutation?

Primary spermatocytes
A 17-year-old girl with a history of pelvic inflammatory
disease presents at the emergency department with
severe pain in her lower right side that came on fairly
quickly. Upon questioning she replies that her last
menstrual period was 6 weeks ago and that she has
never missed a period before. The doctor suspects she
has an ectopic pregnancy and this is quickly confirmed
by ultrasound testing. The surgeon removes her right
uterine tube that is inflamed, scarified, and contains the
implanted embryonic tissue in the region where
fertilization normally occurs. Where is this?

The ampulla region with highly folded mucosa

A 42-year-old woman visits her physician complaining of


recurrent vaginal yeast infections. The doctor explains
the likelihood that the woman’s vaginal lining is
temporarily out of proper acid-base balance, leading to
the increased susceptibility to yeast infections. The
normally low pH in the vagina is maintained by which of
the following?

Bacterial metabolism of glycogen to produce an


organic acid

You might also like